a notas de algebra moderna

262

Upload: davirael-r-ramirez

Post on 10-Aug-2015

81 views

Category:

Documents


6 download

TRANSCRIPT

Page 1: A Notas de Algebra Moderna
Page 2: A Notas de Algebra Moderna

INDICE 1. LAS PROPOSICIONES DE LA TEORIA DE NUMEROS. 2. OTROS PROBLEMAS FAMOSOS. 3. LO NECESARIO Y LO SUFICIENTE. 4. PRIMERAS PROPOSICIONES SOBRE DIVISIBILIDAD 5. ALGUNOS PASATIEMPOS IMPOSIBLES 6. TRES PROPIEDADES DE LOS ENTEROS POSITIVOS 7. SUMATORIA Y MULTIPLICATORIA 8. EL MAXIMO COMUN DIVISOR 9. ECUACIONES LINEALES DIOFANTINAS 10. LA RELACION DE CONGRUENCIA ENTRE ENTEROS 11. NÚMEROS, NUMERALES Y SISTEMAS DE NUMERACIÓN 12. NUMERACIONES POSICIONALES 13. CRITERIOS DE DIVISIBILIDAD Y CONGRUENCIAS 14. TODO DEPENDE DE SABER CONTAR 15. ALGORITMOS BASADOS EN EL SISTEMA DE NUMERACION ANEXOS: A1. EL MÉTODO AXIOMÁTICO A2. AXIOMAS DE LOS ENTEROS A3. EL PRINCIPIO DEL BUEN ORDEN

Page 3: A Notas de Algebra Moderna

1. LAS PROPOSICIONES DE LA TEORIA DE NUMEROS.

Indudablemente la base de todo ese inmenso océano que es la matemática radica en la

aritmética y la geometría. Nuestro estudio se centrará en la primera de éstas que trata,

como la Teoría de Números, de los números naturales y enteros. Se trata de analizar

proposiciones que se refieren a éstos.

El lector entonces, debe tener confianza en el estudio que inicia pues trabajará básicamente

con los elementos con los cuales indudablemente debe estar familiarizado. Sin embargo el

exceso de confianza no es conveniente pues, como comenta James R. Newman: "Se

supone comúnmente que la aritmética es la rama más sencilla de las Matemáticas. Nada

más lejos de la verdad. El tema es difícil de plantear aunque se admite que la práctica de la

aritmética elemental es bastante fácil". En efecto, comprobar la falsedad o veracidad de

proposiciones como 22+23=45 ó 425×236=263020 es una tarea de la aritmética elemental

que puede resolver cualquier alumno de tercero primaria. No se necesita más que el simple

manejo de algoritmos1 y‚ esto es lo que en la mayoría de los casos se enseña en la

matemática de la escuela primaria. La justificación de éstos y otros algoritmos, así como la

verificación de propiedades más generales exigen casi siempre mayor madurez matemática.

Así los griegos pitagóricos sabían que la suma de los n primeros números impares

coinciden con n2. Este hecho se comprueba fácilmente para casos particulares. Por

ejemplo, los primeros 4 números impares son: 1,3,5 y 7 cuya suma es 16. ¿Pero cómo

comprobar que en todos los casos se cumple?

1 Un algoritmo se puede entender como una receta o procedimiento para efectuar una tarea.

Page 4: A Notas de Algebra Moderna

Podríamos sumar los primeros 100 impares desde 1 hasta 199 y obtener 10.000 o

comprobar con ayuda de modernos computadoras para números muy grandes y muchas

veces, obteniendo siempre resultados positivos y sin embargo siempre faltarían casos y no

podremos estar convencidos absolutamente de que la proposición analizada sea cierta para

todo número natural. Con ayuda de la intuición geométrica podemos convencernos de la

certeza de la proposición tal y como razonaban los primeros matemáticos. Observando la

figura 1, se nota cómo al ir añadiendo impares siempre se obtiene un cuadrado de lado igual

al número de impares que se llevan.

Aunque este argumento no es una demostración en el sentido moderno, es indudable que es

una muy buena explicación que nos exime de cualquier intento de comprobación con

muchos números.

Vemos entonces que no vasta sumar, restar, etc. para analizar la verdad o falsedad de las

proposiciones aritméticas, y que para hacerlo es necesario utilizar otras ramas del saber

matemático, como en el caso expuesto en el cual nos hemos ayudado con la geometría. En

realidad son pocas las ramas de las matemáticas que no se utilizan en lo que hoy se

denomina "Teoría de Números" que es al fin y al cabo una aritmética avanzada y que como

se comentaba anteriormente es mucho más difícil de lo que en principio puede uno suponer.

Para dar una dimensión de los problemas centrales de esta rama de la matemática -que no

se trabajarán en este texto-, comentaremos por ahora el famoso "Último teorema de

Fermat" como un problema planteado hace más de tres siglos y que a pesar de los grandes

1 1+3 1+3+5 1+3+5+7 Figura 1. Argumentación geométrica para comprobar que la suma de los primeros

impares es un número cuadrado perfecto.

Page 5: A Notas de Algebra Moderna

esfuerzos dedicados y a los avances de otras ramas de la Matemática, no ha podido ser

resuelto.

Alrededor de 1637 Fermat, jurista y parlamentario francés cuya diversión era las

matemáticas y del cual el lector oirá hablar mucho, escribió al margen de un libro -la

Aritmética de Diofantus-: "Es imposible descomponer un cubo como la suma de dos cubos,

una cuarta potencia o en general cualquier potencia como la suma de dos potencias del

mismo orden mientras éste sea mayor que dos, y ciertamente he encontrado una

demostración magnífica de esto pero el margen es demasiado pequeño para contenerla".

En otras palabras lo que Fermat aseguraba tener demostrado era que la ecuación:

xn+yn=zn no tiene solución para x, y, z, enteros no nulos y siendo n un entero mayor que 2.

Esta sencilla proposición no ha podido ser demostrada ni refutada, a pesar del gran avance

de la matemática y de los grandes matemáticos que lo han intentado. Nótese que para n=2

existen muchas triplas de enteros x, y, z que resuelven la ecuación (por ejemplo, x=3, y=4,

z=5). Estas reciben el nombre de Triplas Pitagóricas.

Esta pequeña introducción ha querido mostrar al lector nuestro objeto de estudio, los

números enteros, objetos con los cuales estamos familiarizados desde niños y que a pesar

de haber sido trabajados por la humanidad por siglos y siglos contienen gran cantidad de

misterios. Por esto la Aritmética coronada por Gauss como la reina de las matemáticas es

al decir de Bell "el último gran continente salvaje de las matemáticas".

PREGUNTAS Y EJERCICIOS.

1. La suma de los primeros n números forman los números triangulares como se muestra

enseguida:

Page 6: A Notas de Algebra Moderna

a. "La suma de dos números triangulares seguidos da un número cuadrado perfecto".

Por ejemplo, l0+l5=52 mientras 10+6=42. Explique con un argumento geométrico

él por qué de esta proposición.

b. Completando el cuadrado demuestre que el n-ésimo número Triangular es n(n+1)/2.

2. Las propiedades que hoy en día se exponen sobre las operaciones entre números se

pueden argumentar de manera geométrica. Por ejemplo la famosa identidad

(a+b)2=a2+2ab+b2 se explica por medio de dibujo de la figura:

a. Argumentar de manera geométrica la propiedad distributiva: a(b+c) = ab +ac.

b. Lo mismo para las siguientes identidades: (a-b)b=ab-b2, (a+b)(a-b)+b2 = a2.

3. ¿Qué civilizaciones anteriores a la griega conocieron las triplas Pitagóricas?

4. Si x, y, z es una tripla pitagórica también son triplas pitagóricas sus dobles y en general

kx, ky, kz también para k entero positivo. ¿Por qué‚? La tripla kx, ky, kz es un múltiplo

de x, y, z. Muestre una tripla pitagórica que no sea múltiplo de la ya citada 3, 4, 5.

a b

a+b

b2

a2

Page 7: A Notas de Algebra Moderna

2. OTROS PROBLEMAS FAMOSOS.

En la aritmética como en la geometría los griegos constituyen la primera gran ruptura que

el hombre conoce en la historia de la matemática, pues no se conoce cultura anterior que

haya comprendido el concepto de 'demostración'. Más aun, la aristocracia griega

despreciaba la aritmética práctica que se aplicaba al comercio y que llamaban logística, tal

vez como reacción o muestra de admiración ante el gran edificio que se construía sobre la

aritmética teórica. Euclides en sus célebres ELEMENTOS DE GEOMETRIA dedica cuatro

de los trece libros a este tema. La siguiente proposición es un magnífico ejemplo de la

elegancia y acierto de la matemática helénica.

Proposición 1. Los números primos son infinitos.

Demostración. Supongamos lo contrario, es decir que sólo hay un número finito de primos

lo que implica que existe una enumeración de tales números digamos p1, p2, p3, ...,pn.

Consideremos el número m=p1×p2×p3×...×pn+1. Este número no puede ser primo por

cuanto estamos suponiendo que todos los primos son p1, p2, p3, ...,pn y m es mayor que

todos ellos, entonces m debe ser no primo y se debe dejar dividir por al menos uno de los

primos digamos pi. Pero esto tampoco es posible (¿Por qué?) Tenemos entonces que m no

es compuesto y tampoco es primo, contradicción que nos obliga a aceptar que hay infinitos

números primos.

La demostración de esta proposición es un bonito ejemplo del método de demostración por

reducción al absurdo que consiste en suponer que la conclusión del teorema no es cierta lo

cual nos lleva por medio de razocinios lógicos a algo que sabemos es falso, obligándonos a

aceptar la conclusión del teorema. Más adelante profundizaremos sobre este método de

demostración.

Page 8: A Notas de Algebra Moderna

A pesar de sus grandes avances las matemáticas griegas no fueron en ningún momento

acabadas por cuanto dejaron abiertos a la humanidad interesantes problemas que ellos no

pudieron resolver en torno a los cuales ha girado buena parte del trabajo posterior. De estos

problemas los que se refieren a la geometría han sido todos resueltos, en diferentes épocas y

tras laboriosos esfuerzos. Sin embargo, hay problemas de la aritmética que permanecen

después de dos mil años aun en el misterio.

Un ejemplo de estos misterios se refiere a los números perfectos: un número perfecto es

aquel que coincide con la suma de sus divisores positivos y menores que él. El primer

número perfecto es 6=1+2+3 y el segundo es 28=1+2+4+7+14. Todos los números

perfectos que se conocen desde los griegos hasta nuestros días son pares, pero nadie ha

podido demostrar que no existen números perfectos impares (¿habrá?) Este es el primer

gran interrogante, si existen o no perfectos impares, pero sobre los perfectos pares tampoco

se sabe mucho! Como se verá mas adelante Euclides demostró que todo perfecto par es de

la forma 2c(2c+1-1) en donde 2c+1-1 es primo. Esta es una caracterización de los perfectos

pares por cuanto todo número de esta forma es necesariamente un perfecto par. Surge

entonces el importante problema de saber cuantos números de la forma 2c+1-1 son primos.

En 1644 el fraile franciscano Martín Mersenne (1588-1648) aseguró que 2n-1 es primo

solamente para los primos n=2,3,5,7,13,19,31,67,127 y 257. En 1880 se demostró que para

n=61 se obtiene un número primo contradiciendo hipótesis de Mersenne, por lo cual se

supuso que 67 solamente era un error de algún copista negligente por 61. Pero en 1903,

Cole demostró que para n=67 se obtiene un número compuesto comprobándose

definitivamente que Mersenne se equivocó.

En 1947 se habían encontrado cinco fallas en la lista de Mersenne. Hoy en día se sigue

trabajando para conseguir un criterio que nos indique cuándo un número de la forma 2n-1,

con n primo, es primo. Además con la ayuda de los modernos computadores se conocen

más de 30 primos para los cuales el correspondiente número de Mersenne es primo. Por

ejemplo en 1988 se descubrió que para n=110503 el número de Mersenne correspondiente

(que tiene 33265 cifras decimales!) es primo.

Page 9: A Notas de Algebra Moderna

Muchos han sido los esfuerzos para encontrar una fórmula que produzca sólo números

primos. Buscando tal fórmula Fermat pensó erróneamente que los números de la forma:

122 +n

cuando n=1,2,3..., son primos. Se necesitó casi un siglo para que Euler demostrara que

para n=5 el número correspondiente de Fermat no es primo y sólo hasta 1880 Next

demostró que para n=6 la hipótesis de Fermat tampoco se tiene. Hoy en día se conocen

muchos números de Fermat que no son primos y se saben criterios para determinar cuando

un número de Fermat es primo. Pero los números de Fermat fueron más importantes de lo

que él mismo pensó. Gauss los utilizó para caracterizar los polígonos regulares que se

pueden construir con regla y compás, caracterización que en 1801 resolvía por completo

uno de los problemas planteados por los griegos referentes a la geometría.

Pero Fermat no sólo planteó problemas que han sido dolor de cabeza para los matemáticos.

Demostró también muchos resultados que hoy en día son clásicos. Por ejemplo los

números primos impares son de la forma 4k+1 o 4k-1; se puede demostrar por un método

parecido al de Euclides (proposición 1), que los primos de la forma 4k-1 son infinitos. No

tan fácil veremos más adelante que los primos de la forma 4k+1 también son infinitos.

Fermat demostró que todo primo de la forma 4k+1 se puede escribir como la suma de dos

cuadrados mientras que ningún primo de la forma 4k-1 se puede expresar así.

Otro de los resultados de este gran matemático es el llamado "Primer teorema de Fermat"

(o "débil”) que indica que si n es cualquier número que no se deja dividir por el primo p

entonces el número np-1-1 es divisible por p. Este resultado, piedra angular de la Teoría de

Números, junto con el teorema de Wilson serán demostrados más adelante. El teorema de

Wilson asegura que si p es primo el producto de los antecesores de p sumado con 1 es un

número divisible por p.

Refirámonos finalmente, a la conocida conjetura de Goldbach (1690-1764) quien en 1742

en una carta escrita a Euler, conjeturó que todo entero positivo par mayor que 2, es la suma

de dos primos y que todo entero positivo impar mayor que 5, es la suma de tres primos. La

Page 10: A Notas de Algebra Moderna

hipótesis para los números pares se ha comprobado para números menores que un millón

pero parece estar muy remota una prueba general.

Aunque no es nuestra intención atacar estos problemas famosos, esperamos que esta breve

incursión histórica de una idea de la dimensión que tiene esta rama de la matemática, de la

cual este libro no pretende ser sino una modesta introducción.

PREGUNTAS Y EJERCICIOS.

1. Explique cuales fueron las primeras actividades del hombre que lo llevaron a

profundizar en el estudio de las propiedades de los números.

2. Por qué‚ los primeros conocimientos sobre las propiedades de los números fueron

considerado mágicos y guardados en secreto por diferentes sectas religiosas? (Aún hoy

en día para mucha de ellas los números guardan ese carácter mágico).

3. Hacer una lista de los primeros cinco números perfectos pares.

4. Expresar como suma de cuadrados perfectos: l3, 29, 53, 101.

5. Demostrar agotando todos los casos que el primo 127 no se puede expresar como la

suma de dos cuadrados.

6. Demuestre que si n es par diferente de 2, 2n-1 no es primo.

7. Demostrar que 3428-1 es divisible por 29.

8. Se define n! (que se lee "n factorial") como el producto de todos los antecesores de n

incluyendo el mismo n, así:

5!=5×4×3×2=120

6!=?

Page 11: A Notas de Algebra Moderna

Formule el Teorema de Wilson en términos de factoriales. Qué se puede decir de la

siguiente proposición: "n siempre divide a n!".

Qué de la proposición: "si n es menor que m entonces m divide a n!". Compruebe el

teorema de Wilson para los primeros cinco primos.

9. Descomponer los siguientes pares como la suma de dos números cada uno primo: 86,

142, 210.

10. En qué‚ consiste la tabla de Eratóstenes para construir primos? Construir la tabla de

Eratóstenes para números menores que 200.

Page 12: A Notas de Algebra Moderna

1a matemática es la ciencia que obtiene conclusiones

necesarias B. Pierce-

3. LO NECESARIO Y LO SUFICIENTE

Supongamos que el lector está familiarizado con los conectivos lógicos que se dan entre

proposiciones, tales como la negación (∼), la conjunción (∧), la disyunción (∨), la

implicación y la equivalencia (⇒, ⇔), así como con los cuantificadores universal y

existencial. Diremos algo sobre la implicación y la equivalencia que nos sirve para analizar

ciertas técnicas de demostración.

La mayoría de las proposiciones de la matemática son de tipo "si p entonces debe cumplir

q", resumiremos p⇒q, en donde p juega el papel de hipótesis y q el de tesis o conclusión.

Se pueden dar diferentes versiones idiomáticas de éste conectivo lógico; en español se usa

"si p entonces q", "p implica q", "q siempre que p", "para que suceda p es necesario que q",

"para que q es suficiente que p", etc. Sea p, por ejemplo, la proposición "a y b son pares" y

q la proposición "a+b es par", p⇒q se puede leer: "Si a y b son pares entonces a+b

también lo es", como quien dice "la suma de dos pares es un par", o, "condición suficiente

para que a+b sea par es que a y b sean pares".

La recíproca de la proposición p⇒q es la proposición q⇒p que en general tiene diferente

valor de verdad. En el ejemplo anterior mientras "la suma de pares es par" es una

proposición cierta, su recíproca "si la suma de dos números es par entonces ambos números

son pares" es falsa, puesto que 5+3 es par siendo uno de los sumandos impar.

La contrarrecíproca de la proposición p⇒q es la proposición ∼q⇒∼p que es equivalente a

la original, por lo tanto, para demostrar una implicación podemos demostrar su

contrarecíproca.

Page 13: A Notas de Algebra Moderna

Así, para, mostrar que "todo par elevado al cuadrado es par" (n par implica n2 par) podemos

mostrar que "si el cuadrado de un número es impar, el número debe ser impar" (n2 no par

implica n no par) que es equivalente.

En el ejemplo inicial la contrarecíproca de "la suma de dos pares es un par" es la

proposición "si la suma de dos números no es par entonces ambos números no pueden ser

pares" o lo que es lo mismo "si la suma de dos números no es par entonces alguno de los

números es impar".

Muchas veces la hipótesis o la tesis viene en forma de conjunción o disyunción. Por

ejemplo la forma (p∧q)⇒r, que es la forma de la proposición que acabamos de analizar.

En efecto, si convenimos en que p, r, q sean las proposiciones "a es par", "b es par", "a+b

es par" respectivamente, se ve más claramente porqué la contrarecíproca tiene como

conclusión que alguno de los números es impar, ya que la negación de p∧q es ∼p∨∼q y la

forma de la contrarecíproca ser ∼r⇒ (∼q∨∼p).

Probar la contrarecíproca es hacer la prueba por contradicción: Para demostrar p⇒q se

supone que la conclusión no es cierta o sea ∼q y se deduce que la hipótesis fallaría o sea

que ∼p; se está demostrando que ∼q⇒∼p.

Otra propiedad que nos interesa resaltar de la lógica de proposiciones es que la negación de

una implicación p⇒q es equivalente a ∼(p∧∼q). Esta es la razón para que negar la

proposición "la suma de dos números es impar implica que ambos son pares", sea afirmar

que "existen números cuya suma es par sin que ambos sean pares". La equivalencia entre

p⇒q y ∼(p∧∼q) nos ayuda también a explicar las demostraciones por contradicción: Se

trata de ver que es imposible que se cumpla la hipótesis sin que se cumpla también la tesis.

Cuando tanto p⇒q como su recíproca q⇒p, son ciertas se dice que p y q son equivalentes y

se nota p⇔q. Por ejemplo, "n2 es par" y "n es par" son proposiciones equivalentes, pues

tanto "si n2 es par entonces n es par" como "si n es par su cuadrado también lo es" son

proposiciones ciertas.

Page 14: A Notas de Algebra Moderna

Otras versiones idiomáticas para esta equivalencia son: "a es par sí y sólo sí a2 lo es" o

"condición necesaria y suficiente para que n2 sea par es que n lo sea". La equivalencia

también se utiliza en las definiciones, por ejemplo para definir par podemos decir "n es par

sí y sólo sí existe un entero k tal que a=2k".

Las equivalencias lógicas (tautológicas) son válidas por su forma sin importar el contenido

de las proposiciones 'internas'. Así, "k no es primo par sí y sólo sí k no es primo o k es

impar", es una equivalencia válida por su forma pues ∼(p∧q)⇔(∼p∨∼q) es cierta sin

importar el valor de verdad de p y q. La tabla 1 muestra una lista de las principales

equivalencias lógicas. Digamos para terminar esta sección, que siempre que p y q sean

equivalentes la proposición p se puede reemplazar por q y el revés. La equivalencia es, con

respecto a las proposiciones, como una igualdad.

∼(∼p) ⇔ p Doble negación es afirmación.

p∧(q∧r)

p∨(q∨r)

(p∧q)∧r

(p∨q)∨r

La conjunción y la disyunción son

asociativas.

p∨(q∧r)

p∧(q∨r)

[(p∨q)∧(p∨r)]

[(p∧q)∨(p∧r)] Disyuntiva.

∼(p∧q) ⇔ (∼p∨∼q) Negación de ∧.

∼(p∨q) ⇔ (∼p∧∼q) Negación de ∨.

(p⇒q) ⇔ (∼q⇒∼p) La contrarecíproca es equivalente a la

original.

∼(p⇒q) ⇔ (p∧∼q ) Negación de la implicación.

((p∧q)⇒r) ⇔ ((p∧∼r)⇒ ∼q)

((p∧q)⇒r) ⇔ (∼r⇒(∼q∨∼q))

TABLA 1: Algunas equivalencias lógicas importantes.

Page 15: A Notas de Algebra Moderna

PREGUNTAS Y EJERCICIOS

1. Dar otras versiones idiomáticas de p⇒q. Cómo se dice en inglés?

2. Sean p,q,r,s las siguientes proposiciones:

p: x es par

q: y es par

r: x+y es par

s: x2+y2 es par

Según esto, identifique cada proposición de la izquierda con su respectiva fórmula a la

derecha.

a. Si x es par, y es par entonces x+y es par. a. (∼q∧r)⇒p

b. Es imposible que x2+y2 sea par, siendo x e

y impares.

b. ∼s⇒(∼p∨∼q)

c. Condición necesaria para que x2+y2 sea

impar es que alguno de ellos sea impar.

c. (p∧q)⇒r

d. Condición suficiente para que x sea impar

es que x+y sea par siendo e impar.

d. ∼(s∧∼p∧∼q)

3. La recíproca de la contrarrecíproca es equivalente a la recíproca. Dé un ejemplo.

4. Las siguientes proposiciones son todas falsas. Muestre en cada caso un contraejemplo:

a. Si a2 no es par a3 sí lo es.

b. n2+2n siempre es par.

c. Si n es primo 2n-1 también lo es.

Page 16: A Notas de Algebra Moderna

d. Si n es positivo n3-6n2+11n-6=0.

5. Expresar en forma de implicación el último teorema de Fermat.

6. ¿Cuál es el recíproco del teorema de Wilson?

7. De las proposiciones siguientes señale aquellas equivalencias al primer teorema de

Fermat:

a. Todo número primo p mayor que 2 divide a np-1-1 y no divide a n.

b. Siendo p un primo mayor que 2, si p no divide a pp-1-1 entonces p divide a n.

c. Si p es número mayor que 2 que no divide a n y no divide a np-1-1 entonces p no es

primo.

8. Cada proposición de la izquierda tiene una proposición a la derecha que es lógicamente

equivalente. Señálela:

a. Si un número es par su cuadrado

también lo es.

a. Condición suficiente para que el

cuadrado de un número sea par, es

que el número lo sea

b. La suma de dos impares es par. b. No hay impares de cuadrado par.

c. Si no es par no es múltiplo de 4. c. No existen impares cuya suma sea

impar.

d. Condición necesaria para ser par es

ser múltiplo de 4.

d. Los múltiplos de 4 son siempre

pares.

e. Todo número cuyo cuadrado es par,

es par.

e. Condición suficiente para ser

múltiplo de 4 es ser par.

Page 17: A Notas de Algebra Moderna

9. Analice la demostración de Euclides de la infinitud de los primos (proposición 1

sección 2) en términos de una implicación entre las proposiciones "A es el conjunto de

todos los números primos" y "A es infinito".

Page 18: A Notas de Algebra Moderna

4. PRIMERAS PROPOSICIONES SOBRE DIVISIBILIDAD

Demostremos a continuación algunas proposiciones más con el fin de familiarizar al lector

con el concepto de prueba que con la idea de avanzar en la teoría. Para ello nos

fundamentaremos solamente en las propiedades que se conocen de los números a través del

álgebra elemental. Empecemos por unas definiciones que despejan pequeñas dudas sobre

términos a emplear.

Definición 1. Un número n es par sí y sólo sí existe un entero k tal que n=2k. Se dice

también que n es múltiplo de 2 o que 2 divide a n.

n es múltiplo de m sí y sólo sí existe k entero tal que n=mk.

Se dice en este caso que m divide a n o que n es múltiplo de m y se nota m|n.

Un número p mayor que 1 es primo si sus únicos divisores positivos son 1 y el mismo p.

Caso contrario, p se dice compuesto.

NOTA: El 1 no se considera ni primo ni compuesto.

La demostración de las das primeras proposiciones son ejemplos de demostraciones

directas, que consisten simplemente en traducir las hipótesis para que después de una leve

manipulación algebraica se encuentre la tesis formulada.

Proposición 1. La suma de dos múltiplos de k es un múltiplo de k.

Demostración. Sean u y v los múltiplos de k, según la definición existen n y m tales que

u=nk y v=mk entonces

u+v=(n+m)k

Page 19: A Notas de Algebra Moderna

lo que nos indica que u+v es múltiplo de k, pues se puede expresar como un número (n+m)

multiplicado por k.

Proposición 2. Si m es múltiplo de k su cuadrado también lo es.

Demostración. Si m es múltiplo de k, existe un n tal que m=nk entonces m2=n2k2 lo que

implica que

m2=(n2k)k

o sea, que m2 también es múltiplo de k.

Corolario. El cuadrado de un par es par.

Demostración. Esto es sólo una particularización (cuando k=2) de la proposición 2.

Este corolario junto con la recíproca nos produce la proposición siguiente que

demostraremos por contradicción.

Aceptamos como cierto que cualquier número entero es par (de forma 2k) o bien impar (de

la forma 2k+1).

Proposición 3. Un número es par sí y sólo sí su cuadrado también lo es.

Demostración. Por el corolario anterior se tiene que si n es par su cuadrado también lo es.

Debemos demostrar que si n2 es par entonces n es par. Si no fuera así (suponiendo lo

contrario), es decir si n2 es par siendo n impar, entonces n sería de la forma 2k+1 y su

cuadrado ser de la forma 2(2k2+2k)+1, o sea, impar. Esto es contrario a lo supuesto,

concluimos entonces que n debe ser par.

Esta última demostración se puede hacer por el método directo (ejercicio). Muchas de las

proposiciones que se demuestran por contradicción se pueden demostrar directamente, sin

Page 20: A Notas de Algebra Moderna

embargo, muchas veces el absurdo es un método de demostración es irremplazable tanto

por la claridad en la exposición como por la dificultad para plantearla directamente.

Piénsese en la demostración de la infinitud de los primos dada por Euclides (proposición 1

sección 2) y nótese la dificultad (imposibilidad?) para ser hecha directamente. La siguiente

proposición es una de esas en donde para su demostración es claramente ventajoso utilizar

el método por reducción al absurdo (por contradicción).

Proposición 4. Si un número n mayor que 1 es compuesto entonces tiene por lo menos un

divisor ente 1 y n .

Demostración. Supongamos lo contrario, es decir que n es compuesto pero que todos sus

divisores no triviales (diferentes de n y 1) son mayores que n . Sea entonces k uno de

tales divisores, por definición existen tal que n=nk. Como m también es un divisor de n no

trivial por la suposición que hemos hecho tenemos

m> n y k> n

Según las leyes de las desigualdades se deduce que

mk> n n

o sea que n>n lo cual es imposible! Concluimos que n debe tener divisores no triviales

menores que n .

En lo que se sigue utilizaremos las siguientes definiciones:

Definición 2. Diremos que a y b son de igual paridad si ambos son pares o ambos

impares.

Si n es entero positivo se define n! (se lee n factorial) como el producto de n por todos sus

antecesores positivos (ver ejercicio 8 sección 2).

EJERCICIOS Y PREGUNTAS

Page 21: A Notas de Algebra Moderna

1. Demostrar que la suma de impares es par.

2. a. Demostrar que la diferencia de múltiplos de k es un múltiplo de k.

b. En la demostración de la infinitud de los primos (proposición 1 sección 2) ¿Dónde

se utiliza este hecho?

3. Demuestre que la diferencia de dos números es par sí y sólo sí ambos números son de

igual paridad.

4. Los únicos números que no tienen sino un divisor son ...

5. ¿Qué números son divisores de 0? ¿Qué números se pueden decir que son divisibles por

0 según la definición 1?

6. A es un conjunto de números, decimos que A es cerrada para cierta operación, si todo

par de éstos al operarlos producen un elemento de A. De ésta forma, la proposición 1

de esta sección prueba que los pares son cerrados para la suma.

a. Pruebe que los impares son cerrados para el producto, pero no para la suma.

b. Pruebe que los números de la forma 4n+1 no son cerrados para la suma pero si para

el producto.

7. Pruebe que los números de la forma 4n+2 son el doble de números impares.

8. Para cada una de las siguientes proposiciones decir si son falsas o ciertas, justificando

cada respuesta con una demostración o un contraejemplo según el caso:

a. n siempre es divisible de (n+1)!

b. Si n no es un primo entonces divide a (n-1)!

c. k divide siempre a cualquier múltiplo de k.

d. Si n es múltiplo de k entonces para cualquier h se tiene que k divide a nh

e. k siempre divide a nk

f. Los números de la forma 4n+2 son cerrados para el producto.

Page 22: A Notas de Algebra Moderna

g. Para todo entero n se tiene que n(n+1) es par.

h. Los números de la forma 2n son cerrados para la suma y para el producto.

i. Un número es divisible por 12 sí y sólo sí es divisible por 3 y por 2.

9. ¿Qué opina de la demostración siguiente, en donde se intenta probar el reciproco del

teorema de Wilson (ejercicio 6 sección 3)? Sea n un número mayor que 1 compuesto

entonces n divide a (n-1)! Hacemos m=(n-1)!+1. Si n divide a m entonces divide a la

diferencia m-(n-1)! (por dividir a ambos miembros), pero está diferencia es 1, lo que

nos lleva a una contradicción que nos obliga a aceptar que si n es primo n no divide a

(n-1)!+1. contradicción que nos obliga a aceptar que si n es primo n no divide

Page 23: A Notas de Algebra Moderna

5. ALGUNOS PASATIEMPOS IMPOSIBLES.

Hemos visto en el numeral anterior algunas propiedades de los números que se refieren a la

divisibilidad, especialmente por 2, de tal forma que podemos decir que dominamos la

aritmética de los pares e impares.

Vamos a mostrar sencillas aplicaciones de estos resultados para completar el vistazo

general que queremos dar al principio de este libro, aunque hacemos la aclaración que el

objetivo de éste no incluye mostrar las posibles aplicaciones de la aritmética y la teoría de

números, que son muchas. Las que trabajaremos aquí se refieren a algunos juegos o

pasatiempos en los cuales se descubre de manera contundente y con un análisis muy

simple, ciertos casos en los cuales no hay solución.

A llenar el cuadrado... Consideremos un rectángulo dividido en n×m cuadrados iguales. Por

ejemplo en la figura 1 el rectángulo se divide en 4×7 cuadrados. En un rectángulo de este

tipo se escogen dos cuadros arbitrarios denotados por I y F (inicial y final).

Se trata de partir del cuadro inicial para llegar

al final, moviéndose cada vez un sólo cuadro

pero siempre horizontal o verticalmente, de tal

forma que al final se haya pasado por cada uno

de los cuadros restantes una única vez.

Así en la figura 2 se muestra una solución

sencillísima que puede animar al lector para

que busque una solución cuando se trata de

ir de I a hasta F'. Si el lector no encuentra

una solución podrá ofrecer con toda

seguridad gran cantidad de dinero a algún

amigo para que le ayude a resolverlo.

I

F’ F

Figura 1.

Figura 2.

Page 24: A Notas de Algebra Moderna

En realidad, aunque el problema planteado

de ir de I hasta F sea tan fácil de resolver,

cuando se trata de ir desde I hasta F' es

imposible.

¿Por qué? Veamos: coloreando los cuadros a manera de un tablero de ajedrez con cuadros

blancos y negros (figura 3) se nota que para ir de un cuadro a otro del mismo color se

necesita un número par de pasos, es más: "para ir de un cuadro a otro se necesita un número

par de pasos si y solo si los cuadros son del mismo color". Además también es fácil darse

cuenta que "en k pasos se recorren k+1 cuadros". Combinando estos hechos tenemos una

explicación del por qué ciertos juegos son imposibles: "Si el número de cuadros a recorrer

es par, el cuadro inicial y el cuadro final deben ser de diferente color, mientras si es impar

los cuadros final e inicial deben ser del mismo color". Nótese que cumplirse la condición

no nos asegura que el problema tenga solución, pero si no se cumple la condición es

garantía para que el pasatiempo sea imposible de resolver.

Dibujar sin levantar el lápiz: Es un pasatiempo que conocemos desde la infancia, a veces

fácil, a veces difícil y otras imposible. En la figura 4 se muestran varios ejemplos.

a. b. c. d. e.

Figura 4. Grafos para dibujar sin levantar el lápiz y sin repetir línea.

Estos objetos que la matemática llama

gráficos o grafos se componen de vértices y

arcos. A cada arco le corresponden dos

vértices v1 y v2 que son los extremos del

arco. Si v1=v2 se forma un bucle (figura 5).

Figura 5. Bucle.

Figura 3.

1 B 2

A 3

Page 25: A Notas de Algebra Moderna

Los arcos los notaremos con números y los vértices con letras mayúsculas. Así el grafo de

la figura 3. a) tiene dos vértices A y B y tres arcos 1,2 y 3. En él todos los arcos tienen los

mismos vértices extremos A y B.

El número de arcos que caen sobre un vértice se llama orden del vértice. En el grafo de la

figura 3.a) el grado del vértice A es 3, igual al del vértice B.

La primera condición para que un grafo se pueda dibujar como queremos es que cualquier

par de vértices se puedan unir por medio de una sucesión de arcos; en términos

matemáticos que el grafo se conexo, en términos intuitivos, simplemente se exige que el

grafo debe estar "junto", no debe haber pedazos aislados. Es claro entonces que si no es

conexo debe haber por lo menos dos vértices que no se pueden conectar por ningún camino

y por lo tanto es imposible resolver el problema. Ser conexo es condición necesaria pero

no suficiente para que el grafo se pueda recorrer pasando por todos los arcos sin repetir

ninguno y sin levantar la mano. Por ejemplo el grafo de la figura 3.b) es conexo pero es

imposible recorrerlo de la manera exigida. Buscamos pues una condición más fuerte que se

convierta en suficiente.

Formalicemos más nuestro lenguaje. Notaremos XnY (donde X e Y representan vértices y n

es un número correspondiente a un arco) el hecho que el arco n tenga vértices X e Y. En

estos términos, que un vértice A se pueda conectar con el vértice B, significa que hay una

sucesión de k+1 vértices A=X0,X1,...,Xk (que pueden estar repetidos) y de k arcos n1,...,nk tal

que

An1 X1,X1n2X2, ...,Xk-1nkB

Ahora bien, si el grafo tiene k arcos y se puede dibujar de la forma descrita partiendo del

vértice I y terminando en el vértice F, entonces los k arcos se podrán colocar de tal forma

que para determinados vértices V1,V2,...,Vk-1, suceda:

In1 V1,V1n2V2, ...,Vk-1nkF (1)

Page 26: A Notas de Algebra Moderna

Nótese ahora que cada uno de los vértices Vi, aunque puede aparecer más de una vez,

perdón, mas de dos veces, siempre aparecer un número par de veces (salvo I y F): en

efecto, siempre que se de la situación Vi-1niXi, enseguida tenemos,

Vini+1Vi+1.

Por otra parte, en (1) aparece cada vértice tantas veces como arcos existan con algún

extremo en él, es decir, tantas veces como el orden del respectivo vértice. Se deduce que

cada vértice "interno" debe tener orden par y se tiene el siguiente resultado:

Proposición 1: Para que un grafo conexo pueda ser dibujado sin levantar el lápiz y sin

repetir línea es necesario que salvo los vértices inicial y final, los demás sean todos de

orden par. Esta condición que no sólo es necesaria sino también suficiente es uno de los

primeros resultados en lo que hoy se conoce como Teoría de Grafos y fue establecido por

Euler en 1735. La suficiencia de la condición no la podemos demostrar con los conceptos

introducidos en este libro. El lector interesado puede consultar cualquier libro elemental de

Teoría de Grafos.

El juego del solitario: Este último juego que analizaremos es de remoto origen aunque la

primera referencia que se conoce de él la debemos a Leibniz. Consideremos un tablero en

forma arbitraria repartido en cuadros arreglados en filas y columnas.

Figura 6.

Page 27: A Notas de Algebra Moderna

En determinados cuadros del tablero aparecen piezas de juego, a lo m s una por cuadro. Un

movimiento, o salto, es posible cuando sobre tres cuadros A, B y C adyacentes sobre una

fila o una columna, hay piezas sobre A y sobre B pero no sobre C. El salto consiste en

mover la pieza que está sobre A hasta C retirando del juego la ficha que estaba sobre B

(Figura 6).

El objeto del juego es llevar las fichas hasta cierta y determinada configuración;

generalmente se debe dejar una única ficha en el tablero (es claro que en la posición inicial

debe haber por lo menos un lugar vacío).

Lo que queremos al aplicar los conceptos de teoría de números es averiguar si algún juego

propuesto es de imposible solución o si el resultado est determinado de alguna manera.

Deseamos escribir una ecuación que describa el proceso del juego y que tendrá como

variables el número de piezas en juego y el número de saltos; es pues natural utilizar

números enteros porque no concebimos "medio salto" o "siete cuartos de fichas".

Para facilitar nuestra tarea coloreamos los cuadros de cada diagonal con tres colores

diferentes. La primera diagonal se colorea digamos de verde, la siguiente de azul y la que

viene de rojo, para seguir con la siguiente que coloreamos de verde y continuar de manera

cíclica: azul, rojo, verde; azul, rojo, verde; etc. La figura 7 muestra un rectángulo 7×5

donde cada cuadro se ha marcado con las letras V, A o R , según se haya coloreado de verde

azul o rojo en el procesos que acabamos de describir.

V A R V A

A R V A R

R V A R V

V A R V A

A R V A R

R V A R V

V A R V A

Figura 7.

Page 28: A Notas de Algebra Moderna

Veamos en este ejemplo una sucesión de saltos permitidos como los que se muestran en la

figura 8. En la tabla 1 se indica en cada posición cuántas fichas están colocadas en cada

color. Vemos que en cada salto crece un color y decrecen los otros dos: crece el color a

donde la ficha desplazada llega y decrecen el valor del color donde estaba y del color de la

que fue sacada del juego. Realmente esta observación es válida en general, podemos

afirmar que: "Si por efecto de un salto se llega a determinado color, el número de fichas

correspondiente a ese color crece una unidad y los otros colores decrecen cada uno,

también en una unidad" .

Verdes Azules Rojos Total

Inicio 11 12 11 34

1 12 11 10 33

2 11 10 11 32

3 12 9 10 31

4 11 10 9 30

Tabla 1. Se muestra los cambios de valores en la sucesión de la

figura 8 según la coloración de la figura 7.

Por esta razón llamando V, A y R el número de fichas que están inicialmente en cada color

y V',A', y R' el número de fichas en cada color finalmente y v, a y r el número de saltos que

terminan en cada color, se debe cumplir:

• • • • • • • • • • • • • • • • • • • • • • • • • • • • • • • • • • •

Inicio

• • • • • • • • • • • • • • • • • • • • • • • • • • • • • • • • • •

Posición 1.

• • • • • • • • • • • • • • • • • • • • • • • • • • • • • • • • •

Posición 2.

• • • • • • • • • • • • • • • • • • • • • • • • • • • • • • • •

Posición 3.

• • • • • • • • • • • • • • • • • • • • • • • • • • • • • •

Posición 4.

Figura 8. Sucesión de saltos.

Page 29: A Notas de Algebra Moderna

V+v-a-r = V'

A+a-v-r = A'

R+r-v-a = R'

Ecuaciones que, por cualquiera de los métodos usuales se puede comprobar, son

equivalentes al sistema de ecuaciones:

2v = (A+R)-(A'+R')

2a = (V+R)-(V'+R')

2r = (A+V)-(A'+R')

Lo que nos indica que:

A+R y A'+R' son de igual paridad

V+R y V'+R' son de igual paridad

A+V y A'+V' son de igual paridad

Estas condiciones necesarias para que un juego sea soluble pueden resultar algunas veces

suficientes para demostrar que ciertos juegos son imposibles.

Otro sistema de condiciones necesarias parecido a (2) se obtiene al colorear las otras

diagonales. Al combinar los dos sistemas de condiciones se alcanza a determinar un buen

número de juegos imposibles como se puede ver en el siguiente ejemplo:

Ejemplo: Supongamos que el juego propuesto se lleva a cabo en un tablero como el de la

figura 7, en donde la posición inicial contiene todos los cuadros con fichas con excepción

del extremo superior izquierdo. Los valores de V, A y R son V=11, A=12 y R=11. Si se

quiere llegar al final del juego con una única ficha, los valores posibles de V', A' y R' serían

(1,0,0), (0,1,0) o (0,0,1). Pero la primera y las última de estas posibilidades no cumplen las

condiciones (2). Por lo tanto la última ficha debe quedar en uno de los cuadros coloreados

de azul.

Page 30: A Notas de Algebra Moderna

Ahora podemos colorear las otras diagonales, con los colores blanco, marrón y negro, (B,

M y N) como se muestra en la figura 9. Haciendo el análisis correspondiente para este

caso obtenemos que la última ficha debe quedar colocada en un cuadro blanco. Los únicos

cuadros que son azules en la primera coloración y blancos en ésta, están marcados con una

× en la figura 10. Estos son naturalmente los únicos lugares en donde puede en donde

puede quedar la última ficha.

EJERCICIOS

1. Determinar si en los siguientes tableros es posible recorrer todos los cuadros partiendo

de I y llegando a F.

N B M N B

M N B M N

B M N B M

N B M N B

M N B M N

B M N B M

N B M N B

Figura 9.

× ×

× ×

× ×

Figura 10.

I F

I

F

I

F

Page 31: A Notas de Algebra Moderna

2. Para cada uno de los grafos conexos siguientes determinar si se puede o no dibujar sin

levantar la mano y sin repetir linea, en caso afirmativo determinar una ruta.

3. En la demostración de la proposición 1 (que se hace antes de enunciarla) ¿qué resultado

de paridad se utiliza? ¿Dónde?

4. En la ciudad de K niesberg desembocan dos ríos formando dos islas como se muestra en

la figura adjunta, configuración en la que además hay siete puentes. Hace algunos siglos

una diversión dominical consistía en tratar de recorrer los siete puentes sin repetir

ninguno. Corría el rumor de que tal propósito era imposible.

Interpretando el problema como un grafo que se debe dibujar sin levantar la mano y sin

repetir línea (aquí los arcos son los puentes) y utilizando la teoría de esta sección,

demostrar que el problema realmente es imposible. Así lo demostró Euler en 1735.

5. a. En un grafo sin bucles la suma de los órdenes de todos los vértices debe se par. Por

qué?

b. Dividiendo los vértices entre los que tienen orden par y los que tienen orden impar y

aprovechando el resultado anterior, demuestre que en un grafo sin bucles, siempre hay

un número par de vértices de orden impar.

Page 32: A Notas de Algebra Moderna

6. Explique por qué, el conjunto de condiciones (2) conjuntamente con sus equivalentes

para la coloración en sentido contrario, son condiciones necesarias pero no suficientes

para que un juego propuesto de solitario tenga solución.

7. Analizar por el método expuesto si los siguientes juegos son posibles y en caso

afirmativo mostrar una solución salto a salto.

8. Considere un juego planteado como en la posición inicial de la figura 8.

a. Si se quiere terminar con una sola ficha en el tablero, dónde quedar colocada ésta?

b. Además de la posición inicial estudiada, cuáles otras posiciones iniciales conllevan

a la posición final mostrada en la figura adjunta ?

c. Muestre que el juego propuesto con la posición inicial de la figura 7 y con posición

final como se muestra en la figura, es efectivamente realizable. muestra en la figura,

es efectivamente realizable.

• • • • • • • • • • • • • • • • • • • • • • • •

• • • • •

• •

• •

?

?

?

•• • •

Page 33: A Notas de Algebra Moderna

6. TRES PROPIEDADES DE LOS ENTEROS POSITIVOS

En ésta sección expondremos tres principios fundamentales que serán el fundamento de la

teoría expuesta en las secciones siguientes. Estos tres principios algoritmo de la división, el

teorema fundamental de la aritmética y el principio de inducción se darán aquí sin su

demostración, que dejaremos pendiente para cuando tratemos la axiomática de los enteros.

Buscamos mas que todo que el lector se familiarice con la exposición formal de éstos para

que pueda aplicarlos cuando sea necesario. En la sección 4 asegurábamos que todo número

entero es o bien par (de la forma 2k) o bien impar (de la forma 2k+1), este resultado es en

realidad un caso particular de la siguiente proposición que formaliza algo que manejamos

desde niños.

Proposición 1. (Algoritmo de la División) Sea n cualquier entero y m un entero positivo,

entonces existen q y r enteros únicos tales que

i) n=mq+r

ii) 0<r<m

Lo que nos dice la proposición es que cualquier entero (n) se puede dividir por otro mayor

que 0 (el divisor es m) obteniendo de manera unívoca un cociente (q) y un residuo (r) que

debe ser positivo menor que el divisor o ser 0. Por ejemplo, si n=19, m=5, los valores de q

y r serán 3 y 4 respectivamente.

Proposición 2. Todo número elevado al cuadrado o bien es múltiplo de 4, o bien es de la

forma 4k+1.

Demostración. Según el algoritmo de la división los números enteros al dividirlos por 4

pueden tener residuos 0, 1, 2 ó 3. Por ésta razón todo número entero es de la forma 4n,

4n+1, 4n+2 o bien 4n+3.

Page 34: A Notas de Algebra Moderna

Sabemos que los números de la forma 4n son cerrados para la multiplicación, igual sucede

con los de la forma 4n+1, lo cual implica que sus cuadrados son de la misma forma lo que

está de acuerdo con la proposición. Por otra parte si un número tiene la forma 4n+2 su

cuadrado 16n2+16n+4 es múltiplo de 4, mientras que si tiene la forma 4n+3 su cuadrado

16n2+ 24n+ 9 es de la forma 4n+1 (tomando n como...).

Vemos entonces que en todos los casos se obtiene o bien un múltiplo de 4 ó un número de

la forma 4n+1, completando la demostración de la proposicióng.

Para enunciar el siguiente principio supongamos que p1,p2,...,pn, ... es la sucesión ordenada

de números primos. Según esto p1=2, p2=3, p3=5, p4=7, etc. El Teorema Fundamental de

la Aritmética nos garantiza que todo número mayor que 1 es factorizable de manera única,

como un producto de potencias de primos.

Proposición 3. (Teorema Fundamental de la Aritmética /1/) Para todo entero positivo a

existen α1,α2,α3, .....,αn naturales únicos tales que:

a= nnppp ααα ...21

21

siendo p1,p2,...,pn la sucesión ordenada de números primos.

Las célebres descomposiciones en factores primos que se aprenden en la aritmética

elemental son ejemplos de aplicación de éste importante teorema. Por ejemplo, si a=24

decimos que a=23×31, entonces n=2, α1=3 y a=21×72 entonces n=4, α1=1,α2=α3=0 y

α4=2.

La siguiente proposición nos enuncia el Principio de Inducción; sin que podamos decir de

él que se aprende en la escuela primaria, sí podemos asegurar que es un principio muy útil

para hacer demostraciones y que su razón lógica se encuentra en la misma razón de ser de

los números naturales. En efecto, una propiedad básica e intuitivamente clara de los

números naturales es que a cada uno le sigue otro, y a éste, otro, y así sucesivamente,

entonces se agotan todos potencialmente. Este "así sucesivamente" es lo que se formaliza

en el citado principio. Veamos primero un ejemplo:

Page 35: A Notas de Algebra Moderna

Ejemplo 1. En la sección 1 hemos visto como los griegos visualizaban el hecho de que los

primeros n números impares suman exactamente n2. También podemos razonar así:

El primer impar r es 1. Su suma que es 12. El segundo impar es 1×2+1 y la suma del

primero y el segundo es 12+1×2+1 que es (1+1)2 o sea 22. Sabiendo que la suma de los

primeros impares es 22 podemos sumarle a ésta el tercer impar que es 2×2+1 y obtenemos:

22+2×2+1=(2+1)2=9; así obtenemos que la suma de los tres primeros impares es 32.

Supongamos que siguiendo este proceso hemos llegado hasta 36 y sabemos que la suma de

los primeros 36 impares es 362. Como el siguiente impar es 2×36+1, la suma de los

primeros 37 números impares ser: 362+2×36+1=(36+1)2 o sea que se cumple la hipótesis

para 37.

Podríamos decir "...y así sucesivamente..." pero es mejor formalizar aún más. En términos

generales lo que sucede es la siguiente: Si aceptamos que para un n particular la

proposición es cierta, es decir, que la suma de los primeros n impares es n2, sumando a

éstos el (n+1) impar, que es 2n+1 obtenemos que la suma de los primeros n+1 impares ser

n2+2n+1=(n+1)2, o sea que la proposición es cierta para n+1. Tenemos entonces, que si

consideramos cierta la proposición para un número particular automáticamente ser cierta

para su siguiente. Así podremos recorrer todos los naturales desde que aseguremos que la

proposición se cumple para los primeros.

Se acepta entonces que la proposición es cierta para cualquier k, por que potencialmente se

sabe que hay un camino, una cadena que se puede recorrer paso a paso. Esto nos asegura el

Principio de Induccióng.

Proposición 4. (Principio de Inducción) Sea P(n) una proposición para cada natural n tal

que:

i) P(0) es cierta

ii) Siempre que P(k) es cierta se sigue que P(k+1) también lo es.

Page 36: A Notas de Algebra Moderna

Entonces la proposición es cierta para cualquier n natural.

Según esto, para demostrar la proposición P(n) por inducción sobre n, debemos probar

primero que P(0) es cierta (casi siempre es muy fácil) y luego deducir que P(k+1) es cierta

suponiendo que P(k) lo es. La verificación de estas dos condiciones asegura, por el

Principio de Inducción, que P(n) es cierta para todo natural n. Veamos otro ejemplo:

Ejemplo 2. Demostrar por inducción sobre n que 10n-1 siempre es divisible por 9.

Demostración.

i) Es claro que la hipótesis se cumpla cuando n=0. También es fácil comprobar que se

cumple para n=1 y n=2 (aunque no es necesario).

ii) Supongamos que P(k) es cierto, o sea, que 10k-1 es múltiplo de 9 es decir que existe un

entero s tal que 10k-1=9 Debemos deducir que se cumple P(k+1) o sea que 10k+1-1 es

también múltiplo de 9, es decir, debemos encontrar un t entero tal que 9t=10 k+1-1. Pero

10 k+1-1=(10 k-1)10+9=9s×10+9

de donde haciendo t=10s+1 , tenemos:

10 k+1-1=9t

y hemos demostrado que P(k+1) también es cierto. Así por el Principio de Inducción

hemos probado que la proposición propuesta es cierta par todo natural n.

El Principio de Inducción no sirve únicamente para hacer demostraciones. Es muy útil

también para hacer definiciones /1/ que son llamadas Definiciones Recursivas.

Primero aclaremos cuáles son los objetos que se van a definir. Una función f que a cada

natural n le asocia cualquier elemento f(n) de un conjunto B se llama una Sucesión en B.

Por ejemplo la sucesión n1 al natural 1 se le asocia 1, a 2 le asocia 2

1 y a 4 le asocia 41 ,

etc.

Page 37: A Notas de Algebra Moderna

A veces, en lugar de escribir una fórmula se escriben los primeros términos con la

esperanza de encontrar una fórmula observando la relación que hay entre los términos

mostrados. Así en el ejemplo anterior se mostrarían 1, 21 , 3

1 , 41 , ..... y se pueden

suponer que sigue 51 , 6

1 , 71 , .... pero nada nos lo asegura.

Una forma de definir ser entonces mostrar unos primeros términos (con uno basta) y decir

cómo se forman los siguientes, o sea, dar una regla para conocer el n-énesimo término

cuando se conocen los primeros n-1. Esto se hace de manera implícita muchas veces. Por

ejemplo para definir an se dice que a0=1, a1=a, a2=a⋅a, a2=a⋅a⋅a y así sucesivamente se

entiende que cada término se obtiene multiplicando el anterior por a. Es decir, si damos

por conocido el valor de an-1, el término an se formar según la fórmula:

an=an-1⋅a

y esta es la ley recursiva que nos permite hablar de an para cada n que sea entero.

Colorario. (Definiciones Recursivas) Si se da un valor a f(1) y se da una regla para

calcular f(n) en base al valor de f(n-1), entonces queda definida y determinada la función

f(n) para todo entero positivo n.

Ejemplo 2. Generalmente cuando hay puntos sucesivos lo que se quiere indicar se puede

formalizar con definiciones recursivas.

Por ejemplo, si queremos que si sea la suma de las potencias desde 0 hasta i de un número

fijo x, no necesariamente natural, es decir queremos que:

Si=1+x+x2 +..+ xi

podemos definir Si=1+x y Sn= Sn-1+xn. Esta última igualdad nos indica que para formar el

término n-ésimo tomamos el término anterior y le sumamos la potencia correspondiente.

Nótese que aquí hay doble aplicación de las definiciones recursivas: Para encontrar la

potencia siguiente y para agregar a la suma que lleva. Hacemos notar también que se

pueden dar otras definiciones para Si. En efecto, si definimos recursivamente Si=1+x y

S'n-1+1 se puede comprobar que para todo k, Sk=S'k (ejercicio 15).

Page 38: A Notas de Algebra Moderna

Las definiciones recursivas son muy útiles puesto que sirven para formar algoritmos que

calculen la función y además son la base para hacer las demostraciones de sus propiedades.

Por ejemplo el ejercicio 13 e) pide que e demuestre por inducción una fórmula directa para

hallar el valor de Sk.

PREGUNTAS Y EJERCICIOS

1. Por qué del algoritmo de la división se deduce que todo número es par o impar?

2. En cada caso encuentre q y r que hagan cumplir el algoritmo de la división para n y m

dados:

a) n=25 m=8 b) m=25 n=8

c) n=25 m=5 d) m=1 N=25

3. ¿Por qué el algoritmo de la división no es cierto para m=0?

4. ¿Qué problema hay en definir el algoritmo de la división para m=0 sin exigir que m sea

positivo?

5. Demuestre que no hay números cuadrados de la forma 6n+2 ó 6n+5.

6. Demuestre que ningún número de la forma 4n+3 es la suma de dos cuadrados.

7. Sean b y m enteros positivos. Si q es el cociente y r es el residuo cuando el entero a se

divide por b, demostrar que cuando ma se divide entre mb el cociente es q y el residuo

es mr.

8. Si p y q son primos tales que p|q demostrar que p=q.

Page 39: A Notas de Algebra Moderna

9. Sean a, b, c enteros con b>0 y c>0. Si q es el cociente cuando a se divide por b y q' es

el cociente cuando q se divide entre c, demostrar que q' es el cociente cuando a se

divide por bc.

10. Si n es entero positivo y p1, p2,...,pn son primos positivos distintos, demostrar que el

entero (p1p2...pn)+1 no es divisible por ninguno de esos primos.

11. Demuestre que si (a, b, c) es una tripla pitagórica y a, b, c no son todos pares, entonces

dos de ellos, incluyendo c, son impares y el otro es par.

12. Dar una demostración para el recíproco del teorema de Wilson.

13. Demostrar por inducción sobre n:

a) La suma de los primeros n números es ( )121+nn

.

b) La suma de los primeros n números pares es n(n+1).

c) an-1 es divisible por a-1.

d) an-bn es divisible por a-b.

e) 1+r+r2+...+rn=r

r n

−− +

11 1

(r no es 1)

14. Definir n! recursivamente.

15. Se define recursivamente Sk y S'k así:

S0 = 1

Sk = Sk-1+xk

S'k = xS'k-1+1

Demostrar que para todo k entero positivo se tiene: Sk=S'k

16. En base a la definición recursiva de an demostrar por inducción sobre n:

i) anam=an+m

Page 40: A Notas de Algebra Moderna

ii) (an)m=anm

17. Sea nnppp ααα ...21

21 y a= nnppp βββ ...21

21 las descomposiciones en factores primos de a y b

según el Teorema Fundamental de la Aritmética.

a) Demuestre que ab tiene representación: nn

nppp βαβαβα +++ ...221121

b) Demuestre que para que a divida a b es condición necesaria y suficiente que para

todo i de 1 a n se cumpla que αi≤βi.

Page 41: A Notas de Algebra Moderna

7. SUMATORIA Y MULTIPLICATORIA

Las operaciones principales entre números, la suma y el producto, son asociativas y

conmutativas, lo cual significa que cuando hay sucesivas sumas o productos los paréntesis

se pueden eliminar. En otras palabras, es posible sumar todos los elementos de un conjunto

finito de números y también se pueden multiplicar. Así, si A es un conjunto finito de

números

∑∈Ax

x

se lee como "Sumatoria de los x de A" y simboliza la suma de todos los números del

conjunto A y

∏∈Ax

x

se lee como "Productoria de los x de A" o "multiplicatoria" ó simplemente "Producto de los

x de A", simbolizando el producto de todos los números del conjunto A.

Generalmente los elementos de A se enumeran y se notan a1,a2,. . . ,an, entonces la suma y

el producto se notan:

∑=

n

iia

1

∏=

n

iia

1

La manipulación de ésta notación es importante porque nos lleva no sólo a comprobar los

valores de ciertas sumas y productos sino también a descubrirlos. Para ésto basta con

manejar unas propiedades de ésta notación. Nos referiremos al símbolo ä aunque estas

propiedades tienen su correspondencia para ∏ .

Propiedades. Sean L<M y P enteros; {ai} una sucesión de números (puede haber

subíndices negativos), k un número cualesquiera, entonces se cumple:

Page 42: A Notas de Algebra Moderna

1. k∑=

M

Liia =∑

=

M

Liika .

2. ( )∑=

+M

Liii ba =∑

=

M

Liia +∑

=

M

Liib .

3. ∑=

M

Lik =(M-L+1)k (Suma de una constante).

4. ∑=

M

Liia = ∑

+

+=−

PM

PLiPia (Cambio de límites).

5. ( )∑=

−−M

Piii aa 1 =aM-aP-1 (Propiedad telescópica).

6. ∑=

M

Liia =∑

+

=

NL

Liia + ∑

++=

M

NLiia

1.

Demostración. Todas las demostraciones se pueden hacer por inducción (si M=L+q la

inducción se hace sobre q; ejercicio 3) sin embargo se dar n argumentos intuitivos:

1. k∑=

M

Liia =k(aL+aL+1+aL+2+...+aM)= kaL+kaL+1+kaL+2+...+kaM=∑

=

M

Liika .

2. ( )∑=

+M

Liii ba =∑

=

M

Liia +∑

=

M

Liib .

La suma de la izquierda es:

(aL+bL)+(aL+1+bL+1)+...+(aM+bM)

y reagrupando se tiene:

(aL+aL+1+...+aM)+(bL+bL+1+...+bM)

que es la suma de la derecha.

3. En ∑ k hay (M-L+1) sumandos, todos iguales a k, su suma es por lo tanto k(M-L+1)

4. Nótese que éstas son simplemente dos formas de escribir lo mismo; ambas sumatorias

tienen M-L+1 sumandos y ambas empiezan en aL terminando en aM.

5. ( )∑=

−−M

Piii aa 1 = (aP-aP-1)+ (aP+1-aP)+ ...+(aM-1-aM-2)+ (aM-aM-1)

= -aP-1+ (aP-aP)+ ...+( aM-1-aM-1)+ aM

= aM-aP-1

Page 43: A Notas de Algebra Moderna

6. ∑=

M

Liia =(aL+aL+1+...+aL+N)+(aL+N+1+aL+N+2+...+aM)=∑

+

=

NL

Liia + ∑

++=

M

NLiia

1

Como decíamos en un principio, con éstas propiedades podemos hallar los valores de

ciertas sumas, por ejemplo de progresiones aritméticas.

Ejemplo 1. Sabiendo que ∑=

n

ii

1= ( )

21+nn encontrar el valor de 1+4+7+...+(3n-2).

Solución. Se nos pide hallar el valor de ( )∑=

−n

ii

123 tenemos:

( )∑=

−n

ii

123 = ∑

=

n

ii

13 -∑

=

n

i 12 (Por propiedad de linealidad)

= 3∑=

n

ii

1

-∑=

n

i 12

= 3 ( )2

1+nn -2(n-1+1) (Aplicamos 3) y el valor de∑=

n

ii

1

= ( )

213 −nng

Nótese que el resultado puede comprobarse por inducción pero éste método, insistimos,

tiene la ventaja de encontrar el valor buscado; la inducción se efectúa cuando ya se conoce

el resultado o cuando se presiente.

En el ejemplo anterior supusimos conocido el valor de ∑=

n

ii

1; en realidad su valor se puede

encontrar aplicando las propiedades enunciadas para ∑ , como veremos a continuación.

La demostración es importante porque ilustra el método para encontrar ∑=

n

ii

1

2 , ∑=

n

ii

1

3 , etc.

Proposición 1. ∑=

n

ii

1= ( )

21+nn

Demostración. Partimos de la igualdad:

Page 44: A Notas de Algebra Moderna

∑−

=

1

1

2n

ii = ( )∑

=

−n

ii

2

21 (Propiedad 4).

= ∑=

n

ii

2

2 -2∑=

n

ii

2+∑

=

n

i 21 (Por linealidad).

= ⎟⎠

⎞⎜⎝

⎛∑=

n

ii

1

2 -1-2∑=

n

ii

2+(n-1)

(Se suma y se resta 1, se aplica

propiedad 3).

= ∑−

=

1

1

2n

ii +(n2-1)-2 ⎥

⎤⎢⎣

⎡−⎟

⎞⎜⎝

⎛∑=

11

n

ii +(n-1)

(Se “saca” n2 de ∑ 2i y en ∑ i se

introduce 1, que enseguida se resta).

Tomando los dos extremos de estas igualdades vemos que en ambos miembros de éstas

aparece

∑−

=

1

1

2n

ii

término que entonces podemos eliminar para despejar la suma buscada y obtener

2∑=

n

i

i2

=n(n+1)

Lo que nos conduce inmediatamente a la igualdad buscada.g

El lector no debe perderse en los cálculos de la anterior demostración, aunque sí los debe

comprobar, lo importante es que vea cúal es el truco fundamental que se utiliza: Se trata de

desarrollar ∑ 2i como ( )∑ − 21i para luego eliminar ∑ 2i .

Este truco como decíamos, se emplea para encontrar ∑ 2i una vez se conoce ∑ i : Se

desarrolla ∑ 3i como ( )∑ − 31i para luego eliminar ∑ 3i (Ejercicio 6).

Otra suma que nos interesa, es la suma de progresión aritmética, es decir la suma de las

potencias de un número (ejemplo 3 y ejercicio 13 e) de la sección 6). De ella da razón la

siguiente proposición:

Proposición 2. ∑=

n

i

ix0

=1

11

−−+

xx n

Page 45: A Notas de Algebra Moderna

Demostración. Como x es una constante:

x∑=

n

i

ix0

= ∑=

+n

i

ix0

1 (Propiedad 1)

= ∑+

=

1

1

n

i

ix (Propiedad 4)

= ⎟⎠

⎞⎜⎝

⎛∑+

=

1

0

n

i

ix -1 (Suma y resta)

= ∑=

n

i

ix1

+(xn+1-1) (Separando el último termino de la suma)

Se tiene entonces que

x∑=

n

i

ix0

=∑=

n

i

ix1

+(xn+1-1)

y despejando ∑ ix obtenemos el resultado de la proposición.

EJERCICIOS

1. Calcular los valores numéricos de las siguientes expresiones:

a) ∑=

6

1

2

ii b) ( ) 1

4

1

2 1 +

=

−∑ i

ii c) ( )∑

=

+10

312

ii

d) ( )∏=

+6

1

1i

i e) ∑∏= =

3

1 1j

j

i

i f) ∏∑= =

3

1 1j

j

ii

2. Exprese en términos de sumatoria n!

3. Demostrar por inducción las propiedades de ∑ Ud. debe partir de la definición

recursiva de ∑ . ¿Cuál propiedad encierra esta definición?

4. Formular propiedades de ∏ análogas a las enunciadas para∑ .

5. Dado por conocido el valor de ∑ i y utilizando las propiedades de ∑ encontrar:

Page 46: A Notas de Algebra Moderna

a) La suma de los primeros n impares.

b) 2+8+14+...+(6n-4).

c) La suma de los n primeros números de la sucesión 3,8,13,...

6. Deducir una fórmula para :

a) ∑=

n

ii

1

2 b) ∑=

n

ii

1

3 c) ∏=

n

i

i

1

2

7. Deducir una fórmula para hallar el valor de:

a) ∑=

n

i

ix1

b) ( )∑= +

n

i kk1 11

8. Deducir una fórmula para :

a) ∏=

⎟⎟⎠

⎞⎜⎜⎝

⎛−

n

j j2

11 b) ∏=

⎟⎟⎠

⎞⎜⎜⎝

⎛−

n

j j12

11

9. Un cuadro mágico de orden n es un cuadrado dividido en cuadrados en donde se

colocan los números de 1 hasta n2 de tal manera que cada fila o diagonal suman lo

mismo. A continuación un cuadro mágico de orden 3:

8 1 6

3 5 7

4 9 2

En general cuánto vale la suma de los números de cada columna fila o diagonal de un

cuadro mágico de orden n?

10. En la demostraciones la proposiciones 1 y 2 se utiliza varias veces la propiedad 6.

¿Dónde?

Page 47: A Notas de Algebra Moderna

8. EL MAXIMO COMUN DIVISOR.

La relación "n divide a m" tiene sentido cuando n y m son enteros o naturales, pero no para

fraccionarios o reales (por qué?) En la sección 4 vimos la forma de demostrar las

propiedades mas elementales sobre esta relación, propiedades que resumimos a

continuación utilizando la notación "n|m", también introducida en esa sección.

Propiedades de la relación "n divide a m". Siendo a, b, c enteros no nulos se tiene:

1) a|0 y ±1|a

2) a|a

3) Si a|b y b|c entonces a|c.

4) Si a|b y b|a entonces a=±b

5) Si a|b y a|c entonces para cualesquier enteros x, y se tiene que a|(xb+yc)

6) Si a|b entonces |a|≤|b|.

En base a estas propiedades desarrollaremos el concepto de máximo común divisor de dos

enteros a y b (no nulos). En aritmética elemental se conocen algoritmos para encontrar el

máximo común divisor de dos enteros y se entiende que por ejemplo el máximo común

divisor de 9 y 12 es 3, ya que de los divisores positivos comunes de 9 y 12 el mayor es 3.

Nosotros nos basaremos en la siguiente definición:

Definición 1. Dados dos enteros a, b ninguno nulo, Máximo Común Divisor de a y b que

notaremos (a,b), ser el entero positivo c tal que:

i) c|a y c|b.

ii) Si x|a y x|b entonces x|c.

La condición i nos indica que c debe ser un común divisor y a condición ii no señala que es

el máximo. En los ejercicios 4 y 5 se da una necesaria discusión sobre esta definición. La

siguiente proposición nos permite hablar del m.c.d. de tres o mas números.

Page 48: A Notas de Algebra Moderna

Proposición. ((a,b),c)=(a,(b,c))

Demostración. Sean d=(a,b), e=(b,c), f=(a,e) y g=(d,c) debemos demostrar que g=f.

Por ser g=(d,c) entonces g|d y g|c. Por ser d=(a,b) y g|d tenemos que d|a y d|b o sea se tiene

que g divide a a,b y c. Pero si g divide a b y a c entonces g debe dividir a e=(b,c) y como

también divide a a entonces g|f. De manera similar se ve que f|g lo que implica que g=ñf ,

pero como ambos son positivos concluimos que g=f.

Para hallar (n,m) un método muy antiguo, llamado el algoritmo de Euclides, consiste en

hacer divisiones sucesivas, como mostraremos en el siguiente ejemplo para enseguida

formalizar:

Ejemplo 1. Para hallar (32,18) dividimos 32 entre 18 y obtenemos como residuo 14, luego

dividimos 18 entre 14 obteniendo como residuo 4, enseguida dividimos 14 entre 4 y

obtenemos residuo 2, y al dividir 4 entre 2 obtenemos residuo 0. Como 2 es el último

residuo no nulo, 2 es el máximo común divisor de 32 y 18.

Dividendo Divisor Residuo

32 18 14

18 14 4

l4 4 2 (32,18)

Tabla 1. Divisiones sucesivas para encontrar

(32,l8) según el algoritmo euclideano.

Proposicion 2. (Algoritmo Euclideano)

Si a y b son enteros positivos por el algoritmo de la división (Propiedad 6-1 Capítulo 1)

podemos encontrar r1...rk y q1...qk+1 tales que:

Page 49: A Notas de Algebra Moderna

(1) a = bq1+r1 0 < r1< b=r0

b = r1q2+r2 0 < r2<r1

r1 = r2q3+r3 0 < r3<r2

rk-3 = rk-2qk-1+rk-1 0 < rk-1<rk-2

rk-2 = rk-1qk+rk 0 < rk<rk-1

rk-1 = rkqk+1

de esta forma, el último residuo no nulo rk, es el máximo común divisor de a y b.

Demostración. Vamos a proceder por inducción sobre k, que es el número de pasos que

hay en el proceso. Notemos que el proceso se detiene cuando rk+1=0 pues no se puede

hacer la siguiente división.

i) Si k=0 o sea el primer residuo r1 es 0, entonces a es múltiplo de b y por tanto (ejercicio

2) el máximo común divisor es b.

ii) Supongamos que se tiene demostrado cuando hay sólo k-1 residuos, entonces

empecemos el proceso en la segunda ecuación de (1) o sea en b=r1q1+r2. Partiendo de esta

ecuación hasta llegar a la última tenemos k-1 residuos no nulos, entonces por hipótesis de

inducción podemos decir que rk=(b1,r1).

Tenemos:

i) rk|b y rk|r1

ii) x|b y x|r1 ⇒ x|rk

Como r0>r1>...>0 entonces algún rk+1 debe ser cero, esto nos garantiza que el proceso

descrito en (1) es finito.

Pero a=bq1+ r1 entonces rk|a y tenemos que

Page 50: A Notas de Algebra Moderna

i)' rk|a y rk|b.

Ahora bien, si x|a y x|b entonces x|a-bq1 o sea x|r1 y por ii) tenemos que x|rk , por tanto:

ii)' x|a y x|b entonces x|rk.

i)' e ii)' nos garantizan que (a,b)=rk con lo cual queda demostrada la proposición.

Corolario. Si a y b son enteros, los números de la forma αa+βb α,β∈Z se llaman

combinación lineal de a y b. La menor combinación lineal positiva de dos enteros no nulos

es el máximo común divisor.

Ejemplo 2. Para expresar (32,18) como combinación lineal de 32 y 18 podemos recurrir

al algoritmo euclidiano pero en sentido inverso. Según este (tabla 1) tendríamos:

32 = 18×1+l4

l8 = 14×1+ 4

l4 = 4×3+ 4

4 = 2×2

Entonces de la penúltima ecuación tenemos:

2=14-4×3

Pero 4=l8-14×1 entonces 2=l4-(18-14×1)×3=14×4-18×3 y como l4=32-18 entonces

2=(32-18)×4-18×3=32×4+(18)×(-7) y hemos encontrado α=4 y β=-7 tal que 2=(32,18)=

32α+18β. Este proceso es el que utilizamos para la demostración general.

Demostración. Nótese primero que si x es combinación lineal de n y m, y a la vez m es

combinación lineal de n y m', entonces x es combinación lineal de n y m' (ejercicio 7). Por

esta razón y según las ecuaciones de (1) vemos que rk es combinación lineal de rk-1 y rk-2 y a

la vez rk-1 es combinación lineal de rk-2 y rk-3 entonces rk es combinación lineal de rk-2 y rk-3.

Por este proceso "vamos subiendo" hasta llegar a que rk es combinación lineal de r1 y b,

pero como r1 es combinación lineal de a y b vemos que rk, el máximo común divisor, es

combinación lineal de a y b.

Page 51: A Notas de Algebra Moderna

Por otra parte, el máximo común divisor divide a a y divide a b y por tanto a cualquier

combinación lineal de a y b y se deduce que es la menor de todas las combinaciones

lineales positivas de a y b.

Definición. a y b se llaman primos relativos si y sólo si (a,b)=1.

Proposición 3. (Lema de Euclides) Supongamos que a y b son primos relativos y que a|bc

entonces a|c.

Demostración. Como (a,b)=1, según el corolario anterior existen α,β tales que 1=αa+βb

multiplicado por c a ambos lados obtenemos que c=αac+βbc como a|bc y a|αac entonces

a|c.

El siguiente resultado, cuya demostración se deja como ejercicio al lector, establece un

método muy usado para construir el máximo común divisor de dos números: Se

descomponen en factores primos y se escogen aquellos factores comunes con su menor

exponente.

Proposición 4. Si la descomposiciones en factores primos de a y b son:

a= nnppp ααα …21

21

y

b= nnppp βββ …21

21

entonces el máximo común divisor de a y b, (a,b) tiene como descomposición en factores

primos n

nppp γγγ …2121

donde γi es el mínimo entre αi y βi.

Page 52: A Notas de Algebra Moderna

EJERCICIOS

1. Encontrar el máximo común divisor de los siguientes pares de enteros. Expresarlo

como combinación lineal de los dos números:

i) 52, 38 ii) 81, 110 iii) 320, 112 iv) 7469,238

2. Demuestre que (a, ka)=a (con a>0) y que (1, a)=1

3. Sea d=(a,b), demuestre que dba. es un entero múltiplo de a y b.

4. Demuestre que la definición 1 es una buena definición. Es decir, que si dos números c

y c' cumplen la definición se debe tener c=c'.

5. El máximo común divisor de a y b se puede definir como aquel entero c tal que:

i) c|a y c|b.

ii) (x|a y x|b) implica x<c.

Demostrar que esta definición es equivalente a la definición 1 (para esto, suponga que c'

cumple la definición 1 y que c cumple la anterior definición y deduzca que c'=c).

6. Demostrar que (a,b)=(a,b+ka) para todo k.

7. Si x es combinación lineal de n y m, y a la vez m es combinación lineal de n y m'

entonces x es combinación lineal de n y m'.

8. Demostrar que a y b son primos relativos si 1 se puede expresar como combinación

lineal de a y b.

9. Si m es un entero positivo, demostrar que (ma,mb)=m(a,b).

Page 53: A Notas de Algebra Moderna

10. Demostrar que si p es un número primo y a es un entero no nulo entonces o (a,p)=1 o

(a,p)=p.

11. Si p y q son primos distintos entonces (p,q)=1

12. Probar que (a,bc)=1 si y solo si (a,b)=1 y (a,c)=1.

13. Si x=yz+t , probar que (x,z)=(z,t)

14. Si a y b son primos relativos y c pertenece a los enteros positivos entonces:

i) existen α y β tales 1=αa+βb.

ii) (a-b.a+b) es 1 o 2.

iii) Si a|bc entonces a|c.

iv) Si (a|c y b|c) entonces ab|c.

v) (c,ab)=(c,a)(c,b).

15. ¿Cómo es (a2+b2,a+b) sabiendo que (a,b)=1?

16. Pruebe que si a es par y b es impar entonces (a,b)= ⎟⎠⎞

⎜⎝⎛ ba ,

2

17. Probar que si c|ab entonces c|(a,c)(b,c).

18. a) Supóngase que (a,b)=1. Pruebe por inducción que (an,b)=1 (Utilice el resultado del

problema 12).

b) Demuestre que si (a,b)=1 entonces (an,bn)=1.

c) Usando b) demostrar que si a y b son enteros tales que an|bn entonces a|b.

19. Si d=(a,b), a=a'd y b=b'd, demostrar que (a',b')=1.

20. Demostrar la proposición 4 (utilice los resultados del ejercicio 17 de la sección 6).

Page 54: A Notas de Algebra Moderna

21. Demuestre que el corolario de la proposición 2 implica lo siguiente: "Si los múltiplos de

a se marcan en rojo sobre una recta y los múltiplos de b en verde donde a y b son

enteros positivos cuyo máximo común divisor es g, entonces g ser la distancia más

corta de cualquier punto verde a cualquier otro rojo".

22. Si (ab,p)=1 demostrar que sk

k

bpapp++

+

1

1

solo cuando k=s y p|(a+b).

23. Supóngase que ba y

dc son dos fracciones reducidas a su expresión más simple

((a,b)=(c,d)=1). Demostrar que si ba +

dc =

bdcbad + es un entero entonces b=d o b=-d.

24. En base a la proposición 4 demostrar que si dos números a y b son primos relativos y su

producto es un cuadrado, entonces cada uno es un cuadrado perfecto. Deducir esta

misma proposición del resultado establecido en el ejercicio l4.

25. Definir formalmente mínimo común múltiplo. Demostrar que éste se puede obtener

multiplicando los dos números y dividiendo el producto por el máximo común divisor.

Demostrar finalmente que también se puede obtener descomponiendo en factores

primos y formando el producto de todos los primos cada uno con su mayor exponente.

26. Definir recursivamente el máximo común divisor de n números. Definir

recursivamente combinación lineal de n números. Demostrar que el máximo común

divisor de n números es la menor combinación lineal positiva de estos n números.

27. Formalizar la demostración dada para el colorario de esta sección procediendo por

inducción sobre k.

28. a) Demostrar que si b y c son enteros positivos tales que bc es un cuadrado perfecto y

(b,c)=1 entonces ambos b y c son cuadrados perfectos.

Page 55: A Notas de Algebra Moderna

b) En base a la anterior demuestre que no existen enteros a y b tales que a2=2b2 (esto

demuestra que raíz de dos no es racional!).

c) Probar que no existen enteros no nulos a y b tales que a2=3b2.

d) Si n es un entero positivo que no es cuadrado perfecto probar que no existen enteros

no nulos a y b tales a2=nb2.

29. Demostrar el teorema fundamental de la aritmética (sección 6).

Page 56: A Notas de Algebra Moderna

9. ECUACIONES LINEALES DIOFANTINAS

Un problema adivinanza típico es el siguiente: María compra pollos a $50 y patos a $70,

con un costo total de $530. Cuántos pollos y cuántos patos compró. Haciendo x el número

de pollos e y el número de patos tenemos la ecuación

50x+70y=530

que es equivalente a

5x+7y=53. (1)

Es claro que la solución x e y deben ser enteras y positivas, pues no se conciben respuestas

como 43 de ppollos y 5

27 de patos ni tampoco (-3) pollos. Ecuaciones como éstas en

que las soluciones deben ser enteras se denominan ECUACIONES DIOFANTINAS en

honor a Diofantos (S. III D.C.), matemático de la “segunda escuela alejandrina” y que es

considerado pionero del álgebra y la teoría de números. En su aritmética Diofantos da

“recetas” para resolver éstas y otras ecuaciones. Es claro que la teoría de números es el

estudio de ecuaciones diofantinas en gran parte, así pues el “Ultimo Teorema de Fermat”

establece la imposibilidad de resolución de ciertas ecuaciones diofantinas. Por ahora,

vamos a trabajar con algunas ecuaciones lineales diofantinas, como la ecuación (1). Con

los elementos que tenemos sobre máximo común divisor podemos justificar el

procedimiento que se ilustra en el siguiente ejemplo:

Ejemplo 1. Sabemos que (5,7)=1 existe según el corolario de la sección anterior una

solución a la ecuación

5α+7β=1

Sea esta α=3 y β=-2. Podemos entonces conocer una solución entera para la ecuación (1) a

saber:

x0=53α=159 y y0=53β=-106

¿Hay otras soluciones a la ecuación? Supongamos que x, y es otra solución, cómo es?

Tendríamos

Page 57: A Notas de Algebra Moderna

5x+7y=53

5x0+7y0=53

Estando estas dos ecuaciones tenemos:

5(x-x0)=7(y0-y)

Como (5,7)=1 y se tiene 5|7(y0-y).

La proposición de la sección anterior nos permite deducir que 5|(y0-y) es decir que para

algún t entero 5t=y0-y de donde tenemos que y=y0-5t=-l06-5t.

Para encontrar los valores de x reemplazamos el valor de (y0-y) en (2) por 5t y obtenemos:

5(x-x0)=7×5t

de donde x-x0=7t o sea que x=7t+x0=159+7t.

Tenemos entonces que:

x=159+7t

y=-(106+5t)

dando valores a t, obtenemos soluciones para la ecuación (1) así para t=0,1,2,3 se tiene

x=159,166,173, 180 y=-106,-111,-116,-121.

Ya habíamos dicho que nos interesan sólo las soluciones positivas. ¿Cuáles t hacen a x e y

positivos? Según (3) tendríamos:

159 +7y>0 y -106-5t>0

desigualdades que al despejar t nos indican:

t> 77159− y t> 5

106−

o sea que t debe estar entre -22.7 y -21..2 y el único valor entero posible para t ser t=-22

por lo tanto las únicas soluciones positivas son: x=5 , y=4

Este proceso es general y los formalizamos en el siguiente resultado.

Proposición 1. Sean a,b,c enteros no nulos, la ecuación

Page 58: A Notas de Algebra Moderna

ax+by=c (4)

tiene solución si y solo (a,b)|c

Demostración. Esto es una consecuencia del corolario de la sección anterior.

En el ejercicio se pide encontrar la forma general de las soluciones a la ecuación (4) cuando

estas existen. El método utilizado en el ejemplo 1 se puede expandir a ecuaciones con más

de dos variables como veremos enseguida.

Ejemplo 2. Supongamos que queremos encontrar:

5x+7y-10z=12 (5)

como (5,7)=1 por la proposición 1 tenemos que la ecuación

5x+7y=u (6)

siempre tiene solución para cualquier u entero, debemos resolver entonces, reemplazando u

en (5):

u-10z=12

que tiene solución particular u0=22 y z0=1 y por el método del ejemplo anterior vemos que

u=22+10s y z=1+s

entonces la ecuación (6) queda:

5x+7y=22+10s (7)

como para s=0, tenemos u=22, z=1, resolviendo (7) para s=0 obtenemos que x0=3, y0=1,

z0=1 es una solución particular de (5), y de (7) podemos plantear

5(x-2s)+7y=22

que nos dan las soluciones para (5) que estamos buscando:

x-2s=3+7t o sea x=3+7t+2s

y=1-5t

z=1+s

al hacer variar t y s obtenemos todas las soluciones posibles enteras.

La existencia de soluciones para ecuaciones diofantinas de más de dos variables se

establece en el resultado siguiente:

Page 59: A Notas de Algebra Moderna

Proposición 2. La ecuación diofantina

a1x1 +a2x2+...+anxn=c

tiene solución si y sólo si el máximo común divisor de a1, a2,..., an divide a c.

Demostración. Procedemos por inducción para n.

i) Para n=2 la proposición 1 nos garantiza el resultado.

ii) Supongamos que el resultado se tiene para n=k y queremos probarlo para n=k+1. Si

tenemos:

a1x1 +a2x2+...+akxk+ak+1xk+1 =c (8)

Sea d' el máximo común divisor de a1, a2,..., ak; sabemos por hipótesis de inducción que la

ecuación

a1x1 +a2x2+...+akxk=c’

tiene solución única y exclusivamente cuando d'|c', o sea cuando c'=d'x. Ahora por la

proposición 1 la ecuación

a1x1 +a2x2+...+akxk+ak+1xk+1=c

tiene solución si y sólo si (d',ak+1)|c que es lo mismo que exigir que el máximo común

divisor de a1, a2,..., ak+1 divide a c.

PREGUNTAS Y EJERCICIOS

1. Determinar una solución general de la ecuación lineal diofantina

i) 23x+37y =

ii) 2072x+1813y=2849.

2. En el plano señalar los puntos enteros de las rectas 3x-2y=2 y 3x-2y=0.

3. Determinar todas las soluciones de 19x+20y=1909 con x>0 y y>0.

Page 60: A Notas de Algebra Moderna

4. Sean m y n enteros diferentes. ¿Cuántos fraccionarios con denominador n o m hay entre

1 y 0? ¿Cuál es la menor distancia entre dos fracciones de éstas?

5. Encontrar una solución general para la ecuación

1321x+5837y+1926z=2983.

6. Cuando el Señor González en 1911 cambió su cheque por x pesos con y centavos, el

cajero se equivocó y pagó y pesos con x centavos. El Señor González recibió el doble

de la cantidad mas dos centavos. ¿De cuánto era el cheque?

7. ¿Qué tan separados están los puntos enteros de la recta

7x+5y=53

8. Demostrar que cuando (a,b)=1 entonces ab<0 si y sólo si existe un número infinito de

soluciones positivas (x>0,y>0) para la ecuación ax+by=c.

9. Resolver en forma general los siguientes sistemas de ecuaciones para x, y, z enteros.

i) 2x+3y+z=25 ii) 12x+16y-4z=4

4x+6y-2z=12 y+z=3

10. Determinar las condiciones necesarias y suficientes para que las ecuaciones

ax+by+cz=d

a'x+b'y+c'z=d'

tengan soluciones en enteros. Exhibir un método general para encontrar la forma

general de las soluciones.

Page 61: A Notas de Algebra Moderna

10. LA RELACION DE CONGRUENCIA ENTRE ENTEROS.

Con base en los resultados obtenidos en la sección 8 desarrollaremos una notación muy útil

dentro de la teoría de números, notación introducida por Gauss.

Definición 1. Siempre que m|(a-b) diremos que a es congruente con b modulo m y se notar

a≡b(mod m) (sólo se exige que m sea diferente de 0).

Esta notación puede interpretarse como que a y b al dividirse por m tienen el mismo

residuo. En efecto, si a y b tiene el mismo residuo al dividirse por m se tiene:

a=k1m+r y b=k2m+r

que implica (a-b)=(k1-k1)m, o sea que, m|(a-b).

Por otra parte, como 0 es el único múltiplo de m que está entre -m y m si

a≡b(mod m),

aplicando algoritmo de la división tendremos

a=q1+r1; b=q2+r2

con

0<r1<m y 0<r2<m;

por tanto

m|(a-b) y (a-b)=(q1-q2)m+(r1-r2)

se sigue que

m|(r1-r2)

pero r1-r2 debe estar entre –m y m ⎟⎠⎞

⎜⎝⎛ <−<

22 más es 21

mrrm- por tanto, r1-r2=0 o sea los

residuos r1 y r2 deben ser iguales.

Hemos demostrado la siguientes caracterización.

Page 62: A Notas de Algebra Moderna

Proposición 1. a≡b(mod m) si y sólo si a y b tienen el mismo residuo al dividirlos por m.

Ejemplo 1. Según el algoritmo de la división al dividir por 4 se puede obtener un único

residuo entre 0 y 3 y por lo tanto un número debe ser de una única forma: 4n, 4n+1, kn+2 o

4n+3. Esto nos ayuda a demostrar, por ejemplo, que todo número cuadrado es un múltiplo

de 4 o es de la forma 4n+1 (Proposición2 sección 6). Los números de la forma 4n, los

múltiplos de 4, son congruentes entre sí, modulo 4. Los de la forma 4n+1, por ejemplo 41

y l009, son congruentes entre sí todos. Lo mismo sucede con los de la forma 4n+2 y por su

lado con los de la forma 4n+3. Hacer congruencias modulo 4 es pues, formar los números

enteros en 4 “grupos” como se ve en la figura 1.

... -8 -4 0 4 8 ...

... -7 -3 1 5 9 ...

... 6 -2 2 6 10 ...

... -5 -1 3 7 11 ...

Figura1. Los números de cada fila son congruentes entre si modulo 4.

En estos “grupos” que se forman, la relación de congruencia modulo 4 hace el papel de

igualdad. Esto nos garantiza en forma general el siguiente resultado.

Proposición 2. La relación “ser congruente(mod 4)” es una relación de equivalencia en

los enteros, es decir, se cumplen las siguientes leyes:

Reflexiva: Siempre a≡a(mod m).

Simétrica: Si a≡b(mod m) ⇒ b≡a(mod m).

Transitiva: Si a≡b(mod m) y b≡c(mod m) ⇒ a≡c(mod m).

Demostración. A manera de ilustración hacemos la demostración de la simetría. La

reflexiva y transitiva quedan a cargo del lector.

Simetría: Si a≡b(mod m) según la definición 1, m|b-a lo que implica que m|-(b-a) o sea

m|a-b que significa que b≡a(mod m).

Page 63: A Notas de Algebra Moderna

Además de ser la relación de congruencia una relación de equivalencia, tiene otra

característica que la hace supremamente útil: es compatible con la suma y la multiplicación

de enteros. Esto es lo que indica el siguiente resultado.

Proposición 3. Si a≡b(mod m) para cualquier c entero se tiene ac≡bc(mod m) y

a+c≡b+c(mod m).

Demostración. Si a≡b(mod m) por definición m|b-a entonces m|c(b-a) y por lo tanto

m|cb-ca lo que indica que ca≡cb(mod m).

Así mismo, si m|b-a entonces m|(b+c)-(a+c) por tanto a+c≡b+c(mod m).

Ejemplo 3. Sabemos que 10≡1(mod 9) por la proposición anterior vemos que

102≡10(mod 9) y aplicando que la relación de congruencia es simétrica y transitiva vemos

que:

102≡10(mod 9) y 10≡1(mod 9) ⇒ 102≡1(mod 9)

multiplicando por el mismo número 3 vemos que 3×102≡3(mod 9) entonces

3×102+1≡4(mod 9) o sea que 301≡4(mod 9).

Resumidamente se ha visto que como 10≡1(mod 9) entonces(3(10)2+1)≡(3(1)2+1)(mod 9).

En el ejercicio 5 se pide demostrar que si a+c≡b+c(mod m) entonces a≡b(mod m). Esta es

una justificación para la ley cancelativa de la suma en congruencia. Se podría esperar tener

una ley parecida para el producto pero se puede buscar un contraejemplo rápidamente ,así

cuando m=24 1×6≡5×6 y sin embargo no es cierto que módulo 24 1≡5. La siguiente

proposición nos indica cuándo es posible cancelar factores comunes en una congruencia.

Proposición 4. Si (m,c)=1 y ac≡bc(mod m) entonces a≡b(mod m).

Page 64: A Notas de Algebra Moderna

Demostración. Si ac≡bc(mod m) entonces m|(b-a)c, como (m,c)=1 según la última

proposición de la sección 8 concluimos que m|b-a y por lo tanto a≡b(mod m).

Una generalización de este resultado se encuentra en el ejercicio 12.

Definición 2. Un conjunto de números {a0, a1, ..., am-1} es un sistema completo de residuos

módulo m si en él hay uno y sólo un representante de cada residuo al dividir por m. En

otras palabras se deben cumplir dos condiciones:

i) i≠j ⇒ ai no es congruente con aj módulo m.

ii) Para cualquier entero a existe un 0≤i<m tal que: ai≡a(mod m).

La primera condición indica que no hay en {a0, a1, ..., am-1} dos números con el mismo

residuo, la segunda condición asegura que ahí están todos los residuos posibles.

Ejemplo 2. Para buscar un sistema completo de residuos módulo 4, según la figura 1, basta

tomar 4 enteros, cada uno de una fila diferente. Así el conjunto {0,-3,6,11} es un sistema

completo de residuos módulo 4, mientras si tomamos {6,10,5,8} no es un sistema completo

de residuos pues 6≡10(mod 4) y además no hay ninguno que tenga residuo 3.

Fijemos nuestra atención en el s.c.r. {8,-3,6,11} teniendo en cuenta las proposiciones 2 y 3

vemos que:

8+(-3)≡(-3) y 6+11≡(-3) y 6+(-3)≡11, etc. y así con el producto (-3)×11≡6 y (-3) ×11≡11

y 6×11≡6, etc. Podemos resumir esto haciendo tablas de multiplicar y sumar tendremos:

Tablas 1.

En este sistema completo de residuos el 8, por ejemplo, representa todos los números que

tienen el mismo residuo que él al ser dividido por 4: todos los múltiplos de 4; -3 representa

los números de la forma 4n+1; el 6 los de la forma 4n+2 y 11 a los de la forma 4n+3. Un

+ 8 -3 6 118 8 -3 6 11-3 -3 6 11 8 6 6 11 8 -3

11 11 8 -3 6

⋅ 8 -3 6 11 8 8 8 8 8 -3 8 -3 8 6 6 8 6 8 6

11 8 11 6 -3

Page 65: A Notas de Algebra Moderna

sistema canónico de residuos equivalente al anterior sería {0,1,2,3} en donde las tablas nos

quedan:

Tablas 2.

Nótese que aquí 3×3≡1 indica que dos números de la forma 4n+3 multiplicados nos da uno

de la forma 4n+1.

Definición 3. Cuando hablemos de la aritmética modulo m nos referiremos a las

operaciones entre los números 0,1,2,..,(m-1) según la relación de congruencia (mod m).

Los cálculos en la aritmética módulo m se hacen como en los números en cuanto se

cumplen propiedades como la distributiva, las dos operaciones sin conmutativa y

modulativa etc. Sin embargo hay una diferencia importante: la ley cancelativa para el

producto es más restringida en la aritmética módulo m según la proposición 4. Por otra

parte cuando el módulo es primo podemos hablar de inversos multiplicativos lo cual no

sucede en los enteros, donde los únicos que tienen inversos multiplicativos son..... Estas

propiedades básicas son formalizadas en la siguiente afirmación.

Proposición 5. (Propiedad de la aritmética mod. m).

i) Ley cancelativa para la suma:

a+x≡a+y ⇒ x≡y.

ii) Para todo a y b existe un único x tal que:

a+x≡b.

iii) m es primo para todo a no congruente con 0 y todo b existe un único x tal que:

a⋅x≡b.

iv) Si m es primo para todo a no congruente con 0, todo b y c existe un único x tal que:

ax+b ≡ c.

+ 0 1 2 3 0 0 1 2 3 1 1 2 3 0 2 2 3 0 1 3 3 0 1 2

⋅ 0 1 2 3 0 0 0 0 0 1 0 1 2 3 2 0 2 0 2 3 0 3 2 1

Page 66: A Notas de Algebra Moderna

Demostración.

i) Si a+x≡a+y(mod m) entonces m|(a+x)-(a+y) lo que implica que m|x-y o sea que

x≡y(mod m).

ii) Vemos primero que para todo a existe (-a) tal que a+(-a)=0.

Hágase simplemente (-a)=m-a cuando a≠0 y (-0)=0. Para resolver la ecuación

a+x≡b(mod m) tómese x≡b+(-a)(mod m) y se tendrá:

a+x≡a+( b+(-a)) ≡b.

iii) Consideremos los residuos 0,a,2a,...,(m-1)a . Entre estos residuos no pueden existir

dos repetidos pues si ia≡ja como m es primo, (m,a)=1 y podemos aplicar la proposición 4

obteniendo i≡j o sea i=j. Esta consideración nos garantiza que entre 0,a,2a,...,(m-1)a a no

hay dos residuos iguales y por lo tanto 0,a,2a,...,(m-1)a es un sistema completo de residuos

modulo m entre los cuales debe estar la clase residual de b, por tanto existe un x, único

como residuo, tal que ax≡b(mod m).

La parte cuatro de la demostración se deja como ejercicio al lector.

La demostración de la parte 3, como ya se indicó, es básica y sutil. Su argumento lo

resaltamos en la siguiente proposición que ser utilizada mas adelante.

Proposición 6. Si a no es congruente con 0 módulo m cuando m es primo entonces el

conjunto 0,a,2a,...,(m-1)a es un sistema completo de residuos.

Como consecuencias de la proposición 6 encontramos la parte iii) de la proposición 5, así

como el Teorema débil de Fermat y el Teorema de Wilson, con los cuales cerramos esta

sección.

Proposición 7. (Teorema débil de Fermat) Si p no es primo y a no es múltiplo de p,

entonces:

ap-1≡1(mod p)

Page 67: A Notas de Algebra Moderna

Demostración. Según la proposición 6 los residuos 0,1,2,3,....,(m-1) son exactamente los

residuos de a,2a,3a,...,(m-1)a; salvo el orden. Por esta razón tenemos:

1×2×...× (p-1)≡1×a×2a×...×(p-1)a

lo cual indica que:

(p-1)!≡(p-1)!ap-1

y como (p-1)! no es múltiplo de p existe según la proposición 5 iii) existe un único x tal

que:

(p-1)!x≡(p-1)!(mod p).

por tanto,

ap-1≡1(mod p).

Proposición 8. (Teorema de Wilson) Si p es primo entonces:

(p-1)! ≡-1(mod p)

Demostración. Sabemos que en 0,1,2,...,(p-1) están todos los residuos módulo p y además

que todo residuo no nulo a tiene su inverso multiplicativo a-1 (ejercicio 22). Cuáles

residuos entre 1 y p-1 tienen residuo igual a si mismo, es decir, para qué x se cumple

xx≡1(mod p)? Claramente para x≡1 y x≡-1 se tiene.

¿Hay otros? Si p divide a x2-1, p debe dividir a (x-1)(x+1) o sea:

(x-1)(x+1)≡0(mod p)

pero esto sólo es posible cuando o bien x-1≡0(mod p) o bien x+1≡0(mod p) (véase ejercicio

15). Esto nos asegura que los únicos residuos que elevados al cuadrado son congruentes

con 1 son 1 y -1. O sea que cada uno tiene su inverso multiplicativo diferente salvo el 1 y

-1 (o sea m-1).

Ahora bien, como p es impar hay p-1 residuos no nulos de los cuales p-3 (salvo el 1 y -1)

tienen su inverso diferente, por tanto al multiplicar 2,3,...,(p-2) tenemos un número par de

residuos que se agrupan 2 a dos anulándose todos, por lo tanto

2×3×...×(p-2)≡1(mod p)

y tenemos que

(p-1)!≡-1(mod p).

Page 68: A Notas de Algebra Moderna

Para aclarar un poco el proceso seguido en estas últimas demostraciones analicemos un

caso concreto.

Ejemplo 3. Sea p=7 y a=4, según la aritmética módulo p (tabla 3) los elementos

0,4,2×4,3×4,4×4,5×4 y 6×4 (la fila 5 de la tabla del producto) es un sistema completo de

residuos (proposición 6) y por tanto

(4×1)×(4×2)×(4×3)×...×(4×6)≡1×2x3×...×6(mod 7)

y se tiene

46×6!≡6!(mod 7)

lo que implica que

46≡1(mod 7)

como lo asegura el Teorema débil de Fermat.

Por otro lado, según la tabla 2×4≡1 y 3×5≡1, por tanto:

6!=2×3×4×5×6≡6≡-1(mod 7)

que es el teorema de Wilson.

Tabla 3. Suma y producto módulo 7.

EJERCICIOS.

1. Demostrar que la relación de congruencia es reflexiva y transitiva.

2. Demostrar que si a≡b(mod m) y c≡d(mod m) entonces,

a+c≡b+d(mod m)

+ 0 1 2 3 4 5 6 1 0 1 2 3 4 5 6 1 1 2 3 4 5 6 0 2 2 3 4 5 6 0 1 3 3 4 5 6 0 1 2 4 4 5 6 0 1 2 3 5 5 6 0 1 2 3 4 6 6 0 1 2 3 4 5

× 0 1 2 3 4 5 6 0 0 0 0 0 0 0 0 1 0 1 2 3 4 5 6 2 0 2 4 6 1 3 5 3 0 3 6 2 5 1 4 4 0 4 1 5 2 6 3 5 0 5 3 1 6 4 2 6 0 6 5 4 3 2 1

Page 69: A Notas de Algebra Moderna

3. Hacer las tablas de adición y multiplicación módulo 11 y 12 y encontrar todos los

residuos x que en cada caso cumplan la ecuación dada:

a. 3x≡6(mod 11) b. 3x≡6(mod 12)

c. 3x≡7(mod 11) d. 3x≡7(mod 12)

e. x2≡1(mod 11) f. x2≡8(mod 12)

g. x2≡3(mod 11)

4. Qué horas indica el reloj si:

a. 29 horas antes indicaba las 11.

b. 100 hora antes eran las 2.

c. 50 horas después ser n las 6.

5. Determine la forma de todos los enteros que cumplen a la vez cada par de congruencias:

a. x≡3(mod 7) y x≡4(mod 9)

b. x≡5(mod 6) y x≡8(mod 12)

6. Explicar en términos de congruencias (módulo 4):

a. El doble de un impar sumado con un múltiplo de 4 es un número de la forma 4n+2.

b. Un número no primo de la forma 4n+3 tiene al menos un divisor diferente de él, de

la forma 4n+3.

c. Lo anterior no es cierto si cambio 4n+3 por 4n+1.

7. ¿Qué se puede concluir de que a2≡b2(mod p) cuando p es primo?

8. En la aritmética módulo m se puede hablar de algoritmo de la división?

9. Encontrar todas las triplas (x, y, z) modulo 5 tales que

x2+y2=z2

10. Demostrar que si a+b≡c+b(mod m) entonces a≡c(mod m).

Page 70: A Notas de Algebra Moderna

11. Demostrar que si n es entero positivo impar entonces

1+2+3+... +(n-1) ≡0(mod n)

12. Sea p(x) un polinomio con coeficientes enteros. Demostrar que x≡y(mod m) implica

que f(x)≡f(y)(mod m).

13. Sea (m,c)=d y m=dn; si ac≡bc(mod m) entonces a≡b(mod n).

14. Demostrar que si p es primo xp+yp≡(x+y)p.

15. Demostrar que si p es primo ab≡0(mod p) implica a≡0(mod p) o b≡0(mod p). ¿Qué se

puede decir si p no es primo?

16. Probar que cuando p es primo impar xp+yp-≡0(mod p) implica xp+yp≡0(mod p2).

17. Siendo p primo ap≡a(mod m).

18. A qué congruencia de grado inferior a 7 es equivalente la congruencia:

2x17+6x16+x14+5x12+3x11+2x10+x9+5x8+2x7+3x5+4x4+6x3+4x2+x+2≡0(mod 7)?

19. Probar el teorema débil de Fermat demostrando que

(1+1 +...+1)p=(1+1+...+1)

siempre que el número de 1's se menor que p.

20. Si a0, a1,..., am-1 es un sistema residual completo módulo m, entonces ka0, ka1,..., kam-1

también lo es. Demostrar que esto se tiene si k es primo relativo con m.

21. Deducir un resultado similar al anterior para los enteros ka0+1, ka1+1,..., kam-1+1,

22. Demostrar que si (a,m)=1 entonces:

Page 71: A Notas de Algebra Moderna

i) Existe a-1 tal que aa-1≡1(mod m).

ii) Si ax≡0(mod m) entonces x≡0(mod m).

23. Demostrar que cuando p es primo, si a0, a1,..., an no son múltiplos de p entonces

a0a1...an no es múltiplo de p.

Page 72: A Notas de Algebra Moderna
Page 73: A Notas de Algebra Moderna

1. LAS PROPOSICIONES DE LA TEORIA DE NUMEROS.

Indudablemente la base de todo ese inmenso océano que es la matemática radica en la

aritmética y la geometría. Nuestro estudio se centrará en la primera de éstas que trata,

como la Teoría de Números, de los números naturales y enteros. Se trata de analizar

proposiciones que se refieren a éstos.

El lector entonces, debe tener confianza en el estudio que inicia pues trabajará básicamente

con los elementos con los cuales indudablemente debe estar familiarizado. Sin embargo el

exceso de confianza no es conveniente pues, como comenta James R. Newman: "Se

supone comúnmente que la aritmética es la rama más sencilla de las Matemáticas. Nada

más lejos de la verdad. El tema es difícil de plantear aunque se admite que la práctica de la

aritmética elemental es bastante fácil". En efecto, comprobar la falsedad o veracidad de

proposiciones como 22+23=45 ó 425×236=263020 es una tarea de la aritmética elemental

que puede resolver cualquier alumno de tercero primaria. No se necesita más que el simple

manejo de algoritmos1 y‚ esto es lo que en la mayoría de los casos se enseña en la

matemática de la escuela primaria. La justificación de éstos y otros algoritmos, así como la

verificación de propiedades más generales exigen casi siempre mayor madurez matemática.

Así los griegos pitagóricos sabían que la suma de los n primeros números impares

coinciden con n2. Este hecho se comprueba fácilmente para casos particulares. Por

ejemplo, los primeros 4 números impares son: 1,3,5 y 7 cuya suma es 16. ¿Pero cómo

comprobar que en todos los casos se cumple?

1 Un algoritmo se puede entender como una receta o procedimiento para efectuar una tarea.

Page 74: A Notas de Algebra Moderna

Podríamos sumar los primeros 100 impares desde 1 hasta 199 y obtener 10.000 o

comprobar con ayuda de modernos computadoras para números muy grandes y muchas

veces, obteniendo siempre resultados positivos y sin embargo siempre faltarían casos y no

podremos estar convencidos absolutamente de que la proposición analizada sea cierta para

todo número natural. Con ayuda de la intuición geométrica podemos convencernos de la

certeza de la proposición tal y como razonaban los primeros matemáticos. Observando la

figura 1, se nota cómo al ir añadiendo impares siempre se obtiene un cuadrado de lado igual

al número de impares que se llevan.

Aunque este argumento no es una demostración en el sentido moderno, es indudable que es

una muy buena explicación que nos exime de cualquier intento de comprobación con

muchos números.

Vemos entonces que no vasta sumar, restar, etc. para analizar la verdad o falsedad de las

proposiciones aritméticas, y que para hacerlo es necesario utilizar otras ramas del saber

matemático, como en el caso expuesto en el cual nos hemos ayudado con la geometría. En

realidad son pocas las ramas de las matemáticas que no se utilizan en lo que hoy se

denomina "Teoría de Números" que es al fin y al cabo una aritmética avanzada y que como

se comentaba anteriormente es mucho más difícil de lo que en principio puede uno suponer.

Para dar una dimensión de los problemas centrales de esta rama de la matemática -que no

se trabajarán en este texto-, comentaremos por ahora el famoso "Último teorema de

Fermat" como un problema planteado hace más de tres siglos y que a pesar de los grandes

1 1+3 1+3+5 1+3+5+7 Figura 1. Argumentación geométrica para comprobar que la suma de los primeros

impares es un número cuadrado perfecto.

Page 75: A Notas de Algebra Moderna

esfuerzos dedicados y a los avances de otras ramas de la Matemática, no ha podido ser

resuelto.

Alrededor de 1637 Fermat, jurista y parlamentario francés cuya diversión era las

matemáticas y del cual el lector oirá hablar mucho, escribió al margen de un libro -la

Aritmética de Diofantus-: "Es imposible descomponer un cubo como la suma de dos cubos,

una cuarta potencia o en general cualquier potencia como la suma de dos potencias del

mismo orden mientras éste sea mayor que dos, y ciertamente he encontrado una

demostración magnífica de esto pero el margen es demasiado pequeño para contenerla".

En otras palabras lo que Fermat aseguraba tener demostrado era que la ecuación:

xn+yn=zn no tiene solución para x, y, z, enteros no nulos y siendo n un entero mayor que 2.

Esta sencilla proposición no ha podido ser demostrada ni refutada, a pesar del gran avance

de la matemática y de los grandes matemáticos que lo han intentado. Nótese que para n=2

existen muchas triplas de enteros x, y, z que resuelven la ecuación (por ejemplo, x=3, y=4,

z=5). Estas reciben el nombre de Triplas Pitagóricas.

Esta pequeña introducción ha querido mostrar al lector nuestro objeto de estudio, los

números enteros, objetos con los cuales estamos familiarizados desde niños y que a pesar

de haber sido trabajados por la humanidad por siglos y siglos contienen gran cantidad de

misterios. Por esto la Aritmética coronada por Gauss como la reina de las matemáticas es

al decir de Bell "el último gran continente salvaje de las matemáticas".

PREGUNTAS Y EJERCICIOS.

1. La suma de los primeros n números forman los números triangulares como se muestra

enseguida:

Page 76: A Notas de Algebra Moderna

a. "La suma de dos números triangulares seguidos da un número cuadrado perfecto".

Por ejemplo, l0+l5=52 mientras 10+6=42. Explique con un argumento geométrico

él por qué de esta proposición.

b. Completando el cuadrado demuestre que el n-ésimo número Triangular es n(n+1)/2.

2. Las propiedades que hoy en día se exponen sobre las operaciones entre números se

pueden argumentar de manera geométrica. Por ejemplo la famosa identidad

(a+b)2=a2+2ab+b2 se explica por medio de dibujo de la figura:

a. Argumentar de manera geométrica la propiedad distributiva: a(b+c) = ab +ac.

b. Lo mismo para las siguientes identidades: (a-b)b=ab-b2, (a+b)(a-b)+b2 = a2.

3. ¿Qué civilizaciones anteriores a la griega conocieron las triplas Pitagóricas?

4. Si x, y, z es una tripla pitagórica también son triplas pitagóricas sus dobles y en general

kx, ky, kz también para k entero positivo. ¿Por qué‚? La tripla kx, ky, kz es un múltiplo

de x, y, z. Muestre una tripla pitagórica que no sea múltiplo de la ya citada 3, 4, 5.

a b

a+b

b2

a2

Page 77: A Notas de Algebra Moderna

2. OTROS PROBLEMAS FAMOSOS.

En la aritmética como en la geometría los griegos constituyen la primera gran ruptura que

el hombre conoce en la historia de la matemática, pues no se conoce cultura anterior que

haya comprendido el concepto de 'demostración'. Más aun, la aristocracia griega

despreciaba la aritmética práctica que se aplicaba al comercio y que llamaban logística, tal

vez como reacción o muestra de admiración ante el gran edificio que se construía sobre la

aritmética teórica. Euclides en sus célebres ELEMENTOS DE GEOMETRIA dedica cuatro

de los trece libros a este tema. La siguiente proposición es un magnífico ejemplo de la

elegancia y acierto de la matemática helénica.

Proposición 1. Los números primos son infinitos.

Demostración. Supongamos lo contrario, es decir que sólo hay un número finito de primos

lo que implica que existe una enumeración de tales números digamos p1, p2, p3, ...,pn.

Consideremos el número m=p1×p2×p3×...×pn+1. Este número no puede ser primo por

cuanto estamos suponiendo que todos los primos son p1, p2, p3, ...,pn y m es mayor que

todos ellos, entonces m debe ser no primo y se debe dejar dividir por al menos uno de los

primos digamos pi. Pero esto tampoco es posible (¿Por qué?) Tenemos entonces que m no

es compuesto y tampoco es primo, contradicción que nos obliga a aceptar que hay infinitos

números primos.

La demostración de esta proposición es un bonito ejemplo del método de demostración por

reducción al absurdo que consiste en suponer que la conclusión del teorema no es cierta lo

cual nos lleva por medio de razocinios lógicos a algo que sabemos es falso, obligándonos a

aceptar la conclusión del teorema. Más adelante profundizaremos sobre este método de

demostración.

Page 78: A Notas de Algebra Moderna

A pesar de sus grandes avances las matemáticas griegas no fueron en ningún momento

acabadas por cuanto dejaron abiertos a la humanidad interesantes problemas que ellos no

pudieron resolver en torno a los cuales ha girado buena parte del trabajo posterior. De estos

problemas los que se refieren a la geometría han sido todos resueltos, en diferentes épocas y

tras laboriosos esfuerzos. Sin embargo, hay problemas de la aritmética que permanecen

después de dos mil años aun en el misterio.

Un ejemplo de estos misterios se refiere a los números perfectos: un número perfecto es

aquel que coincide con la suma de sus divisores positivos y menores que él. El primer

número perfecto es 6=1+2+3 y el segundo es 28=1+2+4+7+14. Todos los números

perfectos que se conocen desde los griegos hasta nuestros días son pares, pero nadie ha

podido demostrar que no existen números perfectos impares (¿habrá?) Este es el primer

gran interrogante, si existen o no perfectos impares, pero sobre los perfectos pares tampoco

se sabe mucho! Como se verá mas adelante Euclides demostró que todo perfecto par es de

la forma 2c(2c+1-1) en donde 2c+1-1 es primo. Esta es una caracterización de los perfectos

pares por cuanto todo número de esta forma es necesariamente un perfecto par. Surge

entonces el importante problema de saber cuantos números de la forma 2c+1-1 son primos.

En 1644 el fraile franciscano Martín Mersenne (1588-1648) aseguró que 2n-1 es primo

solamente para los primos n=2,3,5,7,13,19,31,67,127 y 257. En 1880 se demostró que para

n=61 se obtiene un número primo contradiciendo hipótesis de Mersenne, por lo cual se

supuso que 67 solamente era un error de algún copista negligente por 61. Pero en 1903,

Cole demostró que para n=67 se obtiene un número compuesto comprobándose

definitivamente que Mersenne se equivocó.

En 1947 se habían encontrado cinco fallas en la lista de Mersenne. Hoy en día se sigue

trabajando para conseguir un criterio que nos indique cuándo un número de la forma 2n-1,

con n primo, es primo. Además con la ayuda de los modernos computadores se conocen

más de 30 primos para los cuales el correspondiente número de Mersenne es primo. Por

ejemplo en 1988 se descubrió que para n=110503 el número de Mersenne correspondiente

(que tiene 33265 cifras decimales!) es primo.

Page 79: A Notas de Algebra Moderna

Muchos han sido los esfuerzos para encontrar una fórmula que produzca sólo números

primos. Buscando tal fórmula Fermat pensó erróneamente que los números de la forma:

122 +n

cuando n=1,2,3..., son primos. Se necesitó casi un siglo para que Euler demostrara que

para n=5 el número correspondiente de Fermat no es primo y sólo hasta 1880 Next

demostró que para n=6 la hipótesis de Fermat tampoco se tiene. Hoy en día se conocen

muchos números de Fermat que no son primos y se saben criterios para determinar cuando

un número de Fermat es primo. Pero los números de Fermat fueron más importantes de lo

que él mismo pensó. Gauss los utilizó para caracterizar los polígonos regulares que se

pueden construir con regla y compás, caracterización que en 1801 resolvía por completo

uno de los problemas planteados por los griegos referentes a la geometría.

Pero Fermat no sólo planteó problemas que han sido dolor de cabeza para los matemáticos.

Demostró también muchos resultados que hoy en día son clásicos. Por ejemplo los

números primos impares son de la forma 4k+1 o 4k-1; se puede demostrar por un método

parecido al de Euclides (proposición 1), que los primos de la forma 4k-1 son infinitos. No

tan fácil veremos más adelante que los primos de la forma 4k+1 también son infinitos.

Fermat demostró que todo primo de la forma 4k+1 se puede escribir como la suma de dos

cuadrados mientras que ningún primo de la forma 4k-1 se puede expresar así.

Otro de los resultados de este gran matemático es el llamado "Primer teorema de Fermat"

(o "débil”) que indica que si n es cualquier número que no se deja dividir por el primo p

entonces el número np-1-1 es divisible por p. Este resultado, piedra angular de la Teoría de

Números, junto con el teorema de Wilson serán demostrados más adelante. El teorema de

Wilson asegura que si p es primo el producto de los antecesores de p sumado con 1 es un

número divisible por p.

Refirámonos finalmente, a la conocida conjetura de Goldbach (1690-1764) quien en 1742

en una carta escrita a Euler, conjeturó que todo entero positivo par mayor que 2, es la suma

de dos primos y que todo entero positivo impar mayor que 5, es la suma de tres primos. La

Page 80: A Notas de Algebra Moderna

hipótesis para los números pares se ha comprobado para números menores que un millón

pero parece estar muy remota una prueba general.

Aunque no es nuestra intención atacar estos problemas famosos, esperamos que esta breve

incursión histórica de una idea de la dimensión que tiene esta rama de la matemática, de la

cual este libro no pretende ser sino una modesta introducción.

PREGUNTAS Y EJERCICIOS.

1. Explique cuales fueron las primeras actividades del hombre que lo llevaron a

profundizar en el estudio de las propiedades de los números.

2. Por qué‚ los primeros conocimientos sobre las propiedades de los números fueron

considerado mágicos y guardados en secreto por diferentes sectas religiosas? (Aún hoy

en día para mucha de ellas los números guardan ese carácter mágico).

3. Hacer una lista de los primeros cinco números perfectos pares.

4. Expresar como suma de cuadrados perfectos: l3, 29, 53, 101.

5. Demostrar agotando todos los casos que el primo 127 no se puede expresar como la

suma de dos cuadrados.

6. Demuestre que si n es par diferente de 2, 2n-1 no es primo.

7. Demostrar que 3428-1 es divisible por 29.

8. Se define n! (que se lee "n factorial") como el producto de todos los antecesores de n

incluyendo el mismo n, así:

5!=5×4×3×2=120

6!=?

Page 81: A Notas de Algebra Moderna

Formule el Teorema de Wilson en términos de factoriales. Qué se puede decir de la

siguiente proposición: "n siempre divide a n!".

Qué de la proposición: "si n es menor que m entonces m divide a n!". Compruebe el

teorema de Wilson para los primeros cinco primos.

9. Descomponer los siguientes pares como la suma de dos números cada uno primo: 86,

142, 210.

10. En qué‚ consiste la tabla de Eratóstenes para construir primos? Construir la tabla de

Eratóstenes para números menores que 200.

Page 82: A Notas de Algebra Moderna

1a matemática es la ciencia que obtiene conclusiones

necesarias B. Pierce-

3. LO NECESARIO Y LO SUFICIENTE

Supongamos que el lector está familiarizado con los conectivos lógicos que se dan entre

proposiciones, tales como la negación (∼), la conjunción (∧), la disyunción (∨), la

implicación y la equivalencia (⇒, ⇔), así como con los cuantificadores universal y

existencial. Diremos algo sobre la implicación y la equivalencia que nos sirve para analizar

ciertas técnicas de demostración.

La mayoría de las proposiciones de la matemática son de tipo "si p entonces debe cumplir

q", resumiremos p⇒q, en donde p juega el papel de hipótesis y q el de tesis o conclusión.

Se pueden dar diferentes versiones idiomáticas de éste conectivo lógico; en español se usa

"si p entonces q", "p implica q", "q siempre que p", "para que suceda p es necesario que q",

"para que q es suficiente que p", etc. Sea p, por ejemplo, la proposición "a y b son pares" y

q la proposición "a+b es par", p⇒q se puede leer: "Si a y b son pares entonces a+b

también lo es", como quien dice "la suma de dos pares es un par", o, "condición suficiente

para que a+b sea par es que a y b sean pares".

La recíproca de la proposición p⇒q es la proposición q⇒p que en general tiene diferente

valor de verdad. En el ejemplo anterior mientras "la suma de pares es par" es una

proposición cierta, su recíproca "si la suma de dos números es par entonces ambos números

son pares" es falsa, puesto que 5+3 es par siendo uno de los sumandos impar.

La contrarrecíproca de la proposición p⇒q es la proposición ∼q⇒∼p que es equivalente a

la original, por lo tanto, para demostrar una implicación podemos demostrar su

contrarecíproca.

Page 83: A Notas de Algebra Moderna

Así, para, mostrar que "todo par elevado al cuadrado es par" (n par implica n2 par) podemos

mostrar que "si el cuadrado de un número es impar, el número debe ser impar" (n2 no par

implica n no par) que es equivalente.

En el ejemplo inicial la contrarecíproca de "la suma de dos pares es un par" es la

proposición "si la suma de dos números no es par entonces ambos números no pueden ser

pares" o lo que es lo mismo "si la suma de dos números no es par entonces alguno de los

números es impar".

Muchas veces la hipótesis o la tesis viene en forma de conjunción o disyunción. Por

ejemplo la forma (p∧q)⇒r, que es la forma de la proposición que acabamos de analizar.

En efecto, si convenimos en que p, r, q sean las proposiciones "a es par", "b es par", "a+b

es par" respectivamente, se ve más claramente porqué la contrarecíproca tiene como

conclusión que alguno de los números es impar, ya que la negación de p∧q es ∼p∨∼q y la

forma de la contrarecíproca ser ∼r⇒ (∼q∨∼p).

Probar la contrarecíproca es hacer la prueba por contradicción: Para demostrar p⇒q se

supone que la conclusión no es cierta o sea ∼q y se deduce que la hipótesis fallaría o sea

que ∼p; se está demostrando que ∼q⇒∼p.

Otra propiedad que nos interesa resaltar de la lógica de proposiciones es que la negación de

una implicación p⇒q es equivalente a ∼(p∧∼q). Esta es la razón para que negar la

proposición "la suma de dos números es impar implica que ambos son pares", sea afirmar

que "existen números cuya suma es par sin que ambos sean pares". La equivalencia entre

p⇒q y ∼(p∧∼q) nos ayuda también a explicar las demostraciones por contradicción: Se

trata de ver que es imposible que se cumpla la hipótesis sin que se cumpla también la tesis.

Cuando tanto p⇒q como su recíproca q⇒p, son ciertas se dice que p y q son equivalentes y

se nota p⇔q. Por ejemplo, "n2 es par" y "n es par" son proposiciones equivalentes, pues

tanto "si n2 es par entonces n es par" como "si n es par su cuadrado también lo es" son

proposiciones ciertas.

Page 84: A Notas de Algebra Moderna

Otras versiones idiomáticas para esta equivalencia son: "a es par sí y sólo sí a2 lo es" o

"condición necesaria y suficiente para que n2 sea par es que n lo sea". La equivalencia

también se utiliza en las definiciones, por ejemplo para definir par podemos decir "n es par

sí y sólo sí existe un entero k tal que a=2k".

Las equivalencias lógicas (tautológicas) son válidas por su forma sin importar el contenido

de las proposiciones 'internas'. Así, "k no es primo par sí y sólo sí k no es primo o k es

impar", es una equivalencia válida por su forma pues ∼(p∧q)⇔(∼p∨∼q) es cierta sin

importar el valor de verdad de p y q. La tabla 1 muestra una lista de las principales

equivalencias lógicas. Digamos para terminar esta sección, que siempre que p y q sean

equivalentes la proposición p se puede reemplazar por q y el revés. La equivalencia es, con

respecto a las proposiciones, como una igualdad.

∼(∼p) ⇔ p Doble negación es afirmación.

p∧(q∧r)

p∨(q∨r)

(p∧q)∧r

(p∨q)∨r

La conjunción y la disyunción son

asociativas.

p∨(q∧r)

p∧(q∨r)

[(p∨q)∧(p∨r)]

[(p∧q)∨(p∧r)] Disyuntiva.

∼(p∧q) ⇔ (∼p∨∼q) Negación de ∧.

∼(p∨q) ⇔ (∼p∧∼q) Negación de ∨.

(p⇒q) ⇔ (∼q⇒∼p) La contrarecíproca es equivalente a la

original.

∼(p⇒q) ⇔ (p∧∼q ) Negación de la implicación.

((p∧q)⇒r) ⇔ ((p∧∼r)⇒ ∼q)

((p∧q)⇒r) ⇔ (∼r⇒(∼q∨∼q))

TABLA 1: Algunas equivalencias lógicas importantes.

Page 85: A Notas de Algebra Moderna

PREGUNTAS Y EJERCICIOS

1. Dar otras versiones idiomáticas de p⇒q. Cómo se dice en inglés?

2. Sean p,q,r,s las siguientes proposiciones:

p: x es par

q: y es par

r: x+y es par

s: x2+y2 es par

Según esto, identifique cada proposición de la izquierda con su respectiva fórmula a la

derecha.

a. Si x es par, y es par entonces x+y es par. a. (∼q∧r)⇒p

b. Es imposible que x2+y2 sea par, siendo x e

y impares.

b. ∼s⇒(∼p∨∼q)

c. Condición necesaria para que x2+y2 sea

impar es que alguno de ellos sea impar.

c. (p∧q)⇒r

d. Condición suficiente para que x sea impar

es que x+y sea par siendo e impar.

d. ∼(s∧∼p∧∼q)

3. La recíproca de la contrarrecíproca es equivalente a la recíproca. Dé un ejemplo.

4. Las siguientes proposiciones son todas falsas. Muestre en cada caso un contraejemplo:

a. Si a2 no es par a3 sí lo es.

b. n2+2n siempre es par.

c. Si n es primo 2n-1 también lo es.

Page 86: A Notas de Algebra Moderna

d. Si n es positivo n3-6n2+11n-6=0.

5. Expresar en forma de implicación el último teorema de Fermat.

6. ¿Cuál es el recíproco del teorema de Wilson?

7. De las proposiciones siguientes señale aquellas equivalencias al primer teorema de

Fermat:

a. Todo número primo p mayor que 2 divide a np-1-1 y no divide a n.

b. Siendo p un primo mayor que 2, si p no divide a pp-1-1 entonces p divide a n.

c. Si p es número mayor que 2 que no divide a n y no divide a np-1-1 entonces p no es

primo.

8. Cada proposición de la izquierda tiene una proposición a la derecha que es lógicamente

equivalente. Señálela:

a. Si un número es par su cuadrado

también lo es.

a. Condición suficiente para que el

cuadrado de un número sea par, es

que el número lo sea

b. La suma de dos impares es par. b. No hay impares de cuadrado par.

c. Si no es par no es múltiplo de 4. c. No existen impares cuya suma sea

impar.

d. Condición necesaria para ser par es

ser múltiplo de 4.

d. Los múltiplos de 4 son siempre

pares.

e. Todo número cuyo cuadrado es par,

es par.

e. Condición suficiente para ser

múltiplo de 4 es ser par.

Page 87: A Notas de Algebra Moderna

9. Analice la demostración de Euclides de la infinitud de los primos (proposición 1

sección 2) en términos de una implicación entre las proposiciones "A es el conjunto de

todos los números primos" y "A es infinito".

Page 88: A Notas de Algebra Moderna

4. PRIMERAS PROPOSICIONES SOBRE DIVISIBILIDAD

Demostremos a continuación algunas proposiciones más con el fin de familiarizar al lector

con el concepto de prueba que con la idea de avanzar en la teoría. Para ello nos

fundamentaremos solamente en las propiedades que se conocen de los números a través del

álgebra elemental. Empecemos por unas definiciones que despejan pequeñas dudas sobre

términos a emplear.

Definición 1. Un número n es par sí y sólo sí existe un entero k tal que n=2k. Se dice

también que n es múltiplo de 2 o que 2 divide a n.

n es múltiplo de m sí y sólo sí existe k entero tal que n=mk.

Se dice en este caso que m divide a n o que n es múltiplo de m y se nota m|n.

Un número p mayor que 1 es primo si sus únicos divisores positivos son 1 y el mismo p.

Caso contrario, p se dice compuesto.

NOTA: El 1 no se considera ni primo ni compuesto.

La demostración de las das primeras proposiciones son ejemplos de demostraciones

directas, que consisten simplemente en traducir las hipótesis para que después de una leve

manipulación algebraica se encuentre la tesis formulada.

Proposición 1. La suma de dos múltiplos de k es un múltiplo de k.

Demostración. Sean u y v los múltiplos de k, según la definición existen n y m tales que

u=nk y v=mk entonces

u+v=(n+m)k

Page 89: A Notas de Algebra Moderna

lo que nos indica que u+v es múltiplo de k, pues se puede expresar como un número (n+m)

multiplicado por k.

Proposición 2. Si m es múltiplo de k su cuadrado también lo es.

Demostración. Si m es múltiplo de k, existe un n tal que m=nk entonces m2=n2k2 lo que

implica que

m2=(n2k)k

o sea, que m2 también es múltiplo de k.

Corolario. El cuadrado de un par es par.

Demostración. Esto es sólo una particularización (cuando k=2) de la proposición 2.

Este corolario junto con la recíproca nos produce la proposición siguiente que

demostraremos por contradicción.

Aceptamos como cierto que cualquier número entero es par (de forma 2k) o bien impar (de

la forma 2k+1).

Proposición 3. Un número es par sí y sólo sí su cuadrado también lo es.

Demostración. Por el corolario anterior se tiene que si n es par su cuadrado también lo es.

Debemos demostrar que si n2 es par entonces n es par. Si no fuera así (suponiendo lo

contrario), es decir si n2 es par siendo n impar, entonces n sería de la forma 2k+1 y su

cuadrado ser de la forma 2(2k2+2k)+1, o sea, impar. Esto es contrario a lo supuesto,

concluimos entonces que n debe ser par.

Esta última demostración se puede hacer por el método directo (ejercicio). Muchas de las

proposiciones que se demuestran por contradicción se pueden demostrar directamente, sin

Page 90: A Notas de Algebra Moderna

embargo, muchas veces el absurdo es un método de demostración es irremplazable tanto

por la claridad en la exposición como por la dificultad para plantearla directamente.

Piénsese en la demostración de la infinitud de los primos dada por Euclides (proposición 1

sección 2) y nótese la dificultad (imposibilidad?) para ser hecha directamente. La siguiente

proposición es una de esas en donde para su demostración es claramente ventajoso utilizar

el método por reducción al absurdo (por contradicción).

Proposición 4. Si un número n mayor que 1 es compuesto entonces tiene por lo menos un

divisor ente 1 y n .

Demostración. Supongamos lo contrario, es decir que n es compuesto pero que todos sus

divisores no triviales (diferentes de n y 1) son mayores que n . Sea entonces k uno de

tales divisores, por definición existen tal que n=nk. Como m también es un divisor de n no

trivial por la suposición que hemos hecho tenemos

m> n y k> n

Según las leyes de las desigualdades se deduce que

mk> n n

o sea que n>n lo cual es imposible! Concluimos que n debe tener divisores no triviales

menores que n .

En lo que se sigue utilizaremos las siguientes definiciones:

Definición 2. Diremos que a y b son de igual paridad si ambos son pares o ambos

impares.

Si n es entero positivo se define n! (se lee n factorial) como el producto de n por todos sus

antecesores positivos (ver ejercicio 8 sección 2).

EJERCICIOS Y PREGUNTAS

Page 91: A Notas de Algebra Moderna

1. Demostrar que la suma de impares es par.

2. a. Demostrar que la diferencia de múltiplos de k es un múltiplo de k.

b. En la demostración de la infinitud de los primos (proposición 1 sección 2) ¿Dónde

se utiliza este hecho?

3. Demuestre que la diferencia de dos números es par sí y sólo sí ambos números son de

igual paridad.

4. Los únicos números que no tienen sino un divisor son ...

5. ¿Qué números son divisores de 0? ¿Qué números se pueden decir que son divisibles por

0 según la definición 1?

6. A es un conjunto de números, decimos que A es cerrada para cierta operación, si todo

par de éstos al operarlos producen un elemento de A. De ésta forma, la proposición 1

de esta sección prueba que los pares son cerrados para la suma.

a. Pruebe que los impares son cerrados para el producto, pero no para la suma.

b. Pruebe que los números de la forma 4n+1 no son cerrados para la suma pero si para

el producto.

7. Pruebe que los números de la forma 4n+2 son el doble de números impares.

8. Para cada una de las siguientes proposiciones decir si son falsas o ciertas, justificando

cada respuesta con una demostración o un contraejemplo según el caso:

a. n siempre es divisible de (n+1)!

b. Si n no es un primo entonces divide a (n-1)!

c. k divide siempre a cualquier múltiplo de k.

d. Si n es múltiplo de k entonces para cualquier h se tiene que k divide a nh

e. k siempre divide a nk

f. Los números de la forma 4n+2 son cerrados para el producto.

Page 92: A Notas de Algebra Moderna

g. Para todo entero n se tiene que n(n+1) es par.

h. Los números de la forma 2n son cerrados para la suma y para el producto.

i. Un número es divisible por 12 sí y sólo sí es divisible por 3 y por 2.

9. ¿Qué opina de la demostración siguiente, en donde se intenta probar el reciproco del

teorema de Wilson (ejercicio 6 sección 3)? Sea n un número mayor que 1 compuesto

entonces n divide a (n-1)! Hacemos m=(n-1)!+1. Si n divide a m entonces divide a la

diferencia m-(n-1)! (por dividir a ambos miembros), pero está diferencia es 1, lo que

nos lleva a una contradicción que nos obliga a aceptar que si n es primo n no divide a

(n-1)!+1. contradicción que nos obliga a aceptar que si n es primo n no divide

Page 93: A Notas de Algebra Moderna

5. ALGUNOS PASATIEMPOS IMPOSIBLES.

Hemos visto en el numeral anterior algunas propiedades de los números que se refieren a la

divisibilidad, especialmente por 2, de tal forma que podemos decir que dominamos la

aritmética de los pares e impares.

Vamos a mostrar sencillas aplicaciones de estos resultados para completar el vistazo

general que queremos dar al principio de este libro, aunque hacemos la aclaración que el

objetivo de éste no incluye mostrar las posibles aplicaciones de la aritmética y la teoría de

números, que son muchas. Las que trabajaremos aquí se refieren a algunos juegos o

pasatiempos en los cuales se descubre de manera contundente y con un análisis muy

simple, ciertos casos en los cuales no hay solución.

A llenar el cuadrado... Consideremos un rectángulo dividido en n×m cuadrados iguales. Por

ejemplo en la figura 1 el rectángulo se divide en 4×7 cuadrados. En un rectángulo de este

tipo se escogen dos cuadros arbitrarios denotados por I y F (inicial y final).

Se trata de partir del cuadro inicial para llegar

al final, moviéndose cada vez un sólo cuadro

pero siempre horizontal o verticalmente, de tal

forma que al final se haya pasado por cada uno

de los cuadros restantes una única vez.

Así en la figura 2 se muestra una solución

sencillísima que puede animar al lector para

que busque una solución cuando se trata de

ir de I a hasta F'. Si el lector no encuentra

una solución podrá ofrecer con toda

seguridad gran cantidad de dinero a algún

amigo para que le ayude a resolverlo.

I

F’ F

Figura 1.

Figura 2.

Page 94: A Notas de Algebra Moderna

En realidad, aunque el problema planteado

de ir de I hasta F sea tan fácil de resolver,

cuando se trata de ir desde I hasta F' es

imposible.

¿Por qué? Veamos: coloreando los cuadros a manera de un tablero de ajedrez con cuadros

blancos y negros (figura 3) se nota que para ir de un cuadro a otro del mismo color se

necesita un número par de pasos, es más: "para ir de un cuadro a otro se necesita un número

par de pasos si y solo si los cuadros son del mismo color". Además también es fácil darse

cuenta que "en k pasos se recorren k+1 cuadros". Combinando estos hechos tenemos una

explicación del por qué ciertos juegos son imposibles: "Si el número de cuadros a recorrer

es par, el cuadro inicial y el cuadro final deben ser de diferente color, mientras si es impar

los cuadros final e inicial deben ser del mismo color". Nótese que cumplirse la condición

no nos asegura que el problema tenga solución, pero si no se cumple la condición es

garantía para que el pasatiempo sea imposible de resolver.

Dibujar sin levantar el lápiz: Es un pasatiempo que conocemos desde la infancia, a veces

fácil, a veces difícil y otras imposible. En la figura 4 se muestran varios ejemplos.

a. b. c. d. e.

Figura 4. Grafos para dibujar sin levantar el lápiz y sin repetir línea.

Estos objetos que la matemática llama

gráficos o grafos se componen de vértices y

arcos. A cada arco le corresponden dos

vértices v1 y v2 que son los extremos del

arco. Si v1=v2 se forma un bucle (figura 5).

Figura 5. Bucle.

Figura 3.

1 B 2

A 3

Page 95: A Notas de Algebra Moderna

Los arcos los notaremos con números y los vértices con letras mayúsculas. Así el grafo de

la figura 3. a) tiene dos vértices A y B y tres arcos 1,2 y 3. En él todos los arcos tienen los

mismos vértices extremos A y B.

El número de arcos que caen sobre un vértice se llama orden del vértice. En el grafo de la

figura 3.a) el grado del vértice A es 3, igual al del vértice B.

La primera condición para que un grafo se pueda dibujar como queremos es que cualquier

par de vértices se puedan unir por medio de una sucesión de arcos; en términos

matemáticos que el grafo se conexo, en términos intuitivos, simplemente se exige que el

grafo debe estar "junto", no debe haber pedazos aislados. Es claro entonces que si no es

conexo debe haber por lo menos dos vértices que no se pueden conectar por ningún camino

y por lo tanto es imposible resolver el problema. Ser conexo es condición necesaria pero

no suficiente para que el grafo se pueda recorrer pasando por todos los arcos sin repetir

ninguno y sin levantar la mano. Por ejemplo el grafo de la figura 3.b) es conexo pero es

imposible recorrerlo de la manera exigida. Buscamos pues una condición más fuerte que se

convierta en suficiente.

Formalicemos más nuestro lenguaje. Notaremos XnY (donde X e Y representan vértices y n

es un número correspondiente a un arco) el hecho que el arco n tenga vértices X e Y. En

estos términos, que un vértice A se pueda conectar con el vértice B, significa que hay una

sucesión de k+1 vértices A=X0,X1,...,Xk (que pueden estar repetidos) y de k arcos n1,...,nk tal

que

An1 X1,X1n2X2, ...,Xk-1nkB

Ahora bien, si el grafo tiene k arcos y se puede dibujar de la forma descrita partiendo del

vértice I y terminando en el vértice F, entonces los k arcos se podrán colocar de tal forma

que para determinados vértices V1,V2,...,Vk-1, suceda:

In1 V1,V1n2V2, ...,Vk-1nkF (1)

Page 96: A Notas de Algebra Moderna

Nótese ahora que cada uno de los vértices Vi, aunque puede aparecer más de una vez,

perdón, mas de dos veces, siempre aparecer un número par de veces (salvo I y F): en

efecto, siempre que se de la situación Vi-1niXi, enseguida tenemos,

Vini+1Vi+1.

Por otra parte, en (1) aparece cada vértice tantas veces como arcos existan con algún

extremo en él, es decir, tantas veces como el orden del respectivo vértice. Se deduce que

cada vértice "interno" debe tener orden par y se tiene el siguiente resultado:

Proposición 1: Para que un grafo conexo pueda ser dibujado sin levantar el lápiz y sin

repetir línea es necesario que salvo los vértices inicial y final, los demás sean todos de

orden par. Esta condición que no sólo es necesaria sino también suficiente es uno de los

primeros resultados en lo que hoy se conoce como Teoría de Grafos y fue establecido por

Euler en 1735. La suficiencia de la condición no la podemos demostrar con los conceptos

introducidos en este libro. El lector interesado puede consultar cualquier libro elemental de

Teoría de Grafos.

El juego del solitario: Este último juego que analizaremos es de remoto origen aunque la

primera referencia que se conoce de él la debemos a Leibniz. Consideremos un tablero en

forma arbitraria repartido en cuadros arreglados en filas y columnas.

Figura 6.

Page 97: A Notas de Algebra Moderna

En determinados cuadros del tablero aparecen piezas de juego, a lo m s una por cuadro. Un

movimiento, o salto, es posible cuando sobre tres cuadros A, B y C adyacentes sobre una

fila o una columna, hay piezas sobre A y sobre B pero no sobre C. El salto consiste en

mover la pieza que está sobre A hasta C retirando del juego la ficha que estaba sobre B

(Figura 6).

El objeto del juego es llevar las fichas hasta cierta y determinada configuración;

generalmente se debe dejar una única ficha en el tablero (es claro que en la posición inicial

debe haber por lo menos un lugar vacío).

Lo que queremos al aplicar los conceptos de teoría de números es averiguar si algún juego

propuesto es de imposible solución o si el resultado est determinado de alguna manera.

Deseamos escribir una ecuación que describa el proceso del juego y que tendrá como

variables el número de piezas en juego y el número de saltos; es pues natural utilizar

números enteros porque no concebimos "medio salto" o "siete cuartos de fichas".

Para facilitar nuestra tarea coloreamos los cuadros de cada diagonal con tres colores

diferentes. La primera diagonal se colorea digamos de verde, la siguiente de azul y la que

viene de rojo, para seguir con la siguiente que coloreamos de verde y continuar de manera

cíclica: azul, rojo, verde; azul, rojo, verde; etc. La figura 7 muestra un rectángulo 7×5

donde cada cuadro se ha marcado con las letras V, A o R , según se haya coloreado de verde

azul o rojo en el procesos que acabamos de describir.

V A R V A

A R V A R

R V A R V

V A R V A

A R V A R

R V A R V

V A R V A

Figura 7.

Page 98: A Notas de Algebra Moderna

Veamos en este ejemplo una sucesión de saltos permitidos como los que se muestran en la

figura 8. En la tabla 1 se indica en cada posición cuántas fichas están colocadas en cada

color. Vemos que en cada salto crece un color y decrecen los otros dos: crece el color a

donde la ficha desplazada llega y decrecen el valor del color donde estaba y del color de la

que fue sacada del juego. Realmente esta observación es válida en general, podemos

afirmar que: "Si por efecto de un salto se llega a determinado color, el número de fichas

correspondiente a ese color crece una unidad y los otros colores decrecen cada uno,

también en una unidad" .

Verdes Azules Rojos Total

Inicio 11 12 11 34

1 12 11 10 33

2 11 10 11 32

3 12 9 10 31

4 11 10 9 30

Tabla 1. Se muestra los cambios de valores en la sucesión de la

figura 8 según la coloración de la figura 7.

Por esta razón llamando V, A y R el número de fichas que están inicialmente en cada color

y V',A', y R' el número de fichas en cada color finalmente y v, a y r el número de saltos que

terminan en cada color, se debe cumplir:

• • • • • • • • • • • • • • • • • • • • • • • • • • • • • • • • • • •

Inicio

• • • • • • • • • • • • • • • • • • • • • • • • • • • • • • • • • •

Posición 1.

• • • • • • • • • • • • • • • • • • • • • • • • • • • • • • • • •

Posición 2.

• • • • • • • • • • • • • • • • • • • • • • • • • • • • • • • •

Posición 3.

• • • • • • • • • • • • • • • • • • • • • • • • • • • • • •

Posición 4.

Figura 8. Sucesión de saltos.

Page 99: A Notas de Algebra Moderna

V+v-a-r = V'

A+a-v-r = A'

R+r-v-a = R'

Ecuaciones que, por cualquiera de los métodos usuales se puede comprobar, son

equivalentes al sistema de ecuaciones:

2v = (A+R)-(A'+R')

2a = (V+R)-(V'+R')

2r = (A+V)-(A'+R')

Lo que nos indica que:

A+R y A'+R' son de igual paridad

V+R y V'+R' son de igual paridad

A+V y A'+V' son de igual paridad

Estas condiciones necesarias para que un juego sea soluble pueden resultar algunas veces

suficientes para demostrar que ciertos juegos son imposibles.

Otro sistema de condiciones necesarias parecido a (2) se obtiene al colorear las otras

diagonales. Al combinar los dos sistemas de condiciones se alcanza a determinar un buen

número de juegos imposibles como se puede ver en el siguiente ejemplo:

Ejemplo: Supongamos que el juego propuesto se lleva a cabo en un tablero como el de la

figura 7, en donde la posición inicial contiene todos los cuadros con fichas con excepción

del extremo superior izquierdo. Los valores de V, A y R son V=11, A=12 y R=11. Si se

quiere llegar al final del juego con una única ficha, los valores posibles de V', A' y R' serían

(1,0,0), (0,1,0) o (0,0,1). Pero la primera y las última de estas posibilidades no cumplen las

condiciones (2). Por lo tanto la última ficha debe quedar en uno de los cuadros coloreados

de azul.

Page 100: A Notas de Algebra Moderna

Ahora podemos colorear las otras diagonales, con los colores blanco, marrón y negro, (B,

M y N) como se muestra en la figura 9. Haciendo el análisis correspondiente para este

caso obtenemos que la última ficha debe quedar colocada en un cuadro blanco. Los únicos

cuadros que son azules en la primera coloración y blancos en ésta, están marcados con una

× en la figura 10. Estos son naturalmente los únicos lugares en donde puede en donde

puede quedar la última ficha.

EJERCICIOS

1. Determinar si en los siguientes tableros es posible recorrer todos los cuadros partiendo

de I y llegando a F.

N B M N B

M N B M N

B M N B M

N B M N B

M N B M N

B M N B M

N B M N B

Figura 9.

× ×

× ×

× ×

Figura 10.

I F

I

F

I

F

Page 101: A Notas de Algebra Moderna

2. Para cada uno de los grafos conexos siguientes determinar si se puede o no dibujar sin

levantar la mano y sin repetir linea, en caso afirmativo determinar una ruta.

3. En la demostración de la proposición 1 (que se hace antes de enunciarla) ¿qué resultado

de paridad se utiliza? ¿Dónde?

4. En la ciudad de K niesberg desembocan dos ríos formando dos islas como se muestra en

la figura adjunta, configuración en la que además hay siete puentes. Hace algunos siglos

una diversión dominical consistía en tratar de recorrer los siete puentes sin repetir

ninguno. Corría el rumor de que tal propósito era imposible.

Interpretando el problema como un grafo que se debe dibujar sin levantar la mano y sin

repetir línea (aquí los arcos son los puentes) y utilizando la teoría de esta sección,

demostrar que el problema realmente es imposible. Así lo demostró Euler en 1735.

5. a. En un grafo sin bucles la suma de los órdenes de todos los vértices debe se par. Por

qué?

b. Dividiendo los vértices entre los que tienen orden par y los que tienen orden impar y

aprovechando el resultado anterior, demuestre que en un grafo sin bucles, siempre hay

un número par de vértices de orden impar.

Page 102: A Notas de Algebra Moderna

6. Explique por qué, el conjunto de condiciones (2) conjuntamente con sus equivalentes

para la coloración en sentido contrario, son condiciones necesarias pero no suficientes

para que un juego propuesto de solitario tenga solución.

7. Analizar por el método expuesto si los siguientes juegos son posibles y en caso

afirmativo mostrar una solución salto a salto.

8. Considere un juego planteado como en la posición inicial de la figura 8.

a. Si se quiere terminar con una sola ficha en el tablero, dónde quedar colocada ésta?

b. Además de la posición inicial estudiada, cuáles otras posiciones iniciales conllevan

a la posición final mostrada en la figura adjunta ?

c. Muestre que el juego propuesto con la posición inicial de la figura 7 y con posición

final como se muestra en la figura, es efectivamente realizable. muestra en la figura,

es efectivamente realizable.

• • • • • • • • • • • • • • • • • • • • • • • •

• • • • •

• •

• •

?

?

?

•• • •

Page 103: A Notas de Algebra Moderna

6. TRES PROPIEDADES DE LOS ENTEROS POSITIVOS

En ésta sección expondremos tres principios fundamentales que serán el fundamento de la

teoría expuesta en las secciones siguientes. Estos tres principios algoritmo de la división, el

teorema fundamental de la aritmética y el principio de inducción se darán aquí sin su

demostración, que dejaremos pendiente para cuando tratemos la axiomática de los enteros.

Buscamos mas que todo que el lector se familiarice con la exposición formal de éstos para

que pueda aplicarlos cuando sea necesario. En la sección 4 asegurábamos que todo número

entero es o bien par (de la forma 2k) o bien impar (de la forma 2k+1), este resultado es en

realidad un caso particular de la siguiente proposición que formaliza algo que manejamos

desde niños.

Proposición 1. (Algoritmo de la División) Sea n cualquier entero y m un entero positivo,

entonces existen q y r enteros únicos tales que

i) n=mq+r

ii) 0<r<m

Lo que nos dice la proposición es que cualquier entero (n) se puede dividir por otro mayor

que 0 (el divisor es m) obteniendo de manera unívoca un cociente (q) y un residuo (r) que

debe ser positivo menor que el divisor o ser 0. Por ejemplo, si n=19, m=5, los valores de q

y r serán 3 y 4 respectivamente.

Proposición 2. Todo número elevado al cuadrado o bien es múltiplo de 4, o bien es de la

forma 4k+1.

Demostración. Según el algoritmo de la división los números enteros al dividirlos por 4

pueden tener residuos 0, 1, 2 ó 3. Por ésta razón todo número entero es de la forma 4n,

4n+1, 4n+2 o bien 4n+3.

Page 104: A Notas de Algebra Moderna

Sabemos que los números de la forma 4n son cerrados para la multiplicación, igual sucede

con los de la forma 4n+1, lo cual implica que sus cuadrados son de la misma forma lo que

está de acuerdo con la proposición. Por otra parte si un número tiene la forma 4n+2 su

cuadrado 16n2+16n+4 es múltiplo de 4, mientras que si tiene la forma 4n+3 su cuadrado

16n2+ 24n+ 9 es de la forma 4n+1 (tomando n como...).

Vemos entonces que en todos los casos se obtiene o bien un múltiplo de 4 ó un número de

la forma 4n+1, completando la demostración de la proposicióng.

Para enunciar el siguiente principio supongamos que p1,p2,...,pn, ... es la sucesión ordenada

de números primos. Según esto p1=2, p2=3, p3=5, p4=7, etc. El Teorema Fundamental de

la Aritmética nos garantiza que todo número mayor que 1 es factorizable de manera única,

como un producto de potencias de primos.

Proposición 3. (Teorema Fundamental de la Aritmética /1/) Para todo entero positivo a

existen α1,α2,α3, .....,αn naturales únicos tales que:

a= nnppp ααα ...21

21

siendo p1,p2,...,pn la sucesión ordenada de números primos.

Las célebres descomposiciones en factores primos que se aprenden en la aritmética

elemental son ejemplos de aplicación de éste importante teorema. Por ejemplo, si a=24

decimos que a=23×31, entonces n=2, α1=3 y a=21×72 entonces n=4, α1=1,α2=α3=0 y

α4=2.

La siguiente proposición nos enuncia el Principio de Inducción; sin que podamos decir de

él que se aprende en la escuela primaria, sí podemos asegurar que es un principio muy útil

para hacer demostraciones y que su razón lógica se encuentra en la misma razón de ser de

los números naturales. En efecto, una propiedad básica e intuitivamente clara de los

números naturales es que a cada uno le sigue otro, y a éste, otro, y así sucesivamente,

entonces se agotan todos potencialmente. Este "así sucesivamente" es lo que se formaliza

en el citado principio. Veamos primero un ejemplo:

Page 105: A Notas de Algebra Moderna

Ejemplo 1. En la sección 1 hemos visto como los griegos visualizaban el hecho de que los

primeros n números impares suman exactamente n2. También podemos razonar así:

El primer impar r es 1. Su suma que es 12. El segundo impar es 1×2+1 y la suma del

primero y el segundo es 12+1×2+1 que es (1+1)2 o sea 22. Sabiendo que la suma de los

primeros impares es 22 podemos sumarle a ésta el tercer impar que es 2×2+1 y obtenemos:

22+2×2+1=(2+1)2=9; así obtenemos que la suma de los tres primeros impares es 32.

Supongamos que siguiendo este proceso hemos llegado hasta 36 y sabemos que la suma de

los primeros 36 impares es 362. Como el siguiente impar es 2×36+1, la suma de los

primeros 37 números impares ser: 362+2×36+1=(36+1)2 o sea que se cumple la hipótesis

para 37.

Podríamos decir "...y así sucesivamente..." pero es mejor formalizar aún más. En términos

generales lo que sucede es la siguiente: Si aceptamos que para un n particular la

proposición es cierta, es decir, que la suma de los primeros n impares es n2, sumando a

éstos el (n+1) impar, que es 2n+1 obtenemos que la suma de los primeros n+1 impares ser

n2+2n+1=(n+1)2, o sea que la proposición es cierta para n+1. Tenemos entonces, que si

consideramos cierta la proposición para un número particular automáticamente ser cierta

para su siguiente. Así podremos recorrer todos los naturales desde que aseguremos que la

proposición se cumple para los primeros.

Se acepta entonces que la proposición es cierta para cualquier k, por que potencialmente se

sabe que hay un camino, una cadena que se puede recorrer paso a paso. Esto nos asegura el

Principio de Induccióng.

Proposición 4. (Principio de Inducción) Sea P(n) una proposición para cada natural n tal

que:

i) P(0) es cierta

ii) Siempre que P(k) es cierta se sigue que P(k+1) también lo es.

Page 106: A Notas de Algebra Moderna

Entonces la proposición es cierta para cualquier n natural.

Según esto, para demostrar la proposición P(n) por inducción sobre n, debemos probar

primero que P(0) es cierta (casi siempre es muy fácil) y luego deducir que P(k+1) es cierta

suponiendo que P(k) lo es. La verificación de estas dos condiciones asegura, por el

Principio de Inducción, que P(n) es cierta para todo natural n. Veamos otro ejemplo:

Ejemplo 2. Demostrar por inducción sobre n que 10n-1 siempre es divisible por 9.

Demostración.

i) Es claro que la hipótesis se cumpla cuando n=0. También es fácil comprobar que se

cumple para n=1 y n=2 (aunque no es necesario).

ii) Supongamos que P(k) es cierto, o sea, que 10k-1 es múltiplo de 9 es decir que existe un

entero s tal que 10k-1=9 Debemos deducir que se cumple P(k+1) o sea que 10k+1-1 es

también múltiplo de 9, es decir, debemos encontrar un t entero tal que 9t=10 k+1-1. Pero

10 k+1-1=(10 k-1)10+9=9s×10+9

de donde haciendo t=10s+1 , tenemos:

10 k+1-1=9t

y hemos demostrado que P(k+1) también es cierto. Así por el Principio de Inducción

hemos probado que la proposición propuesta es cierta par todo natural n.

El Principio de Inducción no sirve únicamente para hacer demostraciones. Es muy útil

también para hacer definiciones /1/ que son llamadas Definiciones Recursivas.

Primero aclaremos cuáles son los objetos que se van a definir. Una función f que a cada

natural n le asocia cualquier elemento f(n) de un conjunto B se llama una Sucesión en B.

Por ejemplo la sucesión n1 al natural 1 se le asocia 1, a 2 le asocia 2

1 y a 4 le asocia 41 ,

etc.

Page 107: A Notas de Algebra Moderna

A veces, en lugar de escribir una fórmula se escriben los primeros términos con la

esperanza de encontrar una fórmula observando la relación que hay entre los términos

mostrados. Así en el ejemplo anterior se mostrarían 1, 21 , 3

1 , 41 , ..... y se pueden

suponer que sigue 51 , 6

1 , 71 , .... pero nada nos lo asegura.

Una forma de definir ser entonces mostrar unos primeros términos (con uno basta) y decir

cómo se forman los siguientes, o sea, dar una regla para conocer el n-énesimo término

cuando se conocen los primeros n-1. Esto se hace de manera implícita muchas veces. Por

ejemplo para definir an se dice que a0=1, a1=a, a2=a⋅a, a2=a⋅a⋅a y así sucesivamente se

entiende que cada término se obtiene multiplicando el anterior por a. Es decir, si damos

por conocido el valor de an-1, el término an se formar según la fórmula:

an=an-1⋅a

y esta es la ley recursiva que nos permite hablar de an para cada n que sea entero.

Colorario. (Definiciones Recursivas) Si se da un valor a f(1) y se da una regla para

calcular f(n) en base al valor de f(n-1), entonces queda definida y determinada la función

f(n) para todo entero positivo n.

Ejemplo 2. Generalmente cuando hay puntos sucesivos lo que se quiere indicar se puede

formalizar con definiciones recursivas.

Por ejemplo, si queremos que si sea la suma de las potencias desde 0 hasta i de un número

fijo x, no necesariamente natural, es decir queremos que:

Si=1+x+x2 +..+ xi

podemos definir Si=1+x y Sn= Sn-1+xn. Esta última igualdad nos indica que para formar el

término n-ésimo tomamos el término anterior y le sumamos la potencia correspondiente.

Nótese que aquí hay doble aplicación de las definiciones recursivas: Para encontrar la

potencia siguiente y para agregar a la suma que lleva. Hacemos notar también que se

pueden dar otras definiciones para Si. En efecto, si definimos recursivamente Si=1+x y

S'n-1+1 se puede comprobar que para todo k, Sk=S'k (ejercicio 15).

Page 108: A Notas de Algebra Moderna

Las definiciones recursivas son muy útiles puesto que sirven para formar algoritmos que

calculen la función y además son la base para hacer las demostraciones de sus propiedades.

Por ejemplo el ejercicio 13 e) pide que e demuestre por inducción una fórmula directa para

hallar el valor de Sk.

PREGUNTAS Y EJERCICIOS

1. Por qué del algoritmo de la división se deduce que todo número es par o impar?

2. En cada caso encuentre q y r que hagan cumplir el algoritmo de la división para n y m

dados:

a) n=25 m=8 b) m=25 n=8

c) n=25 m=5 d) m=1 N=25

3. ¿Por qué el algoritmo de la división no es cierto para m=0?

4. ¿Qué problema hay en definir el algoritmo de la división para m=0 sin exigir que m sea

positivo?

5. Demuestre que no hay números cuadrados de la forma 6n+2 ó 6n+5.

6. Demuestre que ningún número de la forma 4n+3 es la suma de dos cuadrados.

7. Sean b y m enteros positivos. Si q es el cociente y r es el residuo cuando el entero a se

divide por b, demostrar que cuando ma se divide entre mb el cociente es q y el residuo

es mr.

8. Si p y q son primos tales que p|q demostrar que p=q.

Page 109: A Notas de Algebra Moderna

9. Sean a, b, c enteros con b>0 y c>0. Si q es el cociente cuando a se divide por b y q' es

el cociente cuando q se divide entre c, demostrar que q' es el cociente cuando a se

divide por bc.

10. Si n es entero positivo y p1, p2,...,pn son primos positivos distintos, demostrar que el

entero (p1p2...pn)+1 no es divisible por ninguno de esos primos.

11. Demuestre que si (a, b, c) es una tripla pitagórica y a, b, c no son todos pares, entonces

dos de ellos, incluyendo c, son impares y el otro es par.

12. Dar una demostración para el recíproco del teorema de Wilson.

13. Demostrar por inducción sobre n:

a) La suma de los primeros n números es ( )121+nn

.

b) La suma de los primeros n números pares es n(n+1).

c) an-1 es divisible por a-1.

d) an-bn es divisible por a-b.

e) 1+r+r2+...+rn=r

r n

−− +

11 1

(r no es 1)

14. Definir n! recursivamente.

15. Se define recursivamente Sk y S'k así:

S0 = 1

Sk = Sk-1+xk

S'k = xS'k-1+1

Demostrar que para todo k entero positivo se tiene: Sk=S'k

16. En base a la definición recursiva de an demostrar por inducción sobre n:

i) anam=an+m

Page 110: A Notas de Algebra Moderna

ii) (an)m=anm

17. Sea nnppp ααα ...21

21 y a= nnppp βββ ...21

21 las descomposiciones en factores primos de a y b

según el Teorema Fundamental de la Aritmética.

a) Demuestre que ab tiene representación: nn

nppp βαβαβα +++ ...221121

b) Demuestre que para que a divida a b es condición necesaria y suficiente que para

todo i de 1 a n se cumpla que αi≤βi.

Page 111: A Notas de Algebra Moderna

7. SUMATORIA Y MULTIPLICATORIA

Las operaciones principales entre números, la suma y el producto, son asociativas y

conmutativas, lo cual significa que cuando hay sucesivas sumas o productos los paréntesis

se pueden eliminar. En otras palabras, es posible sumar todos los elementos de un conjunto

finito de números y también se pueden multiplicar. Así, si A es un conjunto finito de

números

∑∈Ax

x

se lee como "Sumatoria de los x de A" y simboliza la suma de todos los números del

conjunto A y

∏∈Ax

x

se lee como "Productoria de los x de A" o "multiplicatoria" ó simplemente "Producto de los

x de A", simbolizando el producto de todos los números del conjunto A.

Generalmente los elementos de A se enumeran y se notan a1,a2,. . . ,an, entonces la suma y

el producto se notan:

∑=

n

iia

1

∏=

n

iia

1

La manipulación de ésta notación es importante porque nos lleva no sólo a comprobar los

valores de ciertas sumas y productos sino también a descubrirlos. Para ésto basta con

manejar unas propiedades de ésta notación. Nos referiremos al símbolo ä aunque estas

propiedades tienen su correspondencia para ∏ .

Propiedades. Sean L<M y P enteros; {ai} una sucesión de números (puede haber

subíndices negativos), k un número cualesquiera, entonces se cumple:

Page 112: A Notas de Algebra Moderna

1. k∑=

M

Liia =∑

=

M

Liika .

2. ( )∑=

+M

Liii ba =∑

=

M

Liia +∑

=

M

Liib .

3. ∑=

M

Lik =(M-L+1)k (Suma de una constante).

4. ∑=

M

Liia = ∑

+

+=−

PM

PLiPia (Cambio de límites).

5. ( )∑=

−−M

Piii aa 1 =aM-aP-1 (Propiedad telescópica).

6. ∑=

M

Liia =∑

+

=

NL

Liia + ∑

++=

M

NLiia

1.

Demostración. Todas las demostraciones se pueden hacer por inducción (si M=L+q la

inducción se hace sobre q; ejercicio 3) sin embargo se dar n argumentos intuitivos:

1. k∑=

M

Liia =k(aL+aL+1+aL+2+...+aM)= kaL+kaL+1+kaL+2+...+kaM=∑

=

M

Liika .

2. ( )∑=

+M

Liii ba =∑

=

M

Liia +∑

=

M

Liib .

La suma de la izquierda es:

(aL+bL)+(aL+1+bL+1)+...+(aM+bM)

y reagrupando se tiene:

(aL+aL+1+...+aM)+(bL+bL+1+...+bM)

que es la suma de la derecha.

3. En ∑ k hay (M-L+1) sumandos, todos iguales a k, su suma es por lo tanto k(M-L+1)

4. Nótese que éstas son simplemente dos formas de escribir lo mismo; ambas sumatorias

tienen M-L+1 sumandos y ambas empiezan en aL terminando en aM.

5. ( )∑=

−−M

Piii aa 1 = (aP-aP-1)+ (aP+1-aP)+ ...+(aM-1-aM-2)+ (aM-aM-1)

= -aP-1+ (aP-aP)+ ...+( aM-1-aM-1)+ aM

= aM-aP-1

Page 113: A Notas de Algebra Moderna

6. ∑=

M

Liia =(aL+aL+1+...+aL+N)+(aL+N+1+aL+N+2+...+aM)=∑

+

=

NL

Liia + ∑

++=

M

NLiia

1

Como decíamos en un principio, con éstas propiedades podemos hallar los valores de

ciertas sumas, por ejemplo de progresiones aritméticas.

Ejemplo 1. Sabiendo que ∑=

n

ii

1= ( )

21+nn encontrar el valor de 1+4+7+...+(3n-2).

Solución. Se nos pide hallar el valor de ( )∑=

−n

ii

123 tenemos:

( )∑=

−n

ii

123 = ∑

=

n

ii

13 -∑

=

n

i 12 (Por propiedad de linealidad)

= 3∑=

n

ii

1

-∑=

n

i 12

= 3 ( )2

1+nn -2(n-1+1) (Aplicamos 3) y el valor de∑=

n

ii

1

= ( )

213 −nng

Nótese que el resultado puede comprobarse por inducción pero éste método, insistimos,

tiene la ventaja de encontrar el valor buscado; la inducción se efectúa cuando ya se conoce

el resultado o cuando se presiente.

En el ejemplo anterior supusimos conocido el valor de ∑=

n

ii

1; en realidad su valor se puede

encontrar aplicando las propiedades enunciadas para ∑ , como veremos a continuación.

La demostración es importante porque ilustra el método para encontrar ∑=

n

ii

1

2 , ∑=

n

ii

1

3 , etc.

Proposición 1. ∑=

n

ii

1= ( )

21+nn

Demostración. Partimos de la igualdad:

Page 114: A Notas de Algebra Moderna

∑−

=

1

1

2n

ii = ( )∑

=

−n

ii

2

21 (Propiedad 4).

= ∑=

n

ii

2

2 -2∑=

n

ii

2+∑

=

n

i 21 (Por linealidad).

= ⎟⎠

⎞⎜⎝

⎛∑=

n

ii

1

2 -1-2∑=

n

ii

2+(n-1)

(Se suma y se resta 1, se aplica

propiedad 3).

= ∑−

=

1

1

2n

ii +(n2-1)-2 ⎥

⎤⎢⎣

⎡−⎟

⎞⎜⎝

⎛∑=

11

n

ii +(n-1)

(Se “saca” n2 de ∑ 2i y en ∑ i se

introduce 1, que enseguida se resta).

Tomando los dos extremos de estas igualdades vemos que en ambos miembros de éstas

aparece

∑−

=

1

1

2n

ii

término que entonces podemos eliminar para despejar la suma buscada y obtener

2∑=

n

i

i2

=n(n+1)

Lo que nos conduce inmediatamente a la igualdad buscada.g

El lector no debe perderse en los cálculos de la anterior demostración, aunque sí los debe

comprobar, lo importante es que vea cúal es el truco fundamental que se utiliza: Se trata de

desarrollar ∑ 2i como ( )∑ − 21i para luego eliminar ∑ 2i .

Este truco como decíamos, se emplea para encontrar ∑ 2i una vez se conoce ∑ i : Se

desarrolla ∑ 3i como ( )∑ − 31i para luego eliminar ∑ 3i (Ejercicio 6).

Otra suma que nos interesa, es la suma de progresión aritmética, es decir la suma de las

potencias de un número (ejemplo 3 y ejercicio 13 e) de la sección 6). De ella da razón la

siguiente proposición:

Proposición 2. ∑=

n

i

ix0

=1

11

−−+

xx n

Page 115: A Notas de Algebra Moderna

Demostración. Como x es una constante:

x∑=

n

i

ix0

= ∑=

+n

i

ix0

1 (Propiedad 1)

= ∑+

=

1

1

n

i

ix (Propiedad 4)

= ⎟⎠

⎞⎜⎝

⎛∑+

=

1

0

n

i

ix -1 (Suma y resta)

= ∑=

n

i

ix1

+(xn+1-1) (Separando el último termino de la suma)

Se tiene entonces que

x∑=

n

i

ix0

=∑=

n

i

ix1

+(xn+1-1)

y despejando ∑ ix obtenemos el resultado de la proposición.

EJERCICIOS

1. Calcular los valores numéricos de las siguientes expresiones:

a) ∑=

6

1

2

ii b) ( ) 1

4

1

2 1 +

=

−∑ i

ii c) ( )∑

=

+10

312

ii

d) ( )∏=

+6

1

1i

i e) ∑∏= =

3

1 1j

j

i

i f) ∏∑= =

3

1 1j

j

ii

2. Exprese en términos de sumatoria n!

3. Demostrar por inducción las propiedades de ∑ Ud. debe partir de la definición

recursiva de ∑ . ¿Cuál propiedad encierra esta definición?

4. Formular propiedades de ∏ análogas a las enunciadas para∑ .

5. Dado por conocido el valor de ∑ i y utilizando las propiedades de ∑ encontrar:

Page 116: A Notas de Algebra Moderna

a) La suma de los primeros n impares.

b) 2+8+14+...+(6n-4).

c) La suma de los n primeros números de la sucesión 3,8,13,...

6. Deducir una fórmula para :

a) ∑=

n

ii

1

2 b) ∑=

n

ii

1

3 c) ∏=

n

i

i

1

2

7. Deducir una fórmula para hallar el valor de:

a) ∑=

n

i

ix1

b) ( )∑= +

n

i kk1 11

8. Deducir una fórmula para :

a) ∏=

⎟⎟⎠

⎞⎜⎜⎝

⎛−

n

j j2

11 b) ∏=

⎟⎟⎠

⎞⎜⎜⎝

⎛−

n

j j12

11

9. Un cuadro mágico de orden n es un cuadrado dividido en cuadrados en donde se

colocan los números de 1 hasta n2 de tal manera que cada fila o diagonal suman lo

mismo. A continuación un cuadro mágico de orden 3:

8 1 6

3 5 7

4 9 2

En general cuánto vale la suma de los números de cada columna fila o diagonal de un

cuadro mágico de orden n?

10. En la demostraciones la proposiciones 1 y 2 se utiliza varias veces la propiedad 6.

¿Dónde?

Page 117: A Notas de Algebra Moderna

8. EL MAXIMO COMUN DIVISOR.

La relación "n divide a m" tiene sentido cuando n y m son enteros o naturales, pero no para

fraccionarios o reales (por qué?) En la sección 4 vimos la forma de demostrar las

propiedades mas elementales sobre esta relación, propiedades que resumimos a

continuación utilizando la notación "n|m", también introducida en esa sección.

Propiedades de la relación "n divide a m". Siendo a, b, c enteros no nulos se tiene:

1) a|0 y ±1|a

2) a|a

3) Si a|b y b|c entonces a|c.

4) Si a|b y b|a entonces a=±b

5) Si a|b y a|c entonces para cualesquier enteros x, y se tiene que a|(xb+yc)

6) Si a|b entonces |a|≤|b|.

En base a estas propiedades desarrollaremos el concepto de máximo común divisor de dos

enteros a y b (no nulos). En aritmética elemental se conocen algoritmos para encontrar el

máximo común divisor de dos enteros y se entiende que por ejemplo el máximo común

divisor de 9 y 12 es 3, ya que de los divisores positivos comunes de 9 y 12 el mayor es 3.

Nosotros nos basaremos en la siguiente definición:

Definición 1. Dados dos enteros a, b ninguno nulo, Máximo Común Divisor de a y b que

notaremos (a,b), ser el entero positivo c tal que:

i) c|a y c|b.

ii) Si x|a y x|b entonces x|c.

La condición i nos indica que c debe ser un común divisor y a condición ii no señala que es

el máximo. En los ejercicios 4 y 5 se da una necesaria discusión sobre esta definición. La

siguiente proposición nos permite hablar del m.c.d. de tres o mas números.

Page 118: A Notas de Algebra Moderna

Proposición. ((a,b),c)=(a,(b,c))

Demostración. Sean d=(a,b), e=(b,c), f=(a,e) y g=(d,c) debemos demostrar que g=f.

Por ser g=(d,c) entonces g|d y g|c. Por ser d=(a,b) y g|d tenemos que d|a y d|b o sea se tiene

que g divide a a,b y c. Pero si g divide a b y a c entonces g debe dividir a e=(b,c) y como

también divide a a entonces g|f. De manera similar se ve que f|g lo que implica que g=ñf ,

pero como ambos son positivos concluimos que g=f.

Para hallar (n,m) un método muy antiguo, llamado el algoritmo de Euclides, consiste en

hacer divisiones sucesivas, como mostraremos en el siguiente ejemplo para enseguida

formalizar:

Ejemplo 1. Para hallar (32,18) dividimos 32 entre 18 y obtenemos como residuo 14, luego

dividimos 18 entre 14 obteniendo como residuo 4, enseguida dividimos 14 entre 4 y

obtenemos residuo 2, y al dividir 4 entre 2 obtenemos residuo 0. Como 2 es el último

residuo no nulo, 2 es el máximo común divisor de 32 y 18.

Dividendo Divisor Residuo

32 18 14

18 14 4

l4 4 2 (32,18)

Tabla 1. Divisiones sucesivas para encontrar

(32,l8) según el algoritmo euclideano.

Proposicion 2. (Algoritmo Euclideano)

Si a y b son enteros positivos por el algoritmo de la división (Propiedad 6-1 Capítulo 1)

podemos encontrar r1...rk y q1...qk+1 tales que:

Page 119: A Notas de Algebra Moderna

(1) a = bq1+r1 0 < r1< b=r0

b = r1q2+r2 0 < r2<r1

r1 = r2q3+r3 0 < r3<r2

rk-3 = rk-2qk-1+rk-1 0 < rk-1<rk-2

rk-2 = rk-1qk+rk 0 < rk<rk-1

rk-1 = rkqk+1

de esta forma, el último residuo no nulo rk, es el máximo común divisor de a y b.

Demostración. Vamos a proceder por inducción sobre k, que es el número de pasos que

hay en el proceso. Notemos que el proceso se detiene cuando rk+1=0 pues no se puede

hacer la siguiente división.

i) Si k=0 o sea el primer residuo r1 es 0, entonces a es múltiplo de b y por tanto (ejercicio

2) el máximo común divisor es b.

ii) Supongamos que se tiene demostrado cuando hay sólo k-1 residuos, entonces

empecemos el proceso en la segunda ecuación de (1) o sea en b=r1q1+r2. Partiendo de esta

ecuación hasta llegar a la última tenemos k-1 residuos no nulos, entonces por hipótesis de

inducción podemos decir que rk=(b1,r1).

Tenemos:

i) rk|b y rk|r1

ii) x|b y x|r1 ⇒ x|rk

Como r0>r1>...>0 entonces algún rk+1 debe ser cero, esto nos garantiza que el proceso

descrito en (1) es finito.

Pero a=bq1+ r1 entonces rk|a y tenemos que

Page 120: A Notas de Algebra Moderna

i)' rk|a y rk|b.

Ahora bien, si x|a y x|b entonces x|a-bq1 o sea x|r1 y por ii) tenemos que x|rk , por tanto:

ii)' x|a y x|b entonces x|rk.

i)' e ii)' nos garantizan que (a,b)=rk con lo cual queda demostrada la proposición.

Corolario. Si a y b son enteros, los números de la forma αa+βb α,β∈Z se llaman

combinación lineal de a y b. La menor combinación lineal positiva de dos enteros no nulos

es el máximo común divisor.

Ejemplo 2. Para expresar (32,18) como combinación lineal de 32 y 18 podemos recurrir

al algoritmo euclidiano pero en sentido inverso. Según este (tabla 1) tendríamos:

32 = 18×1+l4

l8 = 14×1+ 4

l4 = 4×3+ 4

4 = 2×2

Entonces de la penúltima ecuación tenemos:

2=14-4×3

Pero 4=l8-14×1 entonces 2=l4-(18-14×1)×3=14×4-18×3 y como l4=32-18 entonces

2=(32-18)×4-18×3=32×4+(18)×(-7) y hemos encontrado α=4 y β=-7 tal que 2=(32,18)=

32α+18β. Este proceso es el que utilizamos para la demostración general.

Demostración. Nótese primero que si x es combinación lineal de n y m, y a la vez m es

combinación lineal de n y m', entonces x es combinación lineal de n y m' (ejercicio 7). Por

esta razón y según las ecuaciones de (1) vemos que rk es combinación lineal de rk-1 y rk-2 y a

la vez rk-1 es combinación lineal de rk-2 y rk-3 entonces rk es combinación lineal de rk-2 y rk-3.

Por este proceso "vamos subiendo" hasta llegar a que rk es combinación lineal de r1 y b,

pero como r1 es combinación lineal de a y b vemos que rk, el máximo común divisor, es

combinación lineal de a y b.

Page 121: A Notas de Algebra Moderna

Por otra parte, el máximo común divisor divide a a y divide a b y por tanto a cualquier

combinación lineal de a y b y se deduce que es la menor de todas las combinaciones

lineales positivas de a y b.

Definición. a y b se llaman primos relativos si y sólo si (a,b)=1.

Proposición 3. (Lema de Euclides) Supongamos que a y b son primos relativos y que a|bc

entonces a|c.

Demostración. Como (a,b)=1, según el corolario anterior existen α,β tales que 1=αa+βb

multiplicado por c a ambos lados obtenemos que c=αac+βbc como a|bc y a|αac entonces

a|c.

El siguiente resultado, cuya demostración se deja como ejercicio al lector, establece un

método muy usado para construir el máximo común divisor de dos números: Se

descomponen en factores primos y se escogen aquellos factores comunes con su menor

exponente.

Proposición 4. Si la descomposiciones en factores primos de a y b son:

a= nnppp ααα …21

21

y

b= nnppp βββ …21

21

entonces el máximo común divisor de a y b, (a,b) tiene como descomposición en factores

primos n

nppp γγγ …2121

donde γi es el mínimo entre αi y βi.

Page 122: A Notas de Algebra Moderna

EJERCICIOS

1. Encontrar el máximo común divisor de los siguientes pares de enteros. Expresarlo

como combinación lineal de los dos números:

i) 52, 38 ii) 81, 110 iii) 320, 112 iv) 7469,238

2. Demuestre que (a, ka)=a (con a>0) y que (1, a)=1

3. Sea d=(a,b), demuestre que dba. es un entero múltiplo de a y b.

4. Demuestre que la definición 1 es una buena definición. Es decir, que si dos números c

y c' cumplen la definición se debe tener c=c'.

5. El máximo común divisor de a y b se puede definir como aquel entero c tal que:

i) c|a y c|b.

ii) (x|a y x|b) implica x<c.

Demostrar que esta definición es equivalente a la definición 1 (para esto, suponga que c'

cumple la definición 1 y que c cumple la anterior definición y deduzca que c'=c).

6. Demostrar que (a,b)=(a,b+ka) para todo k.

7. Si x es combinación lineal de n y m, y a la vez m es combinación lineal de n y m'

entonces x es combinación lineal de n y m'.

8. Demostrar que a y b son primos relativos si 1 se puede expresar como combinación

lineal de a y b.

9. Si m es un entero positivo, demostrar que (ma,mb)=m(a,b).

Page 123: A Notas de Algebra Moderna

10. Demostrar que si p es un número primo y a es un entero no nulo entonces o (a,p)=1 o

(a,p)=p.

11. Si p y q son primos distintos entonces (p,q)=1

12. Probar que (a,bc)=1 si y solo si (a,b)=1 y (a,c)=1.

13. Si x=yz+t , probar que (x,z)=(z,t)

14. Si a y b son primos relativos y c pertenece a los enteros positivos entonces:

i) existen α y β tales 1=αa+βb.

ii) (a-b.a+b) es 1 o 2.

iii) Si a|bc entonces a|c.

iv) Si (a|c y b|c) entonces ab|c.

v) (c,ab)=(c,a)(c,b).

15. ¿Cómo es (a2+b2,a+b) sabiendo que (a,b)=1?

16. Pruebe que si a es par y b es impar entonces (a,b)= ⎟⎠⎞

⎜⎝⎛ ba ,

2

17. Probar que si c|ab entonces c|(a,c)(b,c).

18. a) Supóngase que (a,b)=1. Pruebe por inducción que (an,b)=1 (Utilice el resultado del

problema 12).

b) Demuestre que si (a,b)=1 entonces (an,bn)=1.

c) Usando b) demostrar que si a y b son enteros tales que an|bn entonces a|b.

19. Si d=(a,b), a=a'd y b=b'd, demostrar que (a',b')=1.

20. Demostrar la proposición 4 (utilice los resultados del ejercicio 17 de la sección 6).

Page 124: A Notas de Algebra Moderna

21. Demuestre que el corolario de la proposición 2 implica lo siguiente: "Si los múltiplos de

a se marcan en rojo sobre una recta y los múltiplos de b en verde donde a y b son

enteros positivos cuyo máximo común divisor es g, entonces g ser la distancia más

corta de cualquier punto verde a cualquier otro rojo".

22. Si (ab,p)=1 demostrar que sk

k

bpapp++

+

1

1

solo cuando k=s y p|(a+b).

23. Supóngase que ba y

dc son dos fracciones reducidas a su expresión más simple

((a,b)=(c,d)=1). Demostrar que si ba +

dc =

bdcbad + es un entero entonces b=d o b=-d.

24. En base a la proposición 4 demostrar que si dos números a y b son primos relativos y su

producto es un cuadrado, entonces cada uno es un cuadrado perfecto. Deducir esta

misma proposición del resultado establecido en el ejercicio l4.

25. Definir formalmente mínimo común múltiplo. Demostrar que éste se puede obtener

multiplicando los dos números y dividiendo el producto por el máximo común divisor.

Demostrar finalmente que también se puede obtener descomponiendo en factores

primos y formando el producto de todos los primos cada uno con su mayor exponente.

26. Definir recursivamente el máximo común divisor de n números. Definir

recursivamente combinación lineal de n números. Demostrar que el máximo común

divisor de n números es la menor combinación lineal positiva de estos n números.

27. Formalizar la demostración dada para el colorario de esta sección procediendo por

inducción sobre k.

28. a) Demostrar que si b y c son enteros positivos tales que bc es un cuadrado perfecto y

(b,c)=1 entonces ambos b y c son cuadrados perfectos.

Page 125: A Notas de Algebra Moderna

b) En base a la anterior demuestre que no existen enteros a y b tales que a2=2b2 (esto

demuestra que raíz de dos no es racional!).

c) Probar que no existen enteros no nulos a y b tales que a2=3b2.

d) Si n es un entero positivo que no es cuadrado perfecto probar que no existen enteros

no nulos a y b tales a2=nb2.

29. Demostrar el teorema fundamental de la aritmética (sección 6).

Page 126: A Notas de Algebra Moderna

9. ECUACIONES LINEALES DIOFANTINAS

Un problema adivinanza típico es el siguiente: María compra pollos a $50 y patos a $70,

con un costo total de $530. Cuántos pollos y cuántos patos compró. Haciendo x el número

de pollos e y el número de patos tenemos la ecuación

50x+70y=530

que es equivalente a

5x+7y=53. (1)

Es claro que la solución x e y deben ser enteras y positivas, pues no se conciben respuestas

como 43 de ppollos y 5

27 de patos ni tampoco (-3) pollos. Ecuaciones como éstas en

que las soluciones deben ser enteras se denominan ECUACIONES DIOFANTINAS en

honor a Diofantos (S. III D.C.), matemático de la “segunda escuela alejandrina” y que es

considerado pionero del álgebra y la teoría de números. En su aritmética Diofantos da

“recetas” para resolver éstas y otras ecuaciones. Es claro que la teoría de números es el

estudio de ecuaciones diofantinas en gran parte, así pues el “Ultimo Teorema de Fermat”

establece la imposibilidad de resolución de ciertas ecuaciones diofantinas. Por ahora,

vamos a trabajar con algunas ecuaciones lineales diofantinas, como la ecuación (1). Con

los elementos que tenemos sobre máximo común divisor podemos justificar el

procedimiento que se ilustra en el siguiente ejemplo:

Ejemplo 1. Sabemos que (5,7)=1 existe según el corolario de la sección anterior una

solución a la ecuación

5α+7β=1

Sea esta α=3 y β=-2. Podemos entonces conocer una solución entera para la ecuación (1) a

saber:

x0=53α=159 y y0=53β=-106

¿Hay otras soluciones a la ecuación? Supongamos que x, y es otra solución, cómo es?

Tendríamos

Page 127: A Notas de Algebra Moderna

5x+7y=53

5x0+7y0=53

Estando estas dos ecuaciones tenemos:

5(x-x0)=7(y0-y)

Como (5,7)=1 y se tiene 5|7(y0-y).

La proposición de la sección anterior nos permite deducir que 5|(y0-y) es decir que para

algún t entero 5t=y0-y de donde tenemos que y=y0-5t=-l06-5t.

Para encontrar los valores de x reemplazamos el valor de (y0-y) en (2) por 5t y obtenemos:

5(x-x0)=7×5t

de donde x-x0=7t o sea que x=7t+x0=159+7t.

Tenemos entonces que:

x=159+7t

y=-(106+5t)

dando valores a t, obtenemos soluciones para la ecuación (1) así para t=0,1,2,3 se tiene

x=159,166,173, 180 y=-106,-111,-116,-121.

Ya habíamos dicho que nos interesan sólo las soluciones positivas. ¿Cuáles t hacen a x e y

positivos? Según (3) tendríamos:

159 +7y>0 y -106-5t>0

desigualdades que al despejar t nos indican:

t> 77159− y t> 5

106−

o sea que t debe estar entre -22.7 y -21..2 y el único valor entero posible para t ser t=-22

por lo tanto las únicas soluciones positivas son: x=5 , y=4

Este proceso es general y los formalizamos en el siguiente resultado.

Proposición 1. Sean a,b,c enteros no nulos, la ecuación

Page 128: A Notas de Algebra Moderna

ax+by=c (4)

tiene solución si y solo (a,b)|c

Demostración. Esto es una consecuencia del corolario de la sección anterior.

En el ejercicio se pide encontrar la forma general de las soluciones a la ecuación (4) cuando

estas existen. El método utilizado en el ejemplo 1 se puede expandir a ecuaciones con más

de dos variables como veremos enseguida.

Ejemplo 2. Supongamos que queremos encontrar:

5x+7y-10z=12 (5)

como (5,7)=1 por la proposición 1 tenemos que la ecuación

5x+7y=u (6)

siempre tiene solución para cualquier u entero, debemos resolver entonces, reemplazando u

en (5):

u-10z=12

que tiene solución particular u0=22 y z0=1 y por el método del ejemplo anterior vemos que

u=22+10s y z=1+s

entonces la ecuación (6) queda:

5x+7y=22+10s (7)

como para s=0, tenemos u=22, z=1, resolviendo (7) para s=0 obtenemos que x0=3, y0=1,

z0=1 es una solución particular de (5), y de (7) podemos plantear

5(x-2s)+7y=22

que nos dan las soluciones para (5) que estamos buscando:

x-2s=3+7t o sea x=3+7t+2s

y=1-5t

z=1+s

al hacer variar t y s obtenemos todas las soluciones posibles enteras.

La existencia de soluciones para ecuaciones diofantinas de más de dos variables se

establece en el resultado siguiente:

Page 129: A Notas de Algebra Moderna

Proposición 2. La ecuación diofantina

a1x1 +a2x2+...+anxn=c

tiene solución si y sólo si el máximo común divisor de a1, a2,..., an divide a c.

Demostración. Procedemos por inducción para n.

i) Para n=2 la proposición 1 nos garantiza el resultado.

ii) Supongamos que el resultado se tiene para n=k y queremos probarlo para n=k+1. Si

tenemos:

a1x1 +a2x2+...+akxk+ak+1xk+1 =c (8)

Sea d' el máximo común divisor de a1, a2,..., ak; sabemos por hipótesis de inducción que la

ecuación

a1x1 +a2x2+...+akxk=c’

tiene solución única y exclusivamente cuando d'|c', o sea cuando c'=d'x. Ahora por la

proposición 1 la ecuación

a1x1 +a2x2+...+akxk+ak+1xk+1=c

tiene solución si y sólo si (d',ak+1)|c que es lo mismo que exigir que el máximo común

divisor de a1, a2,..., ak+1 divide a c.

PREGUNTAS Y EJERCICIOS

1. Determinar una solución general de la ecuación lineal diofantina

i) 23x+37y =

ii) 2072x+1813y=2849.

2. En el plano señalar los puntos enteros de las rectas 3x-2y=2 y 3x-2y=0.

3. Determinar todas las soluciones de 19x+20y=1909 con x>0 y y>0.

Page 130: A Notas de Algebra Moderna

4. Sean m y n enteros diferentes. ¿Cuántos fraccionarios con denominador n o m hay entre

1 y 0? ¿Cuál es la menor distancia entre dos fracciones de éstas?

5. Encontrar una solución general para la ecuación

1321x+5837y+1926z=2983.

6. Cuando el Señor González en 1911 cambió su cheque por x pesos con y centavos, el

cajero se equivocó y pagó y pesos con x centavos. El Señor González recibió el doble

de la cantidad mas dos centavos. ¿De cuánto era el cheque?

7. ¿Qué tan separados están los puntos enteros de la recta

7x+5y=53

8. Demostrar que cuando (a,b)=1 entonces ab<0 si y sólo si existe un número infinito de

soluciones positivas (x>0,y>0) para la ecuación ax+by=c.

9. Resolver en forma general los siguientes sistemas de ecuaciones para x, y, z enteros.

i) 2x+3y+z=25 ii) 12x+16y-4z=4

4x+6y-2z=12 y+z=3

10. Determinar las condiciones necesarias y suficientes para que las ecuaciones

ax+by+cz=d

a'x+b'y+c'z=d'

tengan soluciones en enteros. Exhibir un método general para encontrar la forma

general de las soluciones.

Page 131: A Notas de Algebra Moderna

10. LA RELACION DE CONGRUENCIA ENTRE ENTEROS.

Con base en los resultados obtenidos en la sección 8 desarrollaremos una notación muy útil

dentro de la teoría de números, notación introducida por Gauss.

Definición 1. Siempre que m|(a-b) diremos que a es congruente con b modulo m y se notar

a≡b(mod m) (sólo se exige que m sea diferente de 0).

Esta notación puede interpretarse como que a y b al dividirse por m tienen el mismo

residuo. En efecto, si a y b tiene el mismo residuo al dividirse por m se tiene:

a=k1m+r y b=k2m+r

que implica (a-b)=(k1-k1)m, o sea que, m|(a-b).

Por otra parte, como 0 es el único múltiplo de m que está entre -m y m si

a≡b(mod m),

aplicando algoritmo de la división tendremos

a=q1+r1; b=q2+r2

con

0<r1<m y 0<r2<m;

por tanto

m|(a-b) y (a-b)=(q1-q2)m+(r1-r2)

se sigue que

m|(r1-r2)

pero r1-r2 debe estar entre –m y m ⎟⎠⎞

⎜⎝⎛ <−<

22 más es 21

mrrm- por tanto, r1-r2=0 o sea los

residuos r1 y r2 deben ser iguales.

Hemos demostrado la siguientes caracterización.

Page 132: A Notas de Algebra Moderna

Proposición 1. a≡b(mod m) si y sólo si a y b tienen el mismo residuo al dividirlos por m.

Ejemplo 1. Según el algoritmo de la división al dividir por 4 se puede obtener un único

residuo entre 0 y 3 y por lo tanto un número debe ser de una única forma: 4n, 4n+1, kn+2 o

4n+3. Esto nos ayuda a demostrar, por ejemplo, que todo número cuadrado es un múltiplo

de 4 o es de la forma 4n+1 (Proposición2 sección 6). Los números de la forma 4n, los

múltiplos de 4, son congruentes entre sí, modulo 4. Los de la forma 4n+1, por ejemplo 41

y l009, son congruentes entre sí todos. Lo mismo sucede con los de la forma 4n+2 y por su

lado con los de la forma 4n+3. Hacer congruencias modulo 4 es pues, formar los números

enteros en 4 “grupos” como se ve en la figura 1.

... -8 -4 0 4 8 ...

... -7 -3 1 5 9 ...

... 6 -2 2 6 10 ...

... -5 -1 3 7 11 ...

Figura1. Los números de cada fila son congruentes entre si modulo 4.

En estos “grupos” que se forman, la relación de congruencia modulo 4 hace el papel de

igualdad. Esto nos garantiza en forma general el siguiente resultado.

Proposición 2. La relación “ser congruente(mod 4)” es una relación de equivalencia en

los enteros, es decir, se cumplen las siguientes leyes:

Reflexiva: Siempre a≡a(mod m).

Simétrica: Si a≡b(mod m) ⇒ b≡a(mod m).

Transitiva: Si a≡b(mod m) y b≡c(mod m) ⇒ a≡c(mod m).

Demostración. A manera de ilustración hacemos la demostración de la simetría. La

reflexiva y transitiva quedan a cargo del lector.

Simetría: Si a≡b(mod m) según la definición 1, m|b-a lo que implica que m|-(b-a) o sea

m|a-b que significa que b≡a(mod m).

Page 133: A Notas de Algebra Moderna

Además de ser la relación de congruencia una relación de equivalencia, tiene otra

característica que la hace supremamente útil: es compatible con la suma y la multiplicación

de enteros. Esto es lo que indica el siguiente resultado.

Proposición 3. Si a≡b(mod m) para cualquier c entero se tiene ac≡bc(mod m) y

a+c≡b+c(mod m).

Demostración. Si a≡b(mod m) por definición m|b-a entonces m|c(b-a) y por lo tanto

m|cb-ca lo que indica que ca≡cb(mod m).

Así mismo, si m|b-a entonces m|(b+c)-(a+c) por tanto a+c≡b+c(mod m).

Ejemplo 3. Sabemos que 10≡1(mod 9) por la proposición anterior vemos que

102≡10(mod 9) y aplicando que la relación de congruencia es simétrica y transitiva vemos

que:

102≡10(mod 9) y 10≡1(mod 9) ⇒ 102≡1(mod 9)

multiplicando por el mismo número 3 vemos que 3×102≡3(mod 9) entonces

3×102+1≡4(mod 9) o sea que 301≡4(mod 9).

Resumidamente se ha visto que como 10≡1(mod 9) entonces(3(10)2+1)≡(3(1)2+1)(mod 9).

En el ejercicio 5 se pide demostrar que si a+c≡b+c(mod m) entonces a≡b(mod m). Esta es

una justificación para la ley cancelativa de la suma en congruencia. Se podría esperar tener

una ley parecida para el producto pero se puede buscar un contraejemplo rápidamente ,así

cuando m=24 1×6≡5×6 y sin embargo no es cierto que módulo 24 1≡5. La siguiente

proposición nos indica cuándo es posible cancelar factores comunes en una congruencia.

Proposición 4. Si (m,c)=1 y ac≡bc(mod m) entonces a≡b(mod m).

Page 134: A Notas de Algebra Moderna

Demostración. Si ac≡bc(mod m) entonces m|(b-a)c, como (m,c)=1 según la última

proposición de la sección 8 concluimos que m|b-a y por lo tanto a≡b(mod m).

Una generalización de este resultado se encuentra en el ejercicio 12.

Definición 2. Un conjunto de números {a0, a1, ..., am-1} es un sistema completo de residuos

módulo m si en él hay uno y sólo un representante de cada residuo al dividir por m. En

otras palabras se deben cumplir dos condiciones:

i) i≠j ⇒ ai no es congruente con aj módulo m.

ii) Para cualquier entero a existe un 0≤i<m tal que: ai≡a(mod m).

La primera condición indica que no hay en {a0, a1, ..., am-1} dos números con el mismo

residuo, la segunda condición asegura que ahí están todos los residuos posibles.

Ejemplo 2. Para buscar un sistema completo de residuos módulo 4, según la figura 1, basta

tomar 4 enteros, cada uno de una fila diferente. Así el conjunto {0,-3,6,11} es un sistema

completo de residuos módulo 4, mientras si tomamos {6,10,5,8} no es un sistema completo

de residuos pues 6≡10(mod 4) y además no hay ninguno que tenga residuo 3.

Fijemos nuestra atención en el s.c.r. {8,-3,6,11} teniendo en cuenta las proposiciones 2 y 3

vemos que:

8+(-3)≡(-3) y 6+11≡(-3) y 6+(-3)≡11, etc. y así con el producto (-3)×11≡6 y (-3) ×11≡11

y 6×11≡6, etc. Podemos resumir esto haciendo tablas de multiplicar y sumar tendremos:

Tablas 1.

En este sistema completo de residuos el 8, por ejemplo, representa todos los números que

tienen el mismo residuo que él al ser dividido por 4: todos los múltiplos de 4; -3 representa

los números de la forma 4n+1; el 6 los de la forma 4n+2 y 11 a los de la forma 4n+3. Un

+ 8 -3 6 118 8 -3 6 11-3 -3 6 11 8 6 6 11 8 -3

11 11 8 -3 6

⋅ 8 -3 6 11 8 8 8 8 8 -3 8 -3 8 6 6 8 6 8 6

11 8 11 6 -3

Page 135: A Notas de Algebra Moderna

sistema canónico de residuos equivalente al anterior sería {0,1,2,3} en donde las tablas nos

quedan:

Tablas 2.

Nótese que aquí 3×3≡1 indica que dos números de la forma 4n+3 multiplicados nos da uno

de la forma 4n+1.

Definición 3. Cuando hablemos de la aritmética modulo m nos referiremos a las

operaciones entre los números 0,1,2,..,(m-1) según la relación de congruencia (mod m).

Los cálculos en la aritmética módulo m se hacen como en los números en cuanto se

cumplen propiedades como la distributiva, las dos operaciones sin conmutativa y

modulativa etc. Sin embargo hay una diferencia importante: la ley cancelativa para el

producto es más restringida en la aritmética módulo m según la proposición 4. Por otra

parte cuando el módulo es primo podemos hablar de inversos multiplicativos lo cual no

sucede en los enteros, donde los únicos que tienen inversos multiplicativos son..... Estas

propiedades básicas son formalizadas en la siguiente afirmación.

Proposición 5. (Propiedad de la aritmética mod. m).

i) Ley cancelativa para la suma:

a+x≡a+y ⇒ x≡y.

ii) Para todo a y b existe un único x tal que:

a+x≡b.

iii) m es primo para todo a no congruente con 0 y todo b existe un único x tal que:

a⋅x≡b.

iv) Si m es primo para todo a no congruente con 0, todo b y c existe un único x tal que:

ax+b ≡ c.

+ 0 1 2 3 0 0 1 2 3 1 1 2 3 0 2 2 3 0 1 3 3 0 1 2

⋅ 0 1 2 3 0 0 0 0 0 1 0 1 2 3 2 0 2 0 2 3 0 3 2 1

Page 136: A Notas de Algebra Moderna

Demostración.

i) Si a+x≡a+y(mod m) entonces m|(a+x)-(a+y) lo que implica que m|x-y o sea que

x≡y(mod m).

ii) Vemos primero que para todo a existe (-a) tal que a+(-a)=0.

Hágase simplemente (-a)=m-a cuando a≠0 y (-0)=0. Para resolver la ecuación

a+x≡b(mod m) tómese x≡b+(-a)(mod m) y se tendrá:

a+x≡a+( b+(-a)) ≡b.

iii) Consideremos los residuos 0,a,2a,...,(m-1)a . Entre estos residuos no pueden existir

dos repetidos pues si ia≡ja como m es primo, (m,a)=1 y podemos aplicar la proposición 4

obteniendo i≡j o sea i=j. Esta consideración nos garantiza que entre 0,a,2a,...,(m-1)a a no

hay dos residuos iguales y por lo tanto 0,a,2a,...,(m-1)a es un sistema completo de residuos

modulo m entre los cuales debe estar la clase residual de b, por tanto existe un x, único

como residuo, tal que ax≡b(mod m).

La parte cuatro de la demostración se deja como ejercicio al lector.

La demostración de la parte 3, como ya se indicó, es básica y sutil. Su argumento lo

resaltamos en la siguiente proposición que ser utilizada mas adelante.

Proposición 6. Si a no es congruente con 0 módulo m cuando m es primo entonces el

conjunto 0,a,2a,...,(m-1)a es un sistema completo de residuos.

Como consecuencias de la proposición 6 encontramos la parte iii) de la proposición 5, así

como el Teorema débil de Fermat y el Teorema de Wilson, con los cuales cerramos esta

sección.

Proposición 7. (Teorema débil de Fermat) Si p no es primo y a no es múltiplo de p,

entonces:

ap-1≡1(mod p)

Page 137: A Notas de Algebra Moderna

Demostración. Según la proposición 6 los residuos 0,1,2,3,....,(m-1) son exactamente los

residuos de a,2a,3a,...,(m-1)a; salvo el orden. Por esta razón tenemos:

1×2×...× (p-1)≡1×a×2a×...×(p-1)a

lo cual indica que:

(p-1)!≡(p-1)!ap-1

y como (p-1)! no es múltiplo de p existe según la proposición 5 iii) existe un único x tal

que:

(p-1)!x≡(p-1)!(mod p).

por tanto,

ap-1≡1(mod p).

Proposición 8. (Teorema de Wilson) Si p es primo entonces:

(p-1)! ≡-1(mod p)

Demostración. Sabemos que en 0,1,2,...,(p-1) están todos los residuos módulo p y además

que todo residuo no nulo a tiene su inverso multiplicativo a-1 (ejercicio 22). Cuáles

residuos entre 1 y p-1 tienen residuo igual a si mismo, es decir, para qué x se cumple

xx≡1(mod p)? Claramente para x≡1 y x≡-1 se tiene.

¿Hay otros? Si p divide a x2-1, p debe dividir a (x-1)(x+1) o sea:

(x-1)(x+1)≡0(mod p)

pero esto sólo es posible cuando o bien x-1≡0(mod p) o bien x+1≡0(mod p) (véase ejercicio

15). Esto nos asegura que los únicos residuos que elevados al cuadrado son congruentes

con 1 son 1 y -1. O sea que cada uno tiene su inverso multiplicativo diferente salvo el 1 y

-1 (o sea m-1).

Ahora bien, como p es impar hay p-1 residuos no nulos de los cuales p-3 (salvo el 1 y -1)

tienen su inverso diferente, por tanto al multiplicar 2,3,...,(p-2) tenemos un número par de

residuos que se agrupan 2 a dos anulándose todos, por lo tanto

2×3×...×(p-2)≡1(mod p)

y tenemos que

(p-1)!≡-1(mod p).

Page 138: A Notas de Algebra Moderna

Para aclarar un poco el proceso seguido en estas últimas demostraciones analicemos un

caso concreto.

Ejemplo 3. Sea p=7 y a=4, según la aritmética módulo p (tabla 3) los elementos

0,4,2×4,3×4,4×4,5×4 y 6×4 (la fila 5 de la tabla del producto) es un sistema completo de

residuos (proposición 6) y por tanto

(4×1)×(4×2)×(4×3)×...×(4×6)≡1×2x3×...×6(mod 7)

y se tiene

46×6!≡6!(mod 7)

lo que implica que

46≡1(mod 7)

como lo asegura el Teorema débil de Fermat.

Por otro lado, según la tabla 2×4≡1 y 3×5≡1, por tanto:

6!=2×3×4×5×6≡6≡-1(mod 7)

que es el teorema de Wilson.

Tabla 3. Suma y producto módulo 7.

EJERCICIOS.

1. Demostrar que la relación de congruencia es reflexiva y transitiva.

2. Demostrar que si a≡b(mod m) y c≡d(mod m) entonces,

a+c≡b+d(mod m)

+ 0 1 2 3 4 5 6 1 0 1 2 3 4 5 6 1 1 2 3 4 5 6 0 2 2 3 4 5 6 0 1 3 3 4 5 6 0 1 2 4 4 5 6 0 1 2 3 5 5 6 0 1 2 3 4 6 6 0 1 2 3 4 5

× 0 1 2 3 4 5 6 0 0 0 0 0 0 0 0 1 0 1 2 3 4 5 6 2 0 2 4 6 1 3 5 3 0 3 6 2 5 1 4 4 0 4 1 5 2 6 3 5 0 5 3 1 6 4 2 6 0 6 5 4 3 2 1

Page 139: A Notas de Algebra Moderna

3. Hacer las tablas de adición y multiplicación módulo 11 y 12 y encontrar todos los

residuos x que en cada caso cumplan la ecuación dada:

a. 3x≡6(mod 11) b. 3x≡6(mod 12)

c. 3x≡7(mod 11) d. 3x≡7(mod 12)

e. x2≡1(mod 11) f. x2≡8(mod 12)

g. x2≡3(mod 11)

4. Qué horas indica el reloj si:

a. 29 horas antes indicaba las 11.

b. 100 hora antes eran las 2.

c. 50 horas después ser n las 6.

5. Determine la forma de todos los enteros que cumplen a la vez cada par de congruencias:

a. x≡3(mod 7) y x≡4(mod 9)

b. x≡5(mod 6) y x≡8(mod 12)

6. Explicar en términos de congruencias (módulo 4):

a. El doble de un impar sumado con un múltiplo de 4 es un número de la forma 4n+2.

b. Un número no primo de la forma 4n+3 tiene al menos un divisor diferente de él, de

la forma 4n+3.

c. Lo anterior no es cierto si cambio 4n+3 por 4n+1.

7. ¿Qué se puede concluir de que a2≡b2(mod p) cuando p es primo?

8. En la aritmética módulo m se puede hablar de algoritmo de la división?

9. Encontrar todas las triplas (x, y, z) modulo 5 tales que

x2+y2=z2

10. Demostrar que si a+b≡c+b(mod m) entonces a≡c(mod m).

Page 140: A Notas de Algebra Moderna

11. Demostrar que si n es entero positivo impar entonces

1+2+3+... +(n-1) ≡0(mod n)

12. Sea p(x) un polinomio con coeficientes enteros. Demostrar que x≡y(mod m) implica

que f(x)≡f(y)(mod m).

13. Sea (m,c)=d y m=dn; si ac≡bc(mod m) entonces a≡b(mod n).

14. Demostrar que si p es primo xp+yp≡(x+y)p.

15. Demostrar que si p es primo ab≡0(mod p) implica a≡0(mod p) o b≡0(mod p). ¿Qué se

puede decir si p no es primo?

16. Probar que cuando p es primo impar xp+yp-≡0(mod p) implica xp+yp≡0(mod p2).

17. Siendo p primo ap≡a(mod m).

18. A qué congruencia de grado inferior a 7 es equivalente la congruencia:

2x17+6x16+x14+5x12+3x11+2x10+x9+5x8+2x7+3x5+4x4+6x3+4x2+x+2≡0(mod 7)?

19. Probar el teorema débil de Fermat demostrando que

(1+1 +...+1)p=(1+1+...+1)

siempre que el número de 1's se menor que p.

20. Si a0, a1,..., am-1 es un sistema residual completo módulo m, entonces ka0, ka1,..., kam-1

también lo es. Demostrar que esto se tiene si k es primo relativo con m.

21. Deducir un resultado similar al anterior para los enteros ka0+1, ka1+1,..., kam-1+1,

22. Demostrar que si (a,m)=1 entonces:

Page 141: A Notas de Algebra Moderna

i) Existe a-1 tal que aa-1≡1(mod m).

ii) Si ax≡0(mod m) entonces x≡0(mod m).

23. Demostrar que cuando p es primo, si a0, a1,..., an no son múltiplos de p entonces

a0a1...an no es múltiplo de p.

Page 142: A Notas de Algebra Moderna

12. NUMERACIONES POSICIONALES.

Los sistemas de numeración utilizados por las antiguas civilizaciones: babilonia y maya son

cada uno sistemas posicionales, el babilonio en base 60 y el maya en base 20, que utilizan

para representar los números desde 1 hasta la base de un sistema aditivo, y para números

mayores que la base por repetición de los primeros, en donde cada uno de éstos vale según

la posición que ocupe. Además, como ya lo anotamos, los sistemas posiciones consideran

de una u otra forma, lo que nosotros hoy en día llamamos el cero. Por ejemplo, en la

numeración maya los números de uno a veinte (sin incluirlo) son notados por un sistema

aditivo en donde las unidades son demarcadas con puntos o círculos llenos y cinco (5)

unidades de demarcan por una raya horizontal (fig. 1). Los mayores que 19, desde 20 en

adelante se representan por combinaciones de números menores que éste y el cero que es

una figura que se ha interpretado como una concha o un fríjol.

Estos dos sistemas (el maya y el babilonio) utilizan pocos caracteres diferentes, lo que hace

que para números grandes sea necesario muchos de éstos. Esta es una desventaja que les

quita practicidad a pesar de que son muy efectivos, como todo sistema posicional, en

cuanto a la realización de la operaciones básicas.

El otro sistema posicional es el sistema decimal hindoarábigo que hoy en día representa el

lenguaje más universal que el hombre ha adquirido. Esta numeración, también de origen

60 63 215 871

3 12 45

Figura 1. Algunos numerales mayas.

Page 143: A Notas de Algebra Moderna

muy remoto se calcula nació en la India hace unos 2. 500 años. En los manuscritos

budistas de Asaka del siglo III a. C. hallamos los símbolos 1,4,6; cien años después

hallamos los símbolos 2,4,5,6,7 y 9 grabados en los monumentos de Nona Ghat y en el

siglo II D. C. , periodo de las cuevas de Nasik, hallamos todas las cifras. El cero empezó

entre los indostánicos como un puntico o circulito y el sánscrito lo designaba con la

expresión "vacante" o "vacío".

La primera referencia concreta a la numeración indostánica fuera de las fronteras de la

India se halla en la nota escrita por un sacerdote mesopotámico, Severus Sebokth, hacia 650

D. C. quien hablaba de nueve signos sin mencionar el cero. A fines del siglo VIII habían

sido trasladadas a Bagdad unas tablas astronómicas, y los rabees doctos de la época

conocieron estos signos. En el año 925 el estudioso Al-Khwarizmi escribió un librito sobre

números el cual 300 años después fu‚ traducido en latín por Abelardo de Bath. El

desarrollo del comercio, del sistema de intereses y en general la necesidad de efectuar las

operaciones básicas de manera más rápida hace que este sistema se imponga

definitivamente con el correr de los siglos. Muy ilustrativo es el cuadro de la figura 2

tomado de un libro muy popular en el siglo XVI (impreso por primera vez en 1503) y que

muestra la disputa entre un abaquista (numeración romana) y un calculista de pluma y tinta

(sistema induarábigo). Repárese en la angustia del abaquista contra la seguridad del

calculista al tratar ambos de dividir 1234 entre 97.

Nuestro interés girar en lo que sigue hacia la profundización en los sistemas de numeración

posicional en diferentes bases con miras a explicar el por qué‚ de los algoritmos que

utilizamos en las operaciones básicas y ciertos hechos que se pueden observar en la

aritmética.

Por ahora pensemos en la siguiente máquina de contar: consideremos tres relojes cada uno

con los números del cero al nueve (fig. 3), que hallaremos Dial I, Dial II y Dial III. Este

sistema funciona como los contadores del kilometraje de los automóviles, si comenzamos a

dar vueltas al dial I en el sentido de las mancillas del reloj y contamos 1,2,. . . . , etc. de

tal manera que cuando vaya a pasar de nueve al cero, el dial II se corre un lugar y éste a su

Page 144: A Notas de Algebra Moderna

vez avanza un lugar al dial III, cuando pase del nueve al cero entonces el dial I determinar

las unidades, cada avance del dial II supone diez unidades que ha recorrido el dial I.

Así mismo el dial III se mueve un lugar cuando el dial II complete una vuelta de diez

movimientos o sea cuando el dial I se haya movido cien veces.

Así en la figura 3 el número representado es 520, 500 por el dial III, 20 por el dial II y cero

por el dial I. Nótese que podemos agregar otros diales si queremos expresar números más

grandes, por ejemplo el sexto dial indicaría "cien miles" mientras un último dial marcaría

los millones.

Cada dial se referirá a diferentes potencias Fig. 3: Diales para presentar números en el

sistema posicionales de 10. Por esto un hindoarábigo.

Figura 2. Grabado del "Margarita Philosofica" de Greisch (1. 503).

Page 145: A Notas de Algebra Moderna

Nuestro sistema de numeración se dice que es en base 10, pero que se trabaje con 10 dígitos

(0,1,2,. . . ,9) es algo arbitrario (tal vez porque tenemos 10 dedos en las manos) pues se

hubiera podido utilizar 3, 5, ó 12 dígitos (señales en los diales).

Supongamos entonces que los diales no tienen sino tres señales: 0,1,2. En la figura 4 se

muestran como funcionarían los tres diales en los primeros 11 movimientos.

Los primeros tres números cero, uno y dos se representan igual que en nuestro sistema,

pero en el tercer movimiento el dial! pasa a 0 y el dial II avanza un lugar quedando en 1,

por eso el número que nosotros notamos 3 será notado 0l0.

El dial II indicar entonces los múltiplos de 3. Así mismo podemos observar que el dial III

representar los múltiplos de 9. Entonces el número escrito en base 3 como 211 equivale al

número que en base 10 escribíamos como 22, ya que se tendrían 2 nueves, 1 tres y una

unidad. Esta igualdad se expresar así: (211)3=(22)10

Aquí también podemos agregar cuantos diales queramos, y naturalmente en base 3,

necesitaremos más dígitos que en base 10 para expresar un numero determinado. Por

ejemplo:

(21102)3=2×34+1×33+1×32+0×3+2=(200)10.

Así como hemos hecho con tres, se puede utilizar cualquier base mayor que 1 para

representar un entero positivo dado. Cuando se quiere utilizar una base mayor que la

nuestra es necesario introducir nuevos dígitos. Por ejemplo, cuando se quiere trabajar en

0 9 1

8 2 7 3

6 4 5

0 9 1

8 2 7 3

6 4 5

0 9 1

8 2 7 3 6 4

5

Figura 3. Diales para presentar números en el sistema posicional indoarábigo.

Page 146: A Notas de Algebra Moderna

base 12 podemos usar como dígitos: 1,2,...,9,A,B en donde A y B representan el 10 y el 11

de nuestra base decimal. Así: (3A)12=3×12+a=36+10=(46)10. Además.

0

2 1 0

2 1 0

2 1

0

2 1 0

2 1 0

2 1

0

2 1 0

2 1 0

2 1

0

2 1 0

2 1 0

2 1

0

2 1 0

2 1 0

2 1

0

2 1 0

2 1 0

2 1

0

2 1 0

2 1 0

2 1

0

2 1 0

2 1 0

2 1

0

2 1 0

2 1 0

2 1

0

2 1 0

2 1 0

2 1

0

2 1 0

2 1 0

2 1

0

2 1 0

2 1 0

2 1

Page 147: A Notas de Algebra Moderna

Es de particular importancia resaltar que el sistema en base 2 también llamado sistema

binario y que utiliza los símbolos 0 y 1, tiene en nuestra ‚poca gran figuración pues es

utilizado por las modernas computadoras y por tanto más adelante profundizaremos sobre

este tema.

A continuación formalizaremos el resultado básico en que se basa toda la discusión, es

decir, que cualquier numero entero positivo se puede expresar en cualquier base mayor que

uno.

Proposición 1. (Teorema fundamental de la numeración) Sea b un entero mayor que 1,

que llamaremos base. Entonces para todo a entero positivo existen an,an-1,..,a1,a0 con cada

ai que cumple 0<a i<b y tales que:

a= a0+a1b+a2b2+...+anbn

Siendo así se nota a=(an an-1 ... a1 a0)b

Demostración. Dividiendo a entre b obtenemos un cociente q0 y un residuo a0 menor que

b y mayor o igual que cero. Si q0 es cero el número a sería expresado con un sólo dígito,

caso contrario dividimos q0 entre b obteniendo como residuo a1 y así sucesivamente

obtenemos:

a = bq0+a0 con 0≤a0<b q0 = bq1+a1 con 0≤a1<b q1 = bq2+a2 con 0≤a2<b

qn-2 = bqn-1+an-1 con 0≤an-1<b qn-1 = bqn+an con 0≤an<b

donde el último cociente qn es cero. Devolviéndonos vemos que:

an = qn-1ban+an-1 = qn-2

b(ban+an-1)+an-2 = qn-3

b(...b(ban+an-1)+...)+ a3)+a2 = q1b(...b(ban+an-1)+...)+a2)+a1 = q0

bnan+bn-1an-1+...+ba1+a0 = a

Page 148: A Notas de Algebra Moderna

Comprobando que los ai cumplen lo exigido, la demostración de que estos números enteros

a0,a1,...,an (llamados dígitos de a) son únicos, la dejamos como ejercicio al lector.

Ejemplo 1. Para pasar un número escrito en una base diferente de 10 a base decimal

tenemos en cuenta la expresión (1) de la proposición anterior así:

(100)3 = 1×32+0×3+0=(9)10

(222)4 = 2×42+2×4+2=(42)

(222)3 = 2×32+2×3+2=(26)

Ejemplo 2. Cuando se utiliza base mayor que la base nuestra, se acostumbra las letras

A,B. C... para expresar lo que nosotros escribimos como 10, 11, 12,... por ejemplo en base

16:

(ACE)16=10×162+6×16+14=(2670)10

pues (A)16=(10)10 y (E)16=(14)10

Ejemplo 3. Para pasar de base decimal a otra diferente aplicamos el algoritmo descrito

en la demostración de la proposición anterior según el conjunto de ecuaciones (2). De esta

forma para determinar el desarrollo decimal de (26)3 dividimos sucesivamente así:

26 3 2 28 3

2 2 3 2 0

dividimos que significan que:

26=3×8+2

8=3×2+2

2=3×0+2

O sea que:

26=3×(3×2+2)+2

es decir:

26=32×2+3×2+2

y tenemos que:

(26)10=(222)3

Page 149: A Notas de Algebra Moderna

Ejemplo 4. Para pasar de base 10 a base mayor que 10 procedemos de igual forma

teniendo en cuenta el significado de las letras A,B,C,... según se explicó en el ejemplo 3.

(1425)10=(?)14

Haciendo las divisiones vemos que:

1425 = 14×101+11

101 = 14×7+3

entonces

1425 = 14(14×7+3)+11

1425 = 142×7+14×3+11

1425 = (73B)14

PREGUNTAS Y EJERCICIOS

1. Enumere las desventajas del sistema de numeración como el maya y el babilonio.

2. ¿Cuántos caracteres se requieren para representar un número en base b?

3. Resuelva:

a. (342)5=( ? )6

b. (241)6=( ? )5

c. (144)10=( ? )12

d. (ABC)16=( ? )10

e. (871)9=( ? )14

f. (6D2)16=( ? )11

4. Para pasar de base 3 a base 9=32 podemos proceder por el siguiente m‚todo ilustrado

para el caso en que nos preguntan (120110)3 =(?)9 tomamos el número de base 3 y

separamos cifras de derecha a izquierda de dos en dos:

Page 150: A Notas de Algebra Moderna

12-01-10

cada pareja la traducimos a base 9 y en su orden estas cifras serán los dígitos del

número en la nueva base:

(12)3=(5)9; (01)3=(1)9; (10)3=(3)9

O sea: (120110)3=(513)9

a. Compruebe con este y otros casos que éste efectivamente es un m‚todo acertado

para pasar de base 3 a base 9.

b. Según esto explique cómo se hace el proceso inverso: pasar números de base 9 a

base 3.

c. Busque un método parecido para pasar numerales de base 2 a base 4 y

numerales de base 2 a base 8.

d. Generalice y demuestre un método para pasar de un número de base b a base bn

y al contrario.

5. Un tendero sólo tiene una pesa de 1 kilo, otra de 2, otra de 4, una de 8, otra de 16 y una

de 32.

a. ¿Cómo hace para pesar 28 kilos, 39?

b. Demuestre que puede pasar cualquier cantidad de kilos de 1 a 63.

6. El siguiente juego puede servir para descretar a sus amigos: se tiene cuatro cartas en

donde están repetidos los números de 1 a 15 así:

I II III IV

1-3-5 2-3-6 4-5 8-9 7-9 7-10 6-7 10-11

11-13 11-14 12-13 12-13 15 15 14-15 14-15

Usted pide a alguien que pida un número secretamente (por ejemplo 11) y sólo le

comunica en qué‚ tarjeta esta escrito (para el caso I, II y IV) y el único que esté escrito

en exactamente esas tarjetas es el 11.

Page 151: A Notas de Algebra Moderna

a. Describa cómo se determina el número pensado conociendo las tarjetas

correspondientes.

b. ¿Por qué‚ esta distribución funciona?

c. Haga un juego parecido con cinco tarjetas y los números de 1 a 32.

Page 152: A Notas de Algebra Moderna

13. CRITERIOS DE DIVISIBILIDAD Y CONGRUENCIAS

Cuando tenemos un número muy grande escrito en base 10 y deseamos saber si es múltiplo

por ejemplo de 9 no necesitamos hacer la división, simplemente sumamos sus cifras y si el

resultado es múltiplo de 9 el número original es múltiplo de 9. Este es un típico criterio de

divisibilidad, que se utiliza desde la escuela primaria. Uno más sencillo es para saber si un

número es par: se mira la última cifra y si ella es par todo el número es par.

La justificación de estos criterios radica en el sistema de numeración que se utiliza y la

demostración de su validez, que haremos aquí as ndonos el la teoría de congruencias, nos

proporciona elementos para formular nuevos criterios de divisibilidad. El siguiente lema,

que es consecuencia inmediata de la definición de congruencia, es la razón por la que

utilizaremos esta teoría para lograr nuestros objetivos.

Lema 1. Un número k es divisible por c si y solo si

k≡0(mod c)

Demostración. (obvio).

Para la lectura de este capítulo además de manejar las congruencias el lector debe estar

familiarizado con los sistemas de numeración posicionales (sección 12). Se aplica

especialmente el Teorema fundamental de la numeración.

1. CRITERIOS DE LA ULTIMA CIFRA

Algunas veces para ver si un número es divisible por otro es suficiente con observar la

última cifra. Esto depende realmente de que el divisor divida la base, como cuando se

trabaja en base 10 y se quiere saber si un entero es divisible por 2 o por 5.

Page 153: A Notas de Algebra Moderna

Proposición 1. Sea a=(anan-1...a1a0)b si b≡0(mod c) entonces a es divisible por c si y solo

si

a0≡0(mod c)

Demostración. Sabemos que

a=a0+a1b+a2b2 + ...+ anbn

y como b≡0(mod c), aplicando aritmética de congruencias se obtiene que a≡a0(mod c), que

en combinación con el lema 1 nos proporciona el resultado deseado.

Nótese que lo que se demuestra es que, en este caso, la última cifra (a0) determina la clase

de congruencia módulo c, a la que pertenece el entero a.

Ejemplo 1. Un número escrito en base 12 es divisible por 4 si termina en 0,4 u 8, pues

estos son los tres dígitos de la base 12 que se dejan dividir por 4.

Ejemplo 2. Como ya se dijo una aplicación de al proposición 1 en base decimal son los

conocidos criterios para saber cuando un entero es divisible por 2 o por 5. Sin embargo

estos criterios no sirven cuando el número est escrito en cualquier base. Para base 15 el

criterio del 2 no es válido aunque el del 5 casi. Por qué?

2. CRITERIOS DE LA SUMA DE LAS CIFRAS

Otras veces la suma de las cifras indica si se es o no divisible por otro. Es el caso de los

conocidos criterios para saber si un número es divisible por 3 o por 9, cuando est escrito en

base decimal. La siguiente proposición justifica‚ estos y otros casos.

Page 154: A Notas de Algebra Moderna

Proposición 2. Sea a=(anan-1...a1a0)b si b≡1(mod c) entonces a es divisible por c si y solo

si

a0 + a1 + a2 + ... + an ≡0(mod c)

Demostración. Sabemos que

a=a0 + a1b + a2b2 + ... + anbn

y como b≡1(mod c), aplicando aritmética de congruencias se obtiene que

a0 ≡ 0(mod c) a1b ≡ a1(mod c)

a2b2 ≡ a2(mod c) anbn ≡ an(mod c)

entonces sumando estas congruencias tenemos

a≡a0+a1+a2+...+an(mod c),

que en combinación con el lema 1 nos proporciona el resultado deseado.

Presentamos las aplicaciones de esta proposición como corolarios:

Corolario 1. Si b es congruente con 1 módulo c y el número a este escrito en base b,

entonces a es congruente con la suma de sus cifras módulo c.

Corolario 2. Si b es congruente con 1 módulo c, entonces para saber si un número (escrito

en base b) es divisible por c es suficiente saber si la suma de sus cifras lo es.

Ejemplo 3. (Base decimal) La suma de las cifras del entero 19168639 es 43 por tanto:

19168639≡1(mod 3)

19168639≡7(mod 9)

Ejemplo 4. En base 4, la suma de las cifras del entero a es congruente con a módulo 3

pero no módulo 9.

Page 155: A Notas de Algebra Moderna

3. CRITERIOS DE LA SUMA Y RESTA DE LAS CIFRAS

Empecemos mostrando un ejemplo:

Ejemplo 5. Para saber si un número escrito en base decimal es divisible por 11, se halla

la diferencia entre la suma de cifras de lugares pares y la suma de las cifras de lugares

impares; el número es múltiplo de 11 , si y solo si, esta diferencia lo es.

Digamos, para saber si 19168639 es divisible por 11, sumamos las cifras de lugares

impares: 9+6+6+9=30 ; sumamos las cifras de lugares pares: 3+8+1 +1=13; hacemos la

diferencia de estas dos sumas: 30-13=17. Como 17 no es múltiplo de 11 entonces

19168639 no es múltiplo de 11. Es mas, como se ve en la demostración de la siguiente

proposición se tiene que

17≡19168639(mod 11).

Nótese que si a=(an, an-1,...,a1, a0)b la diferencia entre la suma de cifras de lugares pares y la

suma de las cifras de lugares impares viene dada por la expresión:

a0-a1+a2-...+(-1)nan

Proposición 3. Sea a=(an, an-1,...,a1, a0)b si b≡-1(mod c) entonces a es divisible por c si y

solo si

a0-a1+a2-...+(-1)nan≡0(mod c)

Demostración. Ejercicio (es similar a la demostración de la proposición 2).

Corolario 1. Si b es congruente con -1 módulo c, entonces para saber si un número

(escrito en base b) es divisible por c es suficiente saber si la suma de cifras de lugares pares

menos la suma de las cifras de lugares impares es múltiplo de c.

Page 156: A Notas de Algebra Moderna

4. CRITERIOS CON PAREJAS Y TRIPLAS DE CIFRAS

Algunas veces es conveniente considerar las cifras de un número tomadas de dos en dos, o

de tres en tres (siempre de derecha a izquierda). Aquí realmente, se está pasando el número

a base b2 o b3, como se resalta en el siguiente lema cuyo enunciado y demostración se dejó

como ejercicio en sección anterior.

Lema 2. Las cifras de a escrito en base bm son las mismas que en base b pero tomadas de

derecha a izquierda en grupos de m: Cada grupo de m cifras en base b corresponde a un

dígito en base bm.

Ejemplo 6. Para pasar de base 2 a base 8 el entero (10010101110001110)2 traducimos

los grupos de 3 así:

(110)2=6; (001)2=1;

(110)2=6; (101)2=5;

(010)2=2; (10)2 =2;

entonces las cifras en base 8 son 6, 1, 6, 5, 2, 2 (tomadas de derecha a izquierda) es decir:

(10010101110001110)2=(225616)8.

Si quisiéramos pasar a base 16 escogeríamos grupos de a 4 y tendríamos:

(1110)2=E ; (1000)2=8; (1011)2=B; (0010)2=2; (1)2=1 y tenemos:

(10010101110001110)2= (12B8E)16.

Cuando la base es muy grande podríamos agotar las letras del alfabeto, entonces no se

acostumbra colocar nuevas letras, sino dejar los dígitos decimales. Por ejemplo para pasar

de base decimal a base 100, se necesitaría agregar 90 nuevos dígitos, mejor entender las

parejas de dígitos decimales como dígitos centesimales, así:

(19168639)10 = (19-16-86-39)100

Page 157: A Notas de Algebra Moderna

El aplicar el lema 2 con alguna de las proposiciones 1, 2 o 3, es un m‚todo para conseguir y

explicar otros criterios de divisibilidad, como se muestra en los siguientes ejemplos:

Ejemplo 7. (Base decimal) Para saber si un entero es múltiplo de 100, todos sabemos que

basta con saber si sus dos últimas cifras son exactamente 00. Claro! Cuando escribimos

los números en base 100 los múltiplos de 100 son los que terminan en cero, pero este dígito

se representa con dos ceros de la base decimal.

Por otra parte, aplicando la proposición 1 al lema 2 obtenemos que por ejemplo cualquier

entero es congruente modulo 25 con sus dos últimas cifras ya que 25 divide a 100.

Otra manera de saber si un número escrito en base 10 es divisible por 11 es hacer la suma

de sus cifras tomadas de dos en dos. Por ejemplo:

19168639≡19+16+86+39=160≡60+1≡6(mod 11).

Compare con el ejemplo 5!

Ejemplo 8. Busquemos un criterio para saber si un entero escrito en base 2 es múltiplo de

3. Como 4≡1(mod 3) entonces en base 4 se puede aplicar la proposición 2 y tenemos que

un número escrito en base 4 es congruente módulo 3 con la suma de sus cifras (ejemplo 4)

pero según el lema 2 las cifras de base 4 son las parejas de cifras en base 2, entonces: "Un

número escrito en base 2 es múltiplo de 3 si y solo si la suma de sus cifras tomadas de dos

es dos de derecha a izquierda es múltiplo de 3".

5. SEPARANDO LAS ÚLTIMAS CIFRAS

El operador "quitar" la última cifra, o las dos últimas, parece que no fuera un operador

aritmético. Según el Teorema fundamental de la numeración, y trabajando en base decimal,

si la última cifra de n es a, entonces n tendrá la forma

n=10n’+a

Page 158: A Notas de Algebra Moderna

con 0<a<10, en donde n' es precisamente “lo que queda”. Si lo que se quita a n son las dos

últimas cifras, entonces convendrá considerar la forma de n como

n=100n'+a

en donde a es el valor de las dos últimas cifras (0<a<100) y n' es “lo que queda”.

Utilizando este sencillo operador, podemos encontrar útiles criterios de divisibilidad, que se

justifican por alguna ecuación de congruencias.

Ejemplo 9. Un número escrito en base 10 es divisible por 17, si y solo si, al quitar sus dos

últimas cifras y restarlas del duplo de lo que queda el resultado es múltiplo de 17.

Por ejemplo, para saber si 4767 es múltiplo de 17: quito 67 y lo que queda es 47, su duplo

94, menos 67, obtengo 27, que no es múltiplo de 17 por tanto 4767 tampoco lo es, pero si

pruebo con 4267 tengo (42×2)-67 =84-67=17, y por tanto 4267 si es múltiplo de 17.

La demostración de que este procedimiento es válido, parte de considerar n con la forma

n=100n'+b , donde n' es lo “que queda” y b es el número representado por las dos últimas

cifras, debemos ver que n≡0(mod 17) si y solo si, 2n'-b≡0(mod 17) y esta es una

equivalencia de congruencias, que se demuestra fácilmente:

n=100n'+b≡0(mod 17) ⇔ 15n'+b ≡0(mod 17) ⇔ -2n'+b≡0(mod 17) ⇔ 2n'-b≡0(mod 17)

PREGUNTAS Y EJERCICIOS.

Demostrar los siguientes criterios de divisibilidad:

1. En base diez un número es divisible por 2, si termina en cifra par.

2. En base diez un número es divisible por 5, si termina en 0 o 5.

Page 159: A Notas de Algebra Moderna

3. En base diez un número es divisible por 20, si termina en 00, 20, 40, 60 ó 80.

4. En base 12 un número es divisible por 3, si su última cifra lo es.

5. En base 12 un número es divisible por 6, si termina en 0 ó 6.

6. En base 2 los múltiplos de 4 son aquellos que terminan en 00.

7. En base diez un número es divisible por 9, si la suma de sus cifras es divisible por 9.

8. En base 6 un número es divisible por 5, si la suma de sus cifras es divisible por 5.

9. En base diez un número es divisible por 37, si la suma de sus cifras tomadas de tres en

tres, es divisible por 37.

10. En base 4 un número es divisible por 15, si la suma de sus cifras, tomadas de os en dos,

es divisible por 15.

11. Si a un número que está escrito en base 10 y es múltiplo de 13, se le quita su última

cifra y se le suma multiplicada por 4 a lo que queda, el resultado es múltiplo de 13.

12. Si a un número que está escrito en base 10 y es múltiplo de 11, se le quita su última

cifra y se le suma a lo que queda, el resultado es múltiplo de 11.

13. Enuncie y demuestre un criterio para saber si un número escrito en base decimal es

divisible por 19.

Page 160: A Notas de Algebra Moderna

14. TODO DEPENDE DE SABER CONTAR

Nos proponemos analizar los algoritmos para efectuar operaciones aritméticas. Otra vez

nuestro interés más que práctico es teórico: El análisis de diferentes algoritmos nos

proporcionar para los números, sus relaciones y operaciones. Buscando lo fundamental

vemos que lo mas elemental que podemos hacer es contar. Sabiendo contar, es decir

sabiendo sumar 1, podemos realizar los algoritmos más comunes de la aritmética: Sumar

dos números, restarlos, multiplicar, hallar cociente y residuo, etc.

Para ver esto imaginemos que se deja programar, pero de aritmética no sabe sino sumar 1.

Por dejarse programar entendemos que guarda números en memoria, que se nota en letras

mayúsculas, asignar 6 a la memoria A, se notará

6 → A

asignar a la memoria B, lo que est en la memoria A así

Determinar si los contenidos de las memorias A y B coinciden y según la respuesta seguir

en procedimiento, lo que se notará así

Para presentar resultado se utilizar el símbolo

y para finalizar el signo

A → B

A = B Si

No

A ES EL RESULTADO

PARE

Page 161: A Notas de Algebra Moderna

Las asignaciones se enumeran en un rectángulo. Si suponemos que nuestra máquina sólo

suma 1, el siguiente algoritmo representa un algoritmo que muestra los números de dos en

dos, hasta llegar a m-2.

N es la variable donde se lleva la cuenta, se inicia en 0, y luego de presentarse se

incrementa en 1 dos veces, para ese N ¿M volverse a presentar y seguir sucesivamente

hasta que N sea igual a M, pero nótese que si en la variable M va un número impar el

algoritmo no se detiene. ¿Cómo hacer para que el proceso se detenga siempre que M sea

entero positivo?

El primer método que un niño conoce para sumar es contar con los dedos de las manos.

Para 3 y 6, se cuenta a partir de 6 y en una mano se lleva la cuenta de 1 incremento, cuando

esa mano se lleve 3 entonces el proceso se detiene. Este algoritmo es representado por el

siguiente diagrama en el cual se suma a y b.

Los sumandos a y b son guardados en las memorias s y t, las memorias s y MANO se van

incrementando de 1 en 1 hasta que MANO alcance a T, cuando esto suceda, la suma de los

dos números es lo que est en la memoria 5.

Para hacer el seguimiento de cualquier algoritmo se puede hacer lo que se llama "una

Prueba de Escritorio", que es construir una tabla que muestre el valor de las diferentes

PARE 0 → N N N+1 →N N+1 →N N = M Si

No

a → S b → T 0 → MANO

MANO=T Si

No

S+1 → S MANO+1 → MANO

a+b=5 PARE

Page 162: A Notas de Algebra Moderna

variables; una vez dados los datos iniciales. Para el ejemplo la tabla 1 muestra una prueba

de escritorio cuando? Sumar 3 y 6.

S T MANO MANO=T

6 3 0 NO 7 1 NO 8 2 NO 9 3 SI

Tabla 1.

Por otra parte debemos observar que esta acertada manera de sumar con los dedos de las

manos, está basada en la deficiencia recursiva de suma (a partir de sumar 1). Ese efecto

recursivamente se puede definir n+m así:

i) n+0=n

ii) n+(m+1)=(n+m)+1

Para sumar 6+3 según esta definición, por i) saben que 6+0=6 por ii) saben que

6+1=6+(0+1)=(6+0)+1=7, otra vez aplicando ii) obtenemos 6+2:

6+(1+1)=(6+1)+1=7+1=8

y así:

6+3=6+(2+1)=(6+2)+1=8+1=9

El algoritmo que acabamos de mostrar es una demostración de la siguiente proposición.

Proposición 1. Una máquina que sólo suma 1, puede sumar cualquier par de números

naturales.

La máquina que pensamos ni siquiera decide cúal es el mayor de un par de números. Ella

sólo sabe decir cuándo un par de números coinciden. Como ejercicio el lector debe

programarla para que decida cúal de los números naturales a y b es el mayor. Se trata de ir

calculando en memorias auxiliares incrementos de cada una de los números hasta que el

uno alcance el otro. Esto demuestra la siguiente proposición.

Page 163: A Notas de Algebra Moderna

Proposicion 2. Una máquina que sólo suma 1, puede programarse para que decida cúal

de los números es mayor. También se puede programar para hallar la diferencia.

Las operaciones que hace nuestra máquina las efectúa con cantidades positivas, pero se

pueden ampliar fácilmente para que trabaje con negativos. Sin embargo seguiremos

trabajando sólo en números naturales.

Las dos proposiciones anteriores no permiten que de ahora en adelante que nuestra máquina

ya puede sumar cualquier par de números naturales. Hallar la diferencia del mayor al

menor y decidir cúal es el más grande, es decir podemos hacer asignaciones del siguiente

tipo: A+B → C, A-B → C y tomar décimas de tipo

Con estos procedimientos describiremos un algoritmo para que dados a y b, enteros

positivos, encuentre el cociente al dividir a entre b.

La base de este algoritmo es supremamente sencilla; el método más fácil para repartir A

objetos entre B personas, es darle un objeto a cada persona (se resta B de A), luego vuelve a

repartir una a cada uno (resta B del RESTO) y así hasta lo que quede (EL RESTO) sea

A ≥ B Si

No

A > B Si

No

A : B A>B

A=B

a → A a → RESTO b → B 0 → C

RESTO<B Si

No

C+1 → C RESTO-B → RESTO

Cociente de dividir a entre b

PARE

Page 164: A Notas de Algebra Moderna

menor que el mínimo de personas (RESTO<B). El cociente es lo que a cada persona le

toco!

Ahora debemos enseñarle a multiplicar, expresar en base b un número, saber si tal número

es primo, etc. Pero esto lo dejamos como ejercicio para el lector.

EJERCICIOS

1. Suponga que la máquina C, que sólo suma uno, sin ninguna subrutina; hacer programas

para:

a. Muestre los primeros n múltiplos de 2.

b. Muestre los primeros n múltiplos de 3.

c. Determine si n es par o impar.

d. Halle el cociente y el residuo al dividir en 2.

e. Halle el cociente y el residuo al dividir en 3.

f. Muestre los números de la forma 4k+1.

g. Halle la suma de los n primeros números.

h. Dados a y b encuentre |a-b|.

i. Dados a y b encuentre a÷b que es a-b si a>b y cero en caso contrario.

2. Suponga una máquina C, que tiene subrutinas para sumar cualquier par de números y

decidir cúal de los dos es el mayor además de restar de un mínimo otro menor o igual.

Diseñar un algoritmo para que la máquina:

a. Dado a y b encuentre el residuo de dividir a entre b.

b. Multiplique a y b.

c. Halle el máximo común divisor de a y b ( se puede hacer sólo restando).

d. Dados n, k, r

( ) ( ) ( ) ( )∑=

+++++++=+n

inkrkrkrrrik

0...2

e. Dados m y n positivos decidir si n|m.

f. Decidir si n es primo.

Page 165: A Notas de Algebra Moderna

g. Encuentre n2.

3. Supóngase ahora que nuestra máquina tiene subrutinas para las operaciones siguientes:

Suma, resta, producto, cociente y residuo (se notan COC (a,b), RES (a,b)). Diseñar

algoritmos para que la máquina:

a. Calcule n!

b. Calcule a-6.

c. Exprese a en base b.

d. Exprese (a,b) como combinación lineal de a y b.

e. Exprese n como producto de factores primos.

4. ¿Qué hace el siguiente algoritmo?

0 → S a → A b → B

RES(4,2)=0 No

Si

COC(4.2) → A B × A → B

S=B+S

PARE

A=0

S

Page 166: A Notas de Algebra Moderna

15. ALGORITMOS BASADOS EN EL SISTEMA DE NUMERACION.

Los algoritmos expuestos en la sección anterior trabajan con números en "abstracto" por

cuanto en ellos nada tiene que ver la representación de los números. Esta es una de las

razones por la que esos algoritmos resultan ser tan poco prácticos, ya que una operación

con números moderadamente grandes supone mucho tiempo de computación para ser

llevada a cabo.

Cuando los números están escritos en determinado sistema de numeración se utilizan

algoritmos muchos más rápidos que los expuestos anteriormente y son estos algoritmos los

que se utilizan para hacer las operaciones básicas tanto manualmente como cuando se

trabaja con aparatos de computación.

El sistema de numeración en base diez, o sistema induarábico, se impuso definitivamente

en nuestra civilización precisamente por ser mas práctico especialmente cuando se trabaja

la multiplicación y la división, como ya se dijo en la sección 14. La palabra 'algoritmo'

apareció en Europa conjuntamente con la aceptación del sistema induarábigo, para referirse

a los métodos de cómputo de los calculistas de "pluma y papel'.

Hoy en día, con la aparición de los ordenadores la palabra algoritmo se refiere a cualquier

procedimiento ejecutable por la máquina, pero los cálculos sobre números que hace un

ordenador se siguen basando en algoritmos parecidos a los manuales pero diseñados para el

sistema de numeración posicional binario.

Estos ser n los algoritmos que trabajaremos en esta sección. Algoritmos rápidos, que se

utilizan para hacer las operaciones manualemente o por computador. Nuestro interés no es

que el lector llegue a manejar estos algoritmos con destreza, lo que perseguimos es que se

Page 167: A Notas de Algebra Moderna

logre descifrar el por qué de su efectividad, lo que a la larga puede dar la oportunidad de

diseñar algoritmos propios.

Empezamos con la descripción de un método muy antiguo empleado para multiplicar, que

identificaremos como método de multiplicación de los campesinos rusos, que además de

ser práctico, evita el uso de las tablas de multiplicar, pues solo exige saber sumar y saber

multiplicar y dividir por dos. Para multiplicar dos números se dice que los campesinos

rusos colocaban los dos números un al lado del otro y mientras uno se va dividiendo por

dos el otro se va multiplicando por dos. Las divisiones se hacen, naturalmente enteras y los

números no pares de esta columna se distinguen digamos con un * . El proceso termina

cuando en la columna de los números que se van dividiendo se encuentre 1. El resultado

del producto se encuentra sumando los números correspondientes a * en la columna de los

números que se van multiplicando por 2. En la tabla 1 se ilustra el proceso

cuando se trata de multiplicar 312 por 45.

El lector puede comprobar con otras multiplicaciones que realmente este método es

efectivo, pero el reto que se plantea es explicar el por qué. Es decir, se quiere una

demostración de que el método sí funciona (ejercicio 6). Por ahora como una pista digamos

312 45 156 90 78 180

* 39 360 * 19 720 * 9 1440 4 2880 2 5760

* 1 11520 312×45=360+720+1440+11520

=14040 Tabla 1. Método de multiplicación de los campesinos rusos aplicado para efectuar 312×45. En la columna de la izquierda se hacen sucesivas divisiones por 2 a partir de 312 (* denota que la división no es exacta). En la otra columna se hacen sucesivas multiplicaciones por 2 a partir del otro multiplicando que es 45. El producto se obtiene sumando los valores de la columna derecha correspondientes a *.

Page 168: A Notas de Algebra Moderna

que aunque este procedimiento es válido no importa en que sistema de numeración se

trabaje, su fundamentación radica en la expresión de uno de los multiplicandos en base 2.

Ahora queremos revisar los algoritmos usuales aprendidos en la escuela primaria para

efectuar las operaciones básicas. Empecemos por imaginarnos cómo se suma y se resta en

sistemas aditivos, por ejemplo en el sistema egipcio explicado en la sección 13. La suma y

la resta en estos sistemas es realmente sencilla y es de suponer que se actuaba tal como lo

hacían los abaquistas, o hoy en día, cualquier tendero que quiere contar su dinero. Se trata

de unir los signos similares y cuando se obtienen suficiente número de ellos se cambian por

otro que represente un número mayor. Veamos el mismo ejemplo para sumar 5 y 9 que se

mostró en la sección 10:

///)(////////////////

/////////∩∩ ==+=+

Cuando se completan 10 palotes se reemplazan por el numeral correspondiente ∩ . Como

cuando se cambian 10 monedas de $1 por una de $10! El lector debe practicar otras sumas

en éste y otros sistemas aditivos. Es al fin y al cabo un ejercicio de tenderos.

Ahora bien, cuando se suma en sistemas posicionales el principio es el mismo: Si

trabajamos en base 10, reunimos primero las unida des, si ellas sobrepasan a 10, guardamos

una decena, así reunimos luego las decenas, siempre que encontremos diez decenas

guardamos una centena y así sucesivamente. La razón por la cual cuando suma mos las

cifras de un columna si la suma pasa de diez "llevamos" a la siguiente columna, lo que

estamos haciendo es cambiar billetes de una denominación por otro de una denominación

mas alta.

El argumento también es válido para bases diferentes de diez. Si se trabaja en base b, para

sumar dos números se van sumando sus cifras correspondientes y cuando la suma

sobrepasa b se "lleva 1" a la columna siguiente. Así, si vamos a sumar (312)5 y (33)5

Page 169: A Notas de Algebra Moderna

podemos colocar los números como si fuéramos a sumarlos en sistema induarábigo y

sumamos columna por columna, de derecha a izquierda, teniendo en cuenta que cuando la

suma sobrepase 5 escribimos la suma menos 5 y "llevamos 1" a la siguiente columna.

40033

312

Al sumar los números de la primera columna de la derecha, las unidades, obtenemos

2+3=5, pero el dígito 5 no existe, en base 5 es 10 por lo tanto se escribe 0 y se lleva 1.

Entonces en la segunda columna sumamos 3 y 1 y el 1 que llevábamos, obtenemos de

nuevo 10 (o sea 5 ), escribimos 0 y llevamos 1 a la tercera columna.

Ahora describiremos este algoritmo de manera mas precisa pensando en una máquina que

recibe números hasta de n+1 dígitos y trabaja en base b. El fundamento del algoritmo lo

especificamos en la siguiente proposición:

Proposición 1. Si a=(an an-1 ... a0)b y entonces a+b=(cn+1 cn ... c0)b donde los ci se

calculan así: r0=0, si ai+bi+ri<b entonces ci=ai+bi+ri y ri+1=0

si ai+bi+ri>b entonces ci=ai+bi+ri-b y ri+1=1 cn+1=rn+1.

Demostración. (Ejercicio).

Los ri indican lo que se lleva de la anterior columna. Claro que si la máquina recibe sólo

números de n+1 cifras cuando rn+1=1 esto significa que la suma ha rebosado la capacidad

de la máquina y se debe emitir una señal de error.

Supongamos que la máquina trabaja con b=2 como en realidad sucede con los modernos

computadores. Un sumador lo podemos ver como una caja negra (en cuanto no nos

interesa cómo actúa internamente) que recibe tres entradas, dos por donde llegan los

números que se van a sumar dígito a dígito y otra por donde se retroalimenta con lo que

"lleva". El sumador tiene además dos salidas: una por donde indica los dígitos de la suma y

otra que indica cuanto lleva y retroalimenta al sumador pasando por un "delay" que retiene

Page 170: A Notas de Algebra Moderna

la señal por un "instante" (estos aparatos funcionan controlados por un reloj que emite

pulsos en intervalos de tiempo muy pequeños).

La figura 1 muestra un esquema del sumador en donde ai y bi son los dígitos de los

números a sumar, ri es lo que se lleva y si indica los dígitos de la suma.

Los ai y bi entran externamente, mientras los ri son producidos por el mismo sumador salvo

r0 que es 0. Insistimos en que el fundamento de este sumador es el mismo que el de la

suma usual y que se expone en la proposición 1. En cierto sentido el sumador sabe sumar

dos cifras en base 2. Esto es muy fácil y se lleva a cabo según se indica la tabla 2.

Recomendamos al lector hacer un análisis similar cambiando la operación suma por la

resta. Habrá que elaborar y probar una proposición análoga a la proposición 1 para en base

a ello describir el método general de restar (que es válido en cualquier base), y luego

elaborar una tabla análoga a la tabla 2 que nos indique cómo funcionaría un "restador

secuencial".

Sumador

Delay

ai bi

ri

si ri+1

Figura 1. Diagrama de un sumador secuencial

ai bi ri si ri+1 0 0 0 0 0 0 0 1 1 0 0 1 0 1 0 1 0 0 1 0 0 1 1 0 1 1 0 1 0 1 1 1 0 0 1 1 1 1 1 1

Tabla 2. Salidas de acuerdo a las entradas del sumador secuencial.

Page 171: A Notas de Algebra Moderna

La multiplicación usual. Sabemos ya que el algoritmo usual que se utiliza para sumar es

válido en cualquier base. La primera pregunta que nos proponemos responder es la

siguiente: Es válido el algoritmo usual de la multiplicación en cualquier base? Antes de

responder esta pregunta deberemos describir generosamente este algoritmo. Se distinguen

dos casos :

Multiplicación de cualquier número por otro de una sola cifra y multiplicación de dos

números cualesquiera.

En cualquier caso el primer paso consiste en memorizar unas odiosas tablas de multiplicar.

Consiste en aprender "de memoria" (o guardar en memoria ?) el producto de todos los

números de una sola cifra. En base 10 debemos aprender 81 resultados!

Para la multiplicación de cualquier número por otro de una sola cifra el procedimiento

secuencial es parecido al de la suma: se va multiplicando cifra por cifra y se "lleva" las

decenas para irlas acumulando con el siguiente producto. Recordemos: 234×6: 6 por 4 da

24 escribe 4 lleva 2; 6 por 3 da 18 y 2 que se llevaba son 20, escribe 0 y lleva 2; 6 por 2 da

2 y 2 que se llevaba son 14, como no hay más cifras escribe 14.

Sabiendo que para multiplicar por la 10 simplemente se agrega un cero la fundamentación

del algoritmo radica en la ley distributiva. Para el ejemplo, el procedimiento quedaría claro

si lo escribieramos así:

234×6 = (2×100 +3×10+4) ×6 = (2×100 +3×10) ×6+4×6 = (2×100 +3×10) ×6+2×10+4 = (2×100) ×6+(3×6+2) ×10+4 = (2×6×100)+(2×10+0) ×10+4 = (2×6+2) ×100)+0×10+4 = (1×10+4)×100+0×10+4 = 1×1000+4×100+0×10+4 = 1404

Page 172: A Notas de Algebra Moderna

Este procedimiento que indudablemente es mas largo pero en él todo queda justificado, lo

denominaremos multiplicación PASO A PASO.

Proposición 2. Si k=(anan1...a0)b y t es un número de una sola cifra entonces

kt=(cn+1cn...c0)b donde los ci se calculan así: r0=0, si ait+ri=(ef)b entonces ci=f y ri+1=e

Demostración. (ejercicio).

Mas importante que la demostración de la proposición 2, es comprender su significado:

describe y justifica el algoritmo usual para multiplicar un número cualquiera por otro de

una sola cifra. Para visualizar esto volvamos al ejemplo de la multiplicación de 234 por 6.

En este caso según la notación de la proposición 2, b=10, c0=4, c1=3, c2=2 y t=4. La

siguiente tabla muestra el procedimiento, que es el mismo utilizado anteriormente en donde

los ri indican lo que se "lleva":

i ai ri (6ai+ri) ri+1 ci 0 4 0 24 2 4 1 3 2 20 2 0 2 2 2 14 1 4 3 0 1 2 0 1

La proposición 2 además implica que el algoritmo para multiplicar en base 10 por un

número de una sola cifra es válido para cualquier base. Trabajemos por ejemplo en base 5.

La tabla de la suma y el producto se muestran en la tabla 3.

Hallemos el producto de (323)5 y 4 tendríamos: b=5, a0=3, a1=2, a2=3, t=4. Deseamos

hallar, según la proposición 2, los ci.

+ 1 2 3 4 1 2 3 4 102 3 4 10 113 4 10 11 124 10 11 12 13

× 2 3 4 2 4 11 13 3 11 14 22 4 13 22 31

Tabla 3. Suma y producto en base 5

Page 173: A Notas de Algebra Moderna

i ai ri (ait+ri)b ri+1 ci 0 3 0 22 2 2 1 2 2 20 2 0 2 3 2 24 2 4 3 0 2 2 0 2

Tenemos entonces que (323)5×4=(2402)5, como prueba podemos pasar a base 10:

(323)5=3×25+2×5 +3=(88)10 y (2402)5=2×125+4×25+2=(352)10, y en efecto sabemos que

88×4=352.

El lector deber ejercitarse en la multiplicación de un número cualquiera por otro de una sola

cifra para cualquier base. Esperamos que así quede explicado totalmente este caso.

Para la multiplicación de dos números cualesquiera el algoritmo usual es también válido

para cualquier base. Para multiplicar, por ejemplo, 342 por 135, primero multiplicamos 5

por 342, luego 3 por 342 y colocamos este producto un lugar corrido a la izquierda,

finalmente 342 por 1 y colocamos el resultado dos lugares hacia la izquierda para así hacer

la suma que nos proporciona el resultado.

4617034210261710

135342×

Lo aparentemente extraño en este procedimiento es por qué se corren los resultados a la

izquierda. Esto queda claro si lo escribimos así:

342×5 = 1710 342×30 = 10260

342×100 = 34200 342×135 = 46170

Page 174: A Notas de Algebra Moderna

Este procedimiento se deriva entonces de la aplicación de la ley distributiva del producto y

del hecho que al multiplicar por 10 (la base) se agrega un cero. Es fácil ver que entonces

nuestro procedimiento es válido en cualquier base. Ahora bien en base dos se tiene la gran

ventaja de que no toca aprender tablas de multiplicar, basta con saber sumar, pues

multiplicar por 0 y por 1 es trivial.

LA DIVISION USUAL: El algoritmo utilizado usualmente para dividir es de todos

indudablemente el más confuso. Se invita al lector a que haga una prueba no matemática:

Pregúntele a diez personas de nivel universitario el por qué hace las divisiones de la forma

corriente, y si alguno le explica satisfactoriamente el por qué, regálele mil pesos. Seguro

no gastar mucha plata!

Nos limitaremos a traducir una división corriente por la misma pero paso a paso, con la

esperanza de que el lector dilucide qué es lo que se hace para poder diseñar una forma de

escribir las divisiones sin que se pierda la esencia de lo que se hace. Pensemos en dividir

3472 entre 32:

3472 32 -32 108

272 -256

16

Ahora observemos la misma división PASO A PASO:

3472 = 3400+72 = 100×32+200+72 = 100×32+272 = 100×32+8×32+16 = (100+8)×32+16 = 1080×32+16

Recordemos primero que cuando se trata de dividir a entre n lo que se busca es un cociente

q y un residuo r tal que a=bn+r y que 0<r<n. Esto es fácil y rápido si los números son de

magnitudes parecidas es decir cuando el cociente no es muy grande, por ejemplo para

Page 175: A Notas de Algebra Moderna

dividir 272 entre 32 se puede ir multiplicando, 32 por 1 luego por 2 y así hasta que el

producto supere a 272.

Este método se sugirió en la sección anterior. Pero para dividir 3472 entre 32 habría que

hacer entonces 108 productos, cosa nada practica ! Según se ve en la división paso a paso

del ejemplo, lo que se hace es primero dividir 3400 entre 3200 (aparece 1 al cociente) luego

se divide 270 entre 320 (aparece el 0 del cociente) y finalmente 272 entre 32 (aparece el 8

del cociente y el residuo final).

Para la división el algoritmo usual también es válido cuando se trabaja con otras bases.

Dividamos por ejemplo (2324)5 entre (13)5.

Por comodidad omitamos la referencia a la base para este ejemplo. Los primeros múltiplos

de 13 en base 5 son: 13, 31, 44, 112; empezamos dividiendo 2300 entre 1300 nos da 1 y

sobra 1000; ahora debemos dividir 1020 entre 130, da 3 y nos sobra 1020-440=30.

Finalmente dividimos 34 entre 13 no da 2 y sobra 3. Esta división paso a paso la

escribimos así:

2324 = 2300+24 = (13×1×100+1000)+24 = 13×1×100+1020+4 = 13×1×100+(13×3×10+30)+4 = 13×1×100+13×3×10+34 = 13×1×100+13×3×10+13×2+3 = 13×(1×100+3×10+2)+3 = 13×132+3

Y resumidamente según el algoritmo usual, la operación se puede escribir así:

2324 13 102 132 34 3

Page 176: A Notas de Algebra Moderna

COMENTARIO FINAL.

El lector después de una lectura superficial a esta sección, puede pensar que lo que se ha

hecho es complicar cosas que para él estaban muy claras. El primer avance es descubrir

que los algoritmos usuales que se enseñan en la escuela primaria para las operaciones

básicas no son tan claros, en especial el caso del producto y la división. Esto se entiende

por cuanto estos algoritmos se impusieron por razones prácticas, como ya se dijo, y para los

calculistas antiguos de pluma y papel, omitir los ceros, por ejemplo en el caso de la

multiplicación, significaba ahorro de tiempo y tinta (que era muy costosa). Hoy en día,

estas razones no son v lidas y el maestro del siglo XXI deber explicar estos algoritmos

buscando primero su comprensión antes que la rapidez y efectividad en los cálculos.

Queda pues el reto pedagógico de diseñar métodos para efectuar los mismos algoritmos de

tal manera que el alumno no pierda de vista el sentido de lo que se est haciendo.

PREGUNTAS Y EJERCICIOS

1. Efectuar en la base indicada:

a. (231123)4+(322122)4

b. (A2B3)16-(BB1)16

c. (10110011000)2- (11101)2

2. Para multiplicar un número por 5 se le agrega un cero y se divide por 2. Demostrar que

esta regla es válida cuando se trabaja en base decimal. Es v lida esta regla en otras

bases?

3. a. Exprese un algoritmo para sumar d días, h horas, m minutos con d' días, h' horas, m'

minutos.

b. Exprese un algoritmo para restar d días, h horas, m minutos con d' días, h' horas, m'

minutos.

Page 177: A Notas de Algebra Moderna

4. Demostrar que si a=(anan-1...a0)b entonces ab=(anbnan-1...a0)b. Esto demuestra que para

multiplicar un número escrito en base b por b simplemente se agrega un 0.

5. Al multiplicar un múltiplo de 3 menor que 30 por 37 se obtiene siempre como producto

un número de 3 cifras iguales (base decimal). ¿Explique por qué?

6. Explicar el por qué del método de multiplicación de los campesinos rusos. (Ayuda:

Exprese en base 2 el multiplicando que se va dividiendo ).

7. Demostrar la proposición 1 por inducción sobre n el número de cifras de los sumandos

(ayuda: Si a=(anan-1...a0)b entonces a=anbn+(an-1...a0)b ) .

8. Una docena son 12 unidades y una grueza son 12 docenas. Usando aritmética en base

12 resolver las siguientes preguntas:

a. Si 3 gruezas,5 docenas y 8 huevos se sustraen te un total de 9 gruesas y 2 docenas,

cuántos huevos quedan.

b. Si un supermercado recibe 3 cajas de remesas de huevos cada una de 3 gruesas 5

docenas y 8 huevos, cuantos huevos recibió en total.

c. Si 11 gruesas, 10 docenas y 6 huevos se dividen en tres grupos de igual tamaño, con

cuantos huevos queda cada grupo?

9. Efectuar las siguientes operaciones en la base indicada:

a. (231123)4×(3)4 b. (A2B3)16×(3)16

c. (10110011000)2×(11101)2 d. (2313)4×(21)4

e. (10110011000)2÷(11101)2 f. (1123)4÷(12)4

10. Efectuar las operaciones de los ejercicios 2 y 9 ,paso a paso.

Page 178: A Notas de Algebra Moderna

11. (Base decimal).

a. Demostrar que si se toma un número de 3 cifras no repetidas y se suma con otro con

las mismas 3 cifras pero en sentido contrario, y se resta del mayor el menor, se

obtiene siempre un número de tres cifras cuyas cifras de los extremos suman 9 y la

intermedia es 9.

b. Si a,b,c son tres dígitos tales que a+c=9 y b=9 entonces demuestre que

(abc)10+(cba)10=1089

c. Lo anterior justifica el siguiente acertijo:

Piense un número de tres cifras cualesquiera (ejemplo 347) sin divulgarlo tome el

número con las cifras contrarias (743) y reste del mayor el menor (743-347=396), el

resultado súmele el mismo número pero de cifras contrarias (396+693=1089) siempre

se obtiene 1089 .

12. Para elevar al cuadrado un número que escrito en base decimal termina en 5 (por

ejemplo 145) se toma el numero que queda al omitir el 5 (es decir 14) y se multiplica

por su siguiente (14×15=210) al resultado se le adjunta 25 y ese es el cuadrado del

número ( 1452=21025 ).

a. Demostrar que esta regla es válida.

b. Enuncie y demuestre una regla similar para elevar al cuadrado números que, escritos

en base 2b, terminen en b.

13. Análogo al sumador secuencial que se presentó en esta sección:

a. Construir un "sustractor secuencial" para números escritos en base 2.

b. Construir un "duplicador secuencial" para números escritos en base decimal.

14. Para multiplicar en base decimal se puede utilizar la siguiente regla:

"Sumar a cada dígito, si es par la mitad de su vecino a la derecha, si es impar sumarle 5

además ( las mitades se consideran en su parte entera y el vecino del primer dígito, de

derecha a izquierda, es el 0)"

Page 179: A Notas de Algebra Moderna

Por ejemplo para multiplicar 152 por 6

2 no tiene vecino a la derecha y es par, pasa 2

5 su vecino es 2, mitad 1, 5+1=6 como 5 es impar se le agrega

además 5, obteniéndose 11, llevamos 1 y pasa 1

1 su vecino es 5, mitad 2, suma 7 mas lo que llevábamos nos da

8 y como 1 es impar se suman 5 obteniendo 13

el producto es…………………………………............................................1312

a. Describir formalmente el algoritmo.

b. Explicar formalmente por qué funciona (Ayuda: Nótese que multiplicar por 6 es

multiplicar por 5+1. Aproveche el ejercicio 2 de esta sección).

15. Diseñe un método para dividir dos números que sea el inverso de la multiplicación de

los campesinos rusos.

16. Demostrar la proposición 2 por inducción sobre n el número de cifras del número de

varias cifras que se va a multiplicar (ayuda: Si a=(anan-1...a0)b entonces

a=anbn+(an-1...a0)b

Page 180: A Notas de Algebra Moderna

" A parte de las imágenes que

es todo lo que podemos ver realmente, imaginemos un mundo de cosas sólidas;

y ..., este mundo está constituido de tal manera que cumple un cierto

código de reglas, algunas llamadas axiomas, otras definiciones, otras postulados y

algunas admitidas en el curso de la demostración ..."

WILLIAM K. CLIFFORD

(1845-1879)

A1. EL MÉTODO AXIOMÁTICO.

Hemos visto anteriormente algunas demostraciones. Se hace una demostración para

despejar dudas sobre una proposición viendo que ella se deriva de otras de las cuales no se

tienen dudas. Pero “tener dudas” es algo relativo. Podría suceder lo que pasa ante un niño

que pregunta y al obtener la respuesta hace otra pregunta y así indefinidamente. Lo mismo

sucede con las definiciones. Se define un término conocido en base a otros ya conocidos,

pero estos a su vez, para no dejar dudas, necesitan ser definidos y así sucesivamente.

¿Hasta cuándo?

Parece que las explicaciones de la vida terminan en círculos viciosos. (Terminan?)

Históricamente, hasta donde se conoce, los griegos fueron los que hicieron demostraciones

matemáticas. Pitágoras o alguno de sus alumnos, demostró el teorema que lleva su nombre,

ya conocido por civilizaciones anteriores como la de babilonia; los pitagóricos y otras

escuelas lograron otras demostraciones tanto de proposiciones de la geometría como de la

aritmética. Fue Euclides quien elaboró un cuerpo admirablemente armónico de todos sus

conocimientos en sus famosos “ELEMENTOS DE GEOMETRÍA” en donde todas las

proposiciones eran demostradas a partir de otras que ya lo habían sido (TEOREMAS) o que

eran aceptadas desde un principio como verdades (AXIOMAS)

Page 181: A Notas de Algebra Moderna

Los “Elementos de Geometría ” de Euclides forman una de las obras más importantes de

toda la historia y por más de veinte siglos son la base de la enseñanza de la geometría y la

matemática en el mundo occidental.

El método axiomático desarrollado por los griegos, esto es, a partir de verdades aceptadas e

indiscutibles para que por medio únicamente de raciocinios lógicos se lleguen a otra s no

triviales, se convierte en la máxima aspiración de filósofos y científicos de casi todas las

épocas, algunas veces con gran éxito como en el caso de la mecánica de Newton.

Pero la geometría de Euclides tuvo su gran crisis en el siglo XIX cuando la discusión sobre

el quinto postulado(*) desemboca en la construcción que hacen Lobachewsky y Bolyai y

más tarde Riemman, de geometrías que contradicen dicho postulado y conservan todo su

rigor lógico. Surgen pues, muchas inquietudes que obligan a replantear ¿QUÉ EL

MÉTODO AXIOMÁTICO? ¿Cuál es la “verdadera geometría”? La que adopta como

axiomas las “verdades evidentes?” Pero, las “verdades evidentes” son claramente relativas

y no siempre son verdades universales pues en principio, por ejemplo, es evidente que la

tierra es plana y que el sol gira alrededor de ella, pero sabemos que esto no es así.

Hoy día el método axiomático sigue siendo tan importante como antes aunque en las épocas

modernas la concepción general sobre dicho método ha cambiado.

Hay dos diferentes que resaltamos: Primero, la teoría que resulta susceptible de ser

aplicada no sólo a una situación particular sino a todas aquellas donde los axiomas,

dándoles cierta interpretación, se cumplen; en estos casos se dice que se tiene un MODELO

para la teoría. Segundo, los axiomas ya no se entienden con verdades autoevidentes por sí

mismas, sino que se consideran simplemente como proposiciones de las que se parte para

de mostrar todo lo demás. Casi siempre se trabaja con referencia a un modelo, por lo que

hay que tener sumo cuidado pues, puede haber verdades “evidentes” según el modelo, pero

que necesitan ser demostradas por todos los modelos posibles.

Page 182: A Notas de Algebra Moderna

Ahora bien, cuando se trabaja con pocos axiomas loa posible modelos son generalmente

muchos. A medida que se agregan axiomas el número de modelos va disminuyendo hasta

que se reduce básicamente a uno. En este caso se han logrado definir los elementos de

dicho modelo, de manera implícita, pues no se ha dicho que son, cual es su naturaleza, sino

que se han dado ciertas propiedades entre ellos que en fin de cuentas los caracterizan. Esto

es lo que haremos a continuación para captar el concepto de número entero

axiomáticamente. Empezaremos mencionando los axiomas para dominios enteros que se

refieren a las operaciones de suma y producto (+,·), pero, para este sistema hay muchos

modelos: los enteros, los polinomios, las clases residuales, los números racionales, los

reales, etc.

Entonces introduciremos unos axiomas que se refieren al orden hasta que básicamente no

encontramos sino un modelo: Los números enteros.

Esperamos que esta introducción sirva al lector para que no se extrañe al demostrar

cuestiones que para él siempre han sido evidentes como que el cero multiplicado por

cualquier número siempre es cero, o que el uno (1) es el primer entero positivo.

Demostraremos precisamente todas estas propiedades básicas de los enteros.

Page 183: A Notas de Algebra Moderna

A2. AXIOMAS DE LOS ENTEROS

No definiremos que es un número entero, ni la suma ni la multiplicación entre ellos.

simplemente diremos que los enteros están dotados de estas operaciones (+,·) las cuales

caracterizamos por cumplir los axiomas. Además aceptamos la existencia de dos enteros 0

y 1 (diferentes) de manera que se cumple:

AXIOMAS ALGEBRAICOS

Siendo a, b, c, d enteros cualesquiera se cumple:

1. CLAUSURA: a=c y b=d ⇒ (a+b)=(c+d)

2. ASOCIATIVIDAD: (a+b)+c=a+(b+c)

(a·b)·c=a·(b·c)

3. MODULATIVA: a+o=a y a·1=a

4. INVERSOS ADITIVOS: Existe (-a) tal que a+(-a)=0

5. CONMUTATIVA: a+b=b+a)

a·b=b·a

6. DISTRIBUTIVA: a·(b+c)=a·b+a·c

7. CANCELATIVA PARA EL PRODUCTO: Si a≠0 a·b=a.c⇒b=c

Estos axiomas los cumplen los enteros, como es fácil ver; pero hay otros objetos, sistemas,

que también los cumplen. Cuando un conjunto (no importa la naturaleza de sus elementos)

dotado de dos operaciones (no importa lo que ellos signifiquen) cumplen los axiomas

algebraicos anteriores se llama DOMINIO ENTERO o un dominio de integridad (Integral

Domain). A continuación mostraremos algunos ejemplos:

Page 184: A Notas de Algebra Moderna

Ejemplo 1. El dominio entero más pequeño en el que se puede pensar debe tener por lo

menos dos elementos: el uno y el cero. Sea A={0,1} y definamos las operaciones + y ·

como se indican en los cuadros:

Entonces se verifican todos los axiomas. La clausura se garantiza por la manera como se

han definido las operaciones. La conmutativa para + se ve haciendo tadas las

combinaciones posibles, se ve que

0+1=1=1+0

1+1=0=1+1

0+0=0=0+0

Así se prueba todos los axiomas

Ejemplo 2. Sea A={a,b,c,d} y se definen las operaciones así

En este caso como en el anterior, hay que demostrar cada propiedad observando la

posibilidad. Vemos que el papel del 0 es jugado por a mientras que b hace el papel del 1,

pues son módulos para el producto y la suma, respectivamente. Se ve, pues que se cumplen

todas las propiedades a excepción de la cancelativa para el producto. En efecto la

cancelativa exige que si x≠0 para todo y, z se tiene que xy=yz debe obligar y=z. En nuestro

caso c≠a (hace el papel de 0) y se tiene cb=cd sin embargo b≠d.

Ejemplo 3. Los números reales R y los racionale Q son como los enteros, dominios

enteros. Es más, R y Q tienen propiedades que no tiene Z (¿cuáles?).

+ 0 1 0 0 1 1 1 0

· 0 1 0 0 0 1 0 1

+ a b c d a a b c d b b c d a c c d a b d d a b c

· a b c d a a a a a b a b c d c a c a c d a d c b

Page 185: A Notas de Algebra Moderna

Ejemplo 4. Sea A el conjunto de polinomios con coeficientes reales, con la suma y

multiplicación corrientes. A forma un dominio entero. El papel de 0 y 1 lo hacen los

polinomios constantes de valores 0 y 1 respectivamente.

PROPIEDADES

Proposición 1. El "0" es único como módulo de la suma.

Demostración: Supongamos que existe otro elemento, llamémoslo 0' que se comporta

como módulo, o sea para todo x∈Z se cumple que:

x+0'=x.

En especial para 0 y aplicando conmutativa se tendrá

0'+0=0. (1)

y como 0 es módulo se tendrá

0'+0=0'.

que en combinación con (1) por clausura, obliga

0=0'. g

Proposición 2. El "1" es único como módulo del producto.

Demostración: (Ejercicio)

Proposición 3. El universo aditivo es único.

Demostración: Supongamos que (-a) y a' son inversos aditivos de a por lo tanto se deben

cumplir:

(-a)+a=0 (1)

a'+a=0 (2)

De (2) por clausura:

(a'+a)+(-a)=0+(-a)

Page 186: A Notas de Algebra Moderna

Como 0 es módulo y asociando a la izquierda tenemos:

a'+(a+(-a))=(-a)

Pero: (a+(-a))=0

Entonces: a'=-a.

Proposición 4. a·0=0

Demostración: Por modulativa 0+0=0, y multiplicando por a La clausura asegura que

a(0+0)=a·0

Vemos que a·0 está actuando como módulo aditivo por la Proposición 1 se tiene que a·0=0

Proposición 5. (-1)a=-a

Demostración: La idea es ver que (-1)a es el inverso aditivo de a. Calculemos pues

(-1)a+a por clausura como 1a=a tenemos:

(-1)a+a=((-1)+1)a.

Pero

(-1)+1=1+(-1)=0

Por lo tanto

(-1)a+a=0· a

Y la proposición 4 nos asegura que

(-1)a+a=0 (1)

lo cual por proposición 3 nos indica que

-a=(-1)a

Pues (-1)a según (1) actúa como módulo.

Proposición 6. -(-a)=a.

Page 187: A Notas de Algebra Moderna

Demostración: Se trata de ver, apoyándonos de nuevo en la proposición 3, que a actúa

como inverso aditivo para (-a) y esto es claro pues

a+(-a)=(-a)+a=0

Proposición 7. (-1)·(-1)=1

Demostración: (Ejercicio)

Proposición 8. (-a)(-a)=a·a

Demostración: (Ejercicio)

Proposición 9. La ecuación x+b=c, cualesquiera que sean b y c, tiene una única solución:

Demostración : Si hacemos

x=c+(-b)

y reemplazando en la ecuación vemos que este valor la satisface. Esto asegura que existe al

menos una solución.

Veamos que ésta es la única. Supongamos otra, o sea x' se cumple también:

x'+ b=c

Entonces por clausura:

(x'+b)+(-b)=c+(-b)

asociando en el miembro de la izquierda:

x'+(b+(-b)=c+(-b)

Pero como b+(-b)=0 aplicando modulativa se obtiene

x'=c+(-b)

Page 188: A Notas de Algebra Moderna

o sea x'=x lo que indica que la solución es única.

NOTACION: Se notará a-b al elemento a+(-b).

Proposición 10. En un dominio entero no hay divisores de cero, es decir, si ab=0 entonces

se debe tener que, o bien a=0, o bien b=0.

Demostración: Esto es una consecuencia del axioma 7, o sea la ley cancelativa del

producto. En efecto, supongamos que a·b=0 y a≠0. Por lo tanto se tiene

a·b=a·0

Aplicando el axioma debemos tener que

b=0

De la misma forma vemos que si a·b=0 y b=0 entonces a≠0.

Se ve entonces que siempre que un producto sea nulo, uno de sus factores debe ser nulo.

EJERCICIOS

1. Sea A={0,1,2} indicar si los axiomas de dominio entero se cumplen para las

operaciones definidas como indican las tablas

a)

b)

¿Quién hace el papel de 1?

+ 0 1 2 0 0 1 2 1 1 2 0 2 2 0 1

· 0 1 2 0 0 0 0 1 0 1 2 2 0 2 2

+ 0 1 2 0 0 1 2 1 1 2 0 2 2 2 0

· 0 1 2 0 0 0 0 1 0 1 2 2 0 2 2

Page 189: A Notas de Algebra Moderna

c)

2. Sea x un conjunto y A el conjunto de todos los subconjuntos de x. Entonces la ∪ e ∩ ;

son operaciones definidas en A. Entendiendo la ∪ como la suma y la ∩ como

producto, ¿Cuáles axiomas de dominio entero se cumplen?

3. Complete la tabla de multiplicación para que el conjunto A={0,1,2,3,4} forme un

dominio entero

NOTA: 4 juega el papel de módulo para el producto. Para averiguar, por ejemplo,

cuanto es 2·3 se sabe que 4+4 = 3 entonces

2·3=2(4+4)=2·4+2·4=2+2+4

4. Demuestre la Proposición 2.

5. Demuestre la Proposición 7.

6. Basándose en la Proposición 7 demostrar 8.

7. Demuestre que en un dominio entero si a, b, c, d son elementos cualesquiera, se

cumplen las siguientes igualdades (justifique cada paso):

a) a·(b-c)=a·b-c·d

b) (a-c)+(c-d)=(a-d)

+ 0 1 2 0 0 1 2 1 1 2 0 2 2 0 1

· 0 1 2 0 0 1 2 1 1 1 2 2 2 2 2

+ 0 1 2 3 4 0 0 1 2 3 4 1 1 2 3 4 0 2 2 3 4 0 1 3 3 4 0 1 2 4 4 0 1 2 3

+ 0 1 2 3 4 0 0 0 0 0 0 1 0 2 0 3 0 4 0 1 2 3 4

Page 190: A Notas de Algebra Moderna

c) (a+b)·(a+c)=a·a+(b+c)·a+b·c

d) (a-b)

e) (a-b)·(a+b)=(a·a)-(b·b)

f) (a-b)·(c-d)=(a.c+b.d)-(b.c+a.d)

8. Demuestre que si en un dominio entero x+a = x+b entonces a = b (cancelativa suma).

9. Se define a2=a·a demuestre que (a+b)2=a2+2ab+b2 y que (a-b)2.

10. Diga dónde está el error de la siguiente demostración:

"Como 6=6 y 2=2 puedo decir que 6+2 = 6+2 lo cual ímplica que 6-6 = 2-2 y se tiene

3(2-2)=1(2-2) de donde 3=1"

11. En Z se define:

a⊕b=a+b-1 a⊗b= a.b - (a + b) + 2

Demostrar que (Z, ⊕, ⊗) también forma un dominio entero en donde el módulo para +

es 1. ¿Cuál es el módulo para el producto?

12. Demostrar que los números reales de la forma a+b 2 de donde a, b son enteros

forman un dominio entero.

13. Entre las matrices 2X2 con coeficientes reales está definida la suma y la multiplicación,

sin embargo, no se forma un dominio entero. (¿Porqué?)

14. Demuestre que las matrices de la forma ⎟⎟⎠

⎞⎜⎜⎝

⎛ −abba

reales sí forman un dominio entero.

Page 191: A Notas de Algebra Moderna
Page 192: A Notas de Algebra Moderna

A4. EL PRINCIPIO DEL BUEN ORDEN

Como vimos en la sección anterior Z es un dominio entero ordenado, pero no es único, pues

existen otros radicalmente diferentes. En esta sección introducimos un nuevo axioma

llamado "Principio de Buen Orden" y desarrollamos sus consecuencias.

Este axioma caracteriza a los enteros en cuanto el único dominio entero ordenado que lo

cumple es Z (salvo notación). No podemos mostrar ejemplos radicalmente distintos, pues

como se puede demostrar (1) básicamente no hay sino un ejemplo. Otro problema es saber

si con estos axiomas podemos demostrar cualquier proposición que sea cierta en Z. La

sorprendente respuesta a este problema fué dada por G del alrededor de 1930 y se la

dejamos como inquietud al lector. Antes de enunciar el citado axioma, damos otra

definición que corresponde a la intuición.

Definición 1. Sea A un subconjunto de un dominio entero ordenado. Se dice que a es el

primer elemento de A. sí y sólo sí, a∈A y a< x para todo x∈A.

Es claro que si un conjunto tiene primer elemento, es único. Por otra parte hay

subconjuntos no vacíos de un dominio entero que no poseen primer elemento, considérese

por ejemplo el conjunto de números pares como subconjunto de Z.

Por las razones explicadas anteriormente, habiendo aceptado este axioma sólo nos

referíamos a Z; tenemos pues que todo subconjunto no vacío de enteros positivos tiene un

primer elemento, en especial los enteros positivos deben tener un primer elemento.

Proposición 1. El primer elemento de los enteros positivos es 1.

AXIOMA DEL BUEN ORDEN:

Todo subconjunto no vacío de positivos tiene un

primer elemento.

Page 193: A Notas de Algebra Moderna

Demostración. Supongamos que existe un elemento c positivo que está antes que 1, a sea

que:

0<c<1

Consideremos el conjunto cuyos elementos estén entre 0 y 1. Llamémoslo A. Entonces:

A es un subconjunto de enteros positivos.

A es no vacío, pues hemos supuesto que c∈A.

Por el principio de buen orden A debe poseer un primer elemento, llamémoslo a. Como

a∈A entonces:

0<a<1

Aplicando propiedades de orden (ejercicio 3f. de la sección anterior) tenemos:

0<a2<a

Luego aparece otro elemento, a2, que pertenece a A y es menor que a, lo cual es una

contradicción pues a es el primer elemento de A.

Concluimos entonces que A debe ser vacío, o lo que es equivalente, que no hay enteros

menores que 1.

Corolario. Si a es entero, entonces entre a y a+1 no hay ningún entero.

Demostración. Supongamos que hay un entero c que está entre a y a+1. Tenemos:

a<c<a+1

Sumando (-a) a estas dos desigualdades nos queda

0<c-a<1

Y como c-a es un entero, se está contradiciendo la proposición 1 nos indica que tal c no

existe.

Proposición 2. (Principio de inducción) Sea A un subconjunto de enteros positivos tal

que:

i) 1∈A

Page 194: A Notas de Algebra Moderna

ii) Si x∈A⇒(x+1)∈A

En estas condiciones A coincide con los enteros positivos.

Demostración. Sea B el conjunto de enteros positivos que no están en A.

Si B es no vacío, por el principio de buen orden debe tener un primer elemento llamémoslo

b. ¿Qué ocurre con b-1? Es entero y positivo pues b=1 ya que 1∈A; entonces b-1 puede

pertenecer o no a A. Analicemos ambos casos:

a. Si b-1 es elemento de A por la hipótesis ii) se tiene que (b-1)+1∈A o sea que b∈A lo

cual es imposible pues b∈B.

b. Si b-1 no es elemento de A pertenece a B, pero b-1< b entonces b no sería el primer

elemento de B, contrario a lo supuesto.

Ambas proposiciones nos lleva a negar la existencia de b y la única posibilidad es que B sea

vacío o sea que A comprende todos los enteros positivos.

Corolario. Sea A un conjunto y a un entero tal que:

i) a∈A

ii) Si k∈A se puede asegurar que a+1∈A entonces A comprende todos los enteros mayores

que a.

Demostración. Sea B={x-a+1| x∈A} demuéstrese por inducción que N⊆B. Sea y entero

tal que y>a entonces y-a+1∈N, lo que implica que (y-a+1)∈B o sea y∈A.

Otra forma del principio de inducción se da en el ejercicio 4. El principio de inducción

también sirve para definir funciones evaluadas en números enteros a partir de k:

Proposición 3. (Definiciones Recursivas) Si se conoce el valor de f(k) y si conociendo el

valor de f(n) se puede conocer el valor de f(n+1), entonces se conocen los valores de f para

todo entero mayor o igual a k.

Page 195: A Notas de Algebra Moderna

Demostración. (Ejercicio)

El lector debe estar familiarizado tanto en las demostraciones por inducción como con las

definiciones recursivas; aquí ilustraremos estas aplicaciones con la demostración de un

hecho muy sutil con el cual estamos tan familiarizados que no se ve generalmente la

necesidad de su demostración. El axioma de asociatividad que se dió para dominios enteros

se enuncia para tres ejemplos e intuitivamente se dá uno cuanto que es válida para n

elementos o sea que en una suma de n elementos se pueden eliminar los paréntesis (por esta

razón, se puede hablar de "una suma de n elementos" no así de "una resta de n elementos").

Demostraremos formalmente este hecho. Primero definíamos lo que sería

(...((a1+a2)+a3)+...)+an-1+an

para a1, a2,..., an-1, an elementos de un dominio entero.

Definición. Sean a1, a2,..., an,... elementos de un dominio entero. Se define:

i) S1=a1

ii) Si+1=Si+ai+1

Así mismo definimos S -r- para k > r -r+1-

i) 1+rrS =ar+1

ii) Sr= 1+krS +ak+1

Nota. Obsérvese que intuitivamente

Si = (...(a1+a2)+a3)+...+ai-1)+ai

y krS = (...((ar+1+ar+2)+...+ak-1)+ak con (k>r)

Si se define por inducción sobre i y Sk por inducción sobre k.

Page 196: A Notas de Algebra Moderna

Proposición 4. (Ley Asociativa Generalizada) En las condiciones de la definición anterior

se tiene que para todo n≥3 y para todo k<n, k y n enteros positivos se cumple: -n-

Sk+ nrS =Sn

Demostración. Haremos inducción sobre n:

i) Para n=3 tenemos:

S1=a1; S2=a1+a2; S3=(a1+a2)+a3

Si k=1: 21S =a2 y 3

1S =a2+a3

Sk+ nkS =S1+S1=a1+(a2+a3)

Y aplicando asociativa al último término de ésta ecuación vemos que

Sk+ nkS =(a1+a2)+a3

ó sea

Sk+ nkS =Sn

Si k=2 compruébelo usted mismo. Es mas fácil!

ii) Supongamos ahora que el teorema es válido para n es decir que para 0<k<n se cumple

Sk+ nkS =Sn

Pero 1+npS = n

pS +an+1 por definición y así mismo

Sn+1=Sn+an+1

entonces

Sp+ 1+npS = n

pS +( npS +an+1)=(Sp+ n

pS )+an+1

Aplicando la hipótesis de inducción al último miembro de la igualdad vemos que:

Sp+ 1+npS =Sn+an+1

Lo que implica que

Sp+ 1+npS =Sn+1

Completando la demostración.

Page 197: A Notas de Algebra Moderna

EJERCICIOS

1. Definir Ultimo Elemento de A para A un subconjunto de un dominio entero ordenado.

2. Mostrar un subconjunto de los reales positivos, no vacío, que no tenga primer elemento.

3. Usando el principio del buen orden probar que si k>0 y a es cualquier entero, entonces

existe un n∈Z tal que nk<a<(n+1)k.

4. Use el hecho de que Z está bien ordenado para probar que la siguiente forma del

principio de inducción es correcta: "Supongamos que a cada entero positivo n está

asociado una proposición Pn Entonces Pn es cierta para cada entero positivo n si

cumple:

i) P1 es cierta

ii) Si k es un entero positivo tal que Pi es cierta para todo i<k, entonces Pk también es

cierta". Esta forma del principio de inducción se llama "Principio de Inducción

completa"

5. Completar la demostración del corolario de la proposición 2.

6. Demuestre la proposición 3.

7. Enuncie y demuestre la ley distributiva generalizada en un dominio entero.

Page 198: A Notas de Algebra Moderna

11. NÚMEROS, NUMERALES Y SISTEMAS DE NUMERACIÓN

En el hombre de todas las épocas incluso en los más primitivos encontramos una facultad

que podemos llamar el "sentido del número": facultad que le permite reconocer de alguna

manera si de su pequeña colección de objetos se ha substraido o añadido alguno. Este

sentido del número no debe ser confundido con la facultad de contar, que es probablemente

mucho más reciente y que implica un proceso mucho más complicado.

El origen del concepto del número está escondido tras del impenetrable velo de las

incontables y remotas edades prehistóricas. Nos podemos dar una idea de cómo sucedió

este desarrollo observando el estado mental de algunas tribus salvajes actuales

correlacionándolo con el análisis socioantropológico de los diferentes estados del hombre

primitivo y con hallazgos de vestigios de numeración escrita.

Podemos partir de un sentido rudimentario del número, de alcance no mayor del que tienen

algunos pájaros y suponer que éste fue el núcleo del que surgió nuestra idea del número.

Sin duda alguna esta tan reducida capacidad mental hubiera llevado al hombre a un

estancamiento total y no hubiera evolucionado más de lo que lo hacen o han hecho dichos

animales. Pero a través de los siglos, las circunstancias economicosociales en las diferentes

culturas hace que el hombre poco a poco vaya cambiando su concepción tan limitada del

número. Muchos pueblos primitivos registran el número de sus rebaños, o de sus soldados,

mediante inscripciones hechas en un trozo de madera o por medio de piedras puestas en una

fila; entonces este hombre no hace otra cosa que comparar dos conjuntos pero es incapaz de

crear el número como signo o concepto .

Pasa el tiempo y el hombre encuentra signos, modelos para comparar en el medio que lo

rodea: Las alas de un ave, para representar el dos, las patas de un animal para el cuatro, los

dedos de la mano para representar el cinco, etc., y así poco a poco mientras avanza la

Page 199: A Notas de Algebra Moderna

interminable evolución, este hombre va empleando mejor, enriqueciendo su lenguaje y

reemplaza los sonidos por imágenes y los modelos originales toman la forma abstracta de

los nombres de los números.

Más tarde los modelos se organizan y se disponen en una sucesión que progresa en

magnitudes crecientes, es decir una sucesión natural: uno, dos, tres,....

Así, una vez creado este sistema, contar era entonces asignarle a cada elemento de un

conjunto dado un término de ésta sucesión hasta que la colección se agota y al último

elemento se le asignaría un término el cual nos daba el dato de la cantidad de objetos de la

colección.

Vemos que el apareamiento por sí solo es incapaz de crear un arte de calcular. Sin nuestra

facultad para disponer los objetos en sucesión ordenada, pocos progresos podían haberse

obtenido. Distingamos entonces el concepto abstracto de Número, del concepto de

numeral, que es el signo (sonoro y/o escrito) tal que le corresponde un número, y de un

sistema de numeración, que es el esquema sistemático para nombrar los números.

Así nuestro sistema de numeración es el hindoArábigo, sistema posicional en base diez, que

nació en la India hace unos 2500 años y que fue llevado a Europa por los árabes, dándose a

conocer en los siglos XIII y XIV e imponiéndose definitivamente en la cultura occidental

con el desarrollo de la cultura capitalista. Nuestro interés es profundizar en éste sistema de

numeración y en otros sistemas posicionales con bases diferentes y con éste fin

describiremos algunos sistemas de numeración antiguos.

Un sistema primitivo de cuentas usaba solamente un símbolo: una marca o una raya el cual

era repetido de acuerdo a la necesidad. Pero para representar un número muy grande

resultaba inconveniente pues demandaba el uso de tantas marcas como fuera la magnitud

del número a representar, lo cual limitaba el uso del sistema. Si por ejemplo queríamos

sumar dos cantidades cualesquiera tendríamos:

Page 200: A Notas de Algebra Moderna

/ / / / / / / / / / / / / / / + / / / / / / / / / / / / / / / = / / / / / / / / / / / / / / / / / / / / / / / / / / / / / /

nos damos cuenta de la dificultad que representa adicionar cantidades que no sean muy

pequeñas. Un sistema de cuentas más refinado surgió de la agrupación de marcas en fardos

del mismo tamaño. Así por ejemplo la suma anterior quedaría:

Entonces cuando se trataba de contar un rebaño de ovejas se procedía a hacerlo en grupos

de a tres y resultaba más práctico que contarlas una a una. Así a cada grupo de éstos se le

asignaba un nuevo símbolo repitiéndose éste cuantas veces fuera necesario. Luego se

creaba un nuevo símbolo que representase un número determinado de los símbolos

anteriores y así repitiendo este proceso de creación de nuevos símbolos como repetición de

símbolos anteriores según las necesidades prácticas del conteo se obtiene un sistema de

numeración que hoy en día denominamos aditivo. Un ejemplo muy ilustrativo de un

sistema de numeración aditivo es el sistema egipcio.

El sistema numérico aditivo que adoptó la civilización egipcia (años 30000-700 A.d.C.)

tenía como primeros nueve numerales los siguientes:

/ / / / / / / / / / / / /

/, / /, / / /, / /, / / /, / / /, / / /, / / /, / / /

/ /, / / /, / / /

El arreglo de las marcas variaba y usualmente se disponía geométricamente; por ejemplo,

un grupo de tres por tres representaba el nueve. La huella talón fue el nuevo símbolo que

representó diez cuentas y se escribía como una ∪ invertida ∩ . Por ejemplo (combinando

la moderna ecuación con el antiguo sistema de numeración egipcio), tendríamos:

///////)(////////////

/////////

∩∩ =+=++=+

Page 201: A Notas de Algebra Moderna

La tabla 1 reproduce los numerales básicos de la numeración egipcia. Algunos ejemplos

con su numeral correspondiente en sistema hindoArábigo, son los siguientes:

273 =

1983 =

1'240.350 =

Numerales egipcios Numerales hindoarabigos

(Unidad) 1

Huella del talón 10

Lazo de cuerda 100

Flor de loto 1.000

Extremo de un dedo 10.000

Mustela 100.000

Hombre asombrado 1'000.000

Tabla 1. Numerales básicos según el sistema de numeración egipcio.

Page 202: A Notas de Algebra Moderna

Es importante notar que éste sistema, como todos los sistemas de numeración aditivos no

considera el cero y además cada signo no representa sino un único número no importa que

lugar ocupe en la expresión numérica.

Un sistema considerablemente más sofisticado que el egipcio fue el sistema griego que

utilizaba 27 símbolos (del alfabeto, la mayoría) como se muestra en la tabla 2.

Todo número menor que mil podía expresarse de manera aditiva por la combinación de a lo

más tres de los símbolos básicos. Por ejemplo para escribir 273 los griegos utilizarían la

siguiente combinación: Hasta aquí, el sistema griego es netamente aditivo con mucha

economía en cuanto a el número de símbolos que utiliza para representar cada número

determinado pero que a cambio exige conocer más símbolos para poder expresar pocos

números. Sin embargo para números mayores que mil su numeración tenía rasgos

posicionales ya que los múltiplos de mil se expresaban colocando un apóstrofe ' al número

correspondiente. Así 3000 se expresaba colocando un apóstrofe ' a la derecha (tres) para

obtener. De la misma manera para expresar 3273 se escribiría. Cuando se quería expresar

múltiplos de 10.000 se colocaba el número correspondiente encima de una M. Así el

número 99.909 se escribía:

Este sistema tiene la propiedad de que ciertos símbolos ligeramente modificados o en

determinadas posiciones cambian de valor, sin embargo para efectos prácticos tenían

Arábigo Griego Arábigo Griego Arábigo Griego 1 α 10 ι 100 ρ 2 β 20 κ 200 σ3 γ 30 λ 300 τ4 δ 40 µ 400 υ5 ε 50 ν 500 ϕ6 ς 60 ξ 600 χ 7 ζ 70 υ 700 ψ 8 η 80 π 800 ω9 θ 90 ο 900 ∏

Tabla 2. Numerales básicos griegos.

Page 203: A Notas de Algebra Moderna

desventajas con el mismo sistema egipcio (figura 2). De todas maneras ante la ausencia

del cero no podemos considerar el sistema griego como posicional.

Los sistemas posicionales se caracterizan por incluir un símbolo para el cero y por cada

símbolo tiene un valor de acuerdo a la posición que ocupe en la expresión numérica.

Además de la numeración hindoarábiga son posicionales los sistemas de numeración de la

antigua Babilonia así como el sistema que utilizó la civilización Maya de la América

precolombina.

PREGUNTAS Y EJERCICIOS.

1. ¿Qué tuvieron que ver las siguientes actividades con el desarrollo de la idea de número

y los numerales: la agricultura, la ganadería, el comercio, la moneda y la religión.?

2. Escriba en numerales egipcios los siguientes números:

a. 220 b. 746 c. 1342 d. 12 346

3. Escribir los numerales actuales de los siguientes números escritos con numerales

egipcios:

HOY 34042154125

(Egipcios)

(Griegos) ρκε + δσι∈ = ?

Figura 2. Tres formas de efectuar la misma suma. Nótese que el sistema griego es el

menos práctico.

Page 204: A Notas de Algebra Moderna

a.

b.

c.

d.

4. ¿Cuál es el número entre 1 y 1000 que para ser escrito en numerales egipcios necesita

más caracteres?

5. ¿Cuáles son los números que escritos en numerales egipcios utilizan exactamente seis

caracteres?

6. Escribir en numerales griegos los números del ejercicio 2.

7. ¿Cuáles son los números entre 100 y 1000 que se escriben en numerales egipcios con

un sólo carácter una o varias veces repetido?

8. Escribir en numerales actuales los siguientes números presentados en numerales

griegos.

a. χµδ b. γβ c. γπε d. σλβαυξη

9. Describir el sistema de numeración romano.

10. Escriba en numerales romanos los numerales del ejercicio 2.

11. La tabla 3 muestra los numerales básicos de la numeración chino-japonesa. Para formar

otros números, por ejemplo el 60 se escribe (seis dieces) o

215 = + (doscientos más un diez, más un cinco).

Page 205: A Notas de Algebra Moderna

a. Escribir los números del ejercicio 2 en numerales chino japoneses.

b. ¿Cuántos numerales a lo más se necesitan para representar un número menor que

1000?

c. Hasta cuántas veces pueden repetirse un carácter en esta numeración cuando se

escribe un número de 1 a 1000?

12. Describir las características más deseables y menos deseables de un sistema de

numeración.

Numeración actual

Numeración China-japonés

Numeración actual

Numeración China-japonés

1

7

2

8

3

9

4 10

5 100

6 1000

Tabla 3. Numerales chino-japonés.

Page 206: A Notas de Algebra Moderna

13. Hacer un análisis comparativo de los cinco sistemas de numeración: El sistema de

cuentas, la numeración egipcia, el sistema griego, la numeración romana y la chino

japonesa.

Page 207: A Notas de Algebra Moderna

12. NUMERACIONES POSICIONALES.

Los sistemas de numeración utilizados por las antiguas civilizaciones: babilonia y maya son

cada uno sistemas posicionales, el babilonio en base 60 y el maya en base 20, que utilizan

para representar los números desde 1 hasta la base de un sistema aditivo, y para números

mayores que la base por repetición de los primeros, en donde cada uno de éstos vale según

la posición que ocupe. Además, como ya lo anotamos, los sistemas posiciones consideran

de una u otra forma, lo que nosotros hoy en día llamamos el cero. Por ejemplo, en la

numeración maya los números de uno a veinte (sin incluirlo) son notados por un sistema

aditivo en donde las unidades son demarcadas con puntos o círculos llenos y cinco (5)

unidades de demarcan por una raya horizontal (fig. 1). Los mayores que 19, desde 20 en

adelante se representan por combinaciones de números menores que éste y el cero que es

una figura que se ha interpretado como una concha o un fríjol.

Estos dos sistemas (el maya y el babilonio) utilizan pocos caracteres diferentes, lo que hace

que para números grandes sea necesario muchos de éstos. Esta es una desventaja que les

quita practicidad a pesar de que son muy efectivos, como todo sistema posicional, en

cuanto a la realización de la operaciones básicas.

El otro sistema posicional es el sistema decimal hindoarábigo que hoy en día representa el

lenguaje más universal que el hombre ha adquirido. Esta numeración, también de origen

60 63 215 871

3 12 45

Figura 1. Algunos numerales mayas.

Page 208: A Notas de Algebra Moderna

muy remoto se calcula nació en la India hace unos 2. 500 años. En los manuscritos

budistas de Asaka del siglo III a. C. hallamos los símbolos 1,4,6; cien años después

hallamos los símbolos 2,4,5,6,7 y 9 grabados en los monumentos de Nona Ghat y en el

siglo II D. C. , periodo de las cuevas de Nasik, hallamos todas las cifras. El cero empezó

entre los indostánicos como un puntico o circulito y el sánscrito lo designaba con la

expresión "vacante" o "vacío".

La primera referencia concreta a la numeración indostánica fuera de las fronteras de la

India se halla en la nota escrita por un sacerdote mesopotámico, Severus Sebokth, hacia 650

D. C. quien hablaba de nueve signos sin mencionar el cero. A fines del siglo VIII habían

sido trasladadas a Bagdad unas tablas astronómicas, y los rabees doctos de la época

conocieron estos signos. En el año 925 el estudioso Al-Khwarizmi escribió un librito sobre

números el cual 300 años después fu‚ traducido en latín por Abelardo de Bath. El

desarrollo del comercio, del sistema de intereses y en general la necesidad de efectuar las

operaciones básicas de manera más rápida hace que este sistema se imponga

definitivamente con el correr de los siglos. Muy ilustrativo es el cuadro de la figura 2

tomado de un libro muy popular en el siglo XVI (impreso por primera vez en 1503) y que

muestra la disputa entre un abaquista (numeración romana) y un calculista de pluma y tinta

(sistema induarábigo). Repárese en la angustia del abaquista contra la seguridad del

calculista al tratar ambos de dividir 1234 entre 97.

Nuestro interés girar en lo que sigue hacia la profundización en los sistemas de numeración

posicional en diferentes bases con miras a explicar el por qué‚ de los algoritmos que

utilizamos en las operaciones básicas y ciertos hechos que se pueden observar en la

aritmética.

Por ahora pensemos en la siguiente máquina de contar: consideremos tres relojes cada uno

con los números del cero al nueve (fig. 3), que hallaremos Dial I, Dial II y Dial III. Este

sistema funciona como los contadores del kilometraje de los automóviles, si comenzamos a

dar vueltas al dial I en el sentido de las mancillas del reloj y contamos 1,2,. . . . , etc. de

tal manera que cuando vaya a pasar de nueve al cero, el dial II se corre un lugar y éste a su

Page 209: A Notas de Algebra Moderna

vez avanza un lugar al dial III, cuando pase del nueve al cero entonces el dial I determinar

las unidades, cada avance del dial II supone diez unidades que ha recorrido el dial I.

Así mismo el dial III se mueve un lugar cuando el dial II complete una vuelta de diez

movimientos o sea cuando el dial I se haya movido cien veces.

Así en la figura 3 el número representado es 520, 500 por el dial III, 20 por el dial II y cero

por el dial I. Nótese que podemos agregar otros diales si queremos expresar números más

grandes, por ejemplo el sexto dial indicaría "cien miles" mientras un último dial marcaría

los millones.

Cada dial se referirá a diferentes potencias Fig. 3: Diales para presentar números en el

sistema posicionales de 10. Por esto un hindoarábigo.

Figura 2. Grabado del "Margarita Philosofica" de Greisch (1. 503).

Page 210: A Notas de Algebra Moderna

Nuestro sistema de numeración se dice que es en base 10, pero que se trabaje con 10 dígitos

(0,1,2,. . . ,9) es algo arbitrario (tal vez porque tenemos 10 dedos en las manos) pues se

hubiera podido utilizar 3, 5, ó 12 dígitos (señales en los diales).

Supongamos entonces que los diales no tienen sino tres señales: 0,1,2. En la figura 4 se

muestran como funcionarían los tres diales en los primeros 11 movimientos.

Los primeros tres números cero, uno y dos se representan igual que en nuestro sistema,

pero en el tercer movimiento el dial! pasa a 0 y el dial II avanza un lugar quedando en 1,

por eso el número que nosotros notamos 3 será notado 0l0.

El dial II indicar entonces los múltiplos de 3. Así mismo podemos observar que el dial III

representar los múltiplos de 9. Entonces el número escrito en base 3 como 211 equivale al

número que en base 10 escribíamos como 22, ya que se tendrían 2 nueves, 1 tres y una

unidad. Esta igualdad se expresar así: (211)3=(22)10

Aquí también podemos agregar cuantos diales queramos, y naturalmente en base 3,

necesitaremos más dígitos que en base 10 para expresar un numero determinado. Por

ejemplo:

(21102)3=2×34+1×33+1×32+0×3+2=(200)10.

Así como hemos hecho con tres, se puede utilizar cualquier base mayor que 1 para

representar un entero positivo dado. Cuando se quiere utilizar una base mayor que la

nuestra es necesario introducir nuevos dígitos. Por ejemplo, cuando se quiere trabajar en

0 9 1

8 2 7 3

6 4 5

0 9 1

8 2 7 3

6 4 5

0 9 1

8 2 7 3 6 4

5

Figura 3. Diales para presentar números en el sistema posicional indoarábigo.

Page 211: A Notas de Algebra Moderna

base 12 podemos usar como dígitos: 1,2,...,9,A,B en donde A y B representan el 10 y el 11

de nuestra base decimal. Así: (3A)12=3×12+a=36+10=(46)10. Además.

0

2 1 0

2 1 0

2 1

0

2 1 0

2 1 0

2 1

0

2 1 0

2 1 0

2 1

0

2 1 0

2 1 0

2 1

0

2 1 0

2 1 0

2 1

0

2 1 0

2 1 0

2 1

0

2 1 0

2 1 0

2 1

0

2 1 0

2 1 0

2 1

0

2 1 0

2 1 0

2 1

0

2 1 0

2 1 0

2 1

0

2 1 0

2 1 0

2 1

0

2 1 0

2 1 0

2 1

Page 212: A Notas de Algebra Moderna

Es de particular importancia resaltar que el sistema en base 2 también llamado sistema

binario y que utiliza los símbolos 0 y 1, tiene en nuestra ‚poca gran figuración pues es

utilizado por las modernas computadoras y por tanto más adelante profundizaremos sobre

este tema.

A continuación formalizaremos el resultado básico en que se basa toda la discusión, es

decir, que cualquier numero entero positivo se puede expresar en cualquier base mayor que

uno.

Proposición 1. (Teorema fundamental de la numeración) Sea b un entero mayor que 1,

que llamaremos base. Entonces para todo a entero positivo existen an,an-1,..,a1,a0 con cada

ai que cumple 0<a i<b y tales que:

a= a0+a1b+a2b2+...+anbn

Siendo así se nota a=(an an-1 ... a1 a0)b

Demostración. Dividiendo a entre b obtenemos un cociente q0 y un residuo a0 menor que

b y mayor o igual que cero. Si q0 es cero el número a sería expresado con un sólo dígito,

caso contrario dividimos q0 entre b obteniendo como residuo a1 y así sucesivamente

obtenemos:

a = bq0+a0 con 0≤a0<b q0 = bq1+a1 con 0≤a1<b q1 = bq2+a2 con 0≤a2<b

qn-2 = bqn-1+an-1 con 0≤an-1<b qn-1 = bqn+an con 0≤an<b

donde el último cociente qn es cero. Devolviéndonos vemos que:

an = qn-1ban+an-1 = qn-2

b(ban+an-1)+an-2 = qn-3

b(...b(ban+an-1)+...)+ a3)+a2 = q1b(...b(ban+an-1)+...)+a2)+a1 = q0

bnan+bn-1an-1+...+ba1+a0 = a

Page 213: A Notas de Algebra Moderna

Comprobando que los ai cumplen lo exigido, la demostración de que estos números enteros

a0,a1,...,an (llamados dígitos de a) son únicos, la dejamos como ejercicio al lector.

Ejemplo 1. Para pasar un número escrito en una base diferente de 10 a base decimal

tenemos en cuenta la expresión (1) de la proposición anterior así:

(100)3 = 1×32+0×3+0=(9)10

(222)4 = 2×42+2×4+2=(42)

(222)3 = 2×32+2×3+2=(26)

Ejemplo 2. Cuando se utiliza base mayor que la base nuestra, se acostumbra las letras

A,B. C... para expresar lo que nosotros escribimos como 10, 11, 12,... por ejemplo en base

16:

(ACE)16=10×162+6×16+14=(2670)10

pues (A)16=(10)10 y (E)16=(14)10

Ejemplo 3. Para pasar de base decimal a otra diferente aplicamos el algoritmo descrito

en la demostración de la proposición anterior según el conjunto de ecuaciones (2). De esta

forma para determinar el desarrollo decimal de (26)3 dividimos sucesivamente así:

26 3 2 28 3

2 2 3 2 0

dividimos que significan que:

26=3×8+2

8=3×2+2

2=3×0+2

O sea que:

26=3×(3×2+2)+2

es decir:

26=32×2+3×2+2

y tenemos que:

(26)10=(222)3

Page 214: A Notas de Algebra Moderna

Ejemplo 4. Para pasar de base 10 a base mayor que 10 procedemos de igual forma

teniendo en cuenta el significado de las letras A,B,C,... según se explicó en el ejemplo 3.

(1425)10=(?)14

Haciendo las divisiones vemos que:

1425 = 14×101+11

101 = 14×7+3

entonces

1425 = 14(14×7+3)+11

1425 = 142×7+14×3+11

1425 = (73B)14

PREGUNTAS Y EJERCICIOS

1. Enumere las desventajas del sistema de numeración como el maya y el babilonio.

2. ¿Cuántos caracteres se requieren para representar un número en base b?

3. Resuelva:

a. (342)5=( ? )6

b. (241)6=( ? )5

c. (144)10=( ? )12

d. (ABC)16=( ? )10

e. (871)9=( ? )14

f. (6D2)16=( ? )11

4. Para pasar de base 3 a base 9=32 podemos proceder por el siguiente m‚todo ilustrado

para el caso en que nos preguntan (120110)3 =(?)9 tomamos el número de base 3 y

separamos cifras de derecha a izquierda de dos en dos:

Page 215: A Notas de Algebra Moderna

12-01-10

cada pareja la traducimos a base 9 y en su orden estas cifras serán los dígitos del

número en la nueva base:

(12)3=(5)9; (01)3=(1)9; (10)3=(3)9

O sea: (120110)3=(513)9

a. Compruebe con este y otros casos que éste efectivamente es un m‚todo acertado

para pasar de base 3 a base 9.

b. Según esto explique cómo se hace el proceso inverso: pasar números de base 9 a

base 3.

c. Busque un método parecido para pasar numerales de base 2 a base 4 y

numerales de base 2 a base 8.

d. Generalice y demuestre un método para pasar de un número de base b a base bn

y al contrario.

5. Un tendero sólo tiene una pesa de 1 kilo, otra de 2, otra de 4, una de 8, otra de 16 y una

de 32.

a. ¿Cómo hace para pesar 28 kilos, 39?

b. Demuestre que puede pasar cualquier cantidad de kilos de 1 a 63.

6. El siguiente juego puede servir para descretar a sus amigos: se tiene cuatro cartas en

donde están repetidos los números de 1 a 15 así:

I II III IV

1-3-5 2-3-6 4-5 8-9 7-9 7-10 6-7 10-11

11-13 11-14 12-13 12-13 15 15 14-15 14-15

Usted pide a alguien que pida un número secretamente (por ejemplo 11) y sólo le

comunica en qué‚ tarjeta esta escrito (para el caso I, II y IV) y el único que esté escrito

en exactamente esas tarjetas es el 11.

Page 216: A Notas de Algebra Moderna

a. Describa cómo se determina el número pensado conociendo las tarjetas

correspondientes.

b. ¿Por qué‚ esta distribución funciona?

c. Haga un juego parecido con cinco tarjetas y los números de 1 a 32.

Page 217: A Notas de Algebra Moderna

13. CRITERIOS DE DIVISIBILIDAD Y CONGRUENCIAS

Cuando tenemos un número muy grande escrito en base 10 y deseamos saber si es múltiplo

por ejemplo de 9 no necesitamos hacer la división, simplemente sumamos sus cifras y si el

resultado es múltiplo de 9 el número original es múltiplo de 9. Este es un típico criterio de

divisibilidad, que se utiliza desde la escuela primaria. Uno más sencillo es para saber si un

número es par: se mira la última cifra y si ella es par todo el número es par.

La justificación de estos criterios radica en el sistema de numeración que se utiliza y la

demostración de su validez, que haremos aquí as ndonos el la teoría de congruencias, nos

proporciona elementos para formular nuevos criterios de divisibilidad. El siguiente lema,

que es consecuencia inmediata de la definición de congruencia, es la razón por la que

utilizaremos esta teoría para lograr nuestros objetivos.

Lema 1. Un número k es divisible por c si y solo si

k≡0(mod c)

Demostración. (obvio).

Para la lectura de este capítulo además de manejar las congruencias el lector debe estar

familiarizado con los sistemas de numeración posicionales (sección 12). Se aplica

especialmente el Teorema fundamental de la numeración.

1. CRITERIOS DE LA ULTIMA CIFRA

Algunas veces para ver si un número es divisible por otro es suficiente con observar la

última cifra. Esto depende realmente de que el divisor divida la base, como cuando se

trabaja en base 10 y se quiere saber si un entero es divisible por 2 o por 5.

Page 218: A Notas de Algebra Moderna

Proposición 1. Sea a=(anan-1...a1a0)b si b≡0(mod c) entonces a es divisible por c si y solo

si

a0≡0(mod c)

Demostración. Sabemos que

a=a0+a1b+a2b2 + ...+ anbn

y como b≡0(mod c), aplicando aritmética de congruencias se obtiene que a≡a0(mod c), que

en combinación con el lema 1 nos proporciona el resultado deseado.

Nótese que lo que se demuestra es que, en este caso, la última cifra (a0) determina la clase

de congruencia módulo c, a la que pertenece el entero a.

Ejemplo 1. Un número escrito en base 12 es divisible por 4 si termina en 0,4 u 8, pues

estos son los tres dígitos de la base 12 que se dejan dividir por 4.

Ejemplo 2. Como ya se dijo una aplicación de al proposición 1 en base decimal son los

conocidos criterios para saber cuando un entero es divisible por 2 o por 5. Sin embargo

estos criterios no sirven cuando el número est escrito en cualquier base. Para base 15 el

criterio del 2 no es válido aunque el del 5 casi. Por qué?

2. CRITERIOS DE LA SUMA DE LAS CIFRAS

Otras veces la suma de las cifras indica si se es o no divisible por otro. Es el caso de los

conocidos criterios para saber si un número es divisible por 3 o por 9, cuando est escrito en

base decimal. La siguiente proposición justifica‚ estos y otros casos.

Page 219: A Notas de Algebra Moderna

Proposición 2. Sea a=(anan-1...a1a0)b si b≡1(mod c) entonces a es divisible por c si y solo

si

a0 + a1 + a2 + ... + an ≡0(mod c)

Demostración. Sabemos que

a=a0 + a1b + a2b2 + ... + anbn

y como b≡1(mod c), aplicando aritmética de congruencias se obtiene que

a0 ≡ 0(mod c) a1b ≡ a1(mod c)

a2b2 ≡ a2(mod c) anbn ≡ an(mod c)

entonces sumando estas congruencias tenemos

a≡a0+a1+a2+...+an(mod c),

que en combinación con el lema 1 nos proporciona el resultado deseado.

Presentamos las aplicaciones de esta proposición como corolarios:

Corolario 1. Si b es congruente con 1 módulo c y el número a este escrito en base b,

entonces a es congruente con la suma de sus cifras módulo c.

Corolario 2. Si b es congruente con 1 módulo c, entonces para saber si un número (escrito

en base b) es divisible por c es suficiente saber si la suma de sus cifras lo es.

Ejemplo 3. (Base decimal) La suma de las cifras del entero 19168639 es 43 por tanto:

19168639≡1(mod 3)

19168639≡7(mod 9)

Ejemplo 4. En base 4, la suma de las cifras del entero a es congruente con a módulo 3

pero no módulo 9.

Page 220: A Notas de Algebra Moderna

3. CRITERIOS DE LA SUMA Y RESTA DE LAS CIFRAS

Empecemos mostrando un ejemplo:

Ejemplo 5. Para saber si un número escrito en base decimal es divisible por 11, se halla

la diferencia entre la suma de cifras de lugares pares y la suma de las cifras de lugares

impares; el número es múltiplo de 11 , si y solo si, esta diferencia lo es.

Digamos, para saber si 19168639 es divisible por 11, sumamos las cifras de lugares

impares: 9+6+6+9=30 ; sumamos las cifras de lugares pares: 3+8+1 +1=13; hacemos la

diferencia de estas dos sumas: 30-13=17. Como 17 no es múltiplo de 11 entonces

19168639 no es múltiplo de 11. Es mas, como se ve en la demostración de la siguiente

proposición se tiene que

17≡19168639(mod 11).

Nótese que si a=(an, an-1,...,a1, a0)b la diferencia entre la suma de cifras de lugares pares y la

suma de las cifras de lugares impares viene dada por la expresión:

a0-a1+a2-...+(-1)nan

Proposición 3. Sea a=(an, an-1,...,a1, a0)b si b≡-1(mod c) entonces a es divisible por c si y

solo si

a0-a1+a2-...+(-1)nan≡0(mod c)

Demostración. Ejercicio (es similar a la demostración de la proposición 2).

Corolario 1. Si b es congruente con -1 módulo c, entonces para saber si un número

(escrito en base b) es divisible por c es suficiente saber si la suma de cifras de lugares pares

menos la suma de las cifras de lugares impares es múltiplo de c.

Page 221: A Notas de Algebra Moderna

4. CRITERIOS CON PAREJAS Y TRIPLAS DE CIFRAS

Algunas veces es conveniente considerar las cifras de un número tomadas de dos en dos, o

de tres en tres (siempre de derecha a izquierda). Aquí realmente, se está pasando el número

a base b2 o b3, como se resalta en el siguiente lema cuyo enunciado y demostración se dejó

como ejercicio en sección anterior.

Lema 2. Las cifras de a escrito en base bm son las mismas que en base b pero tomadas de

derecha a izquierda en grupos de m: Cada grupo de m cifras en base b corresponde a un

dígito en base bm.

Ejemplo 6. Para pasar de base 2 a base 8 el entero (10010101110001110)2 traducimos

los grupos de 3 así:

(110)2=6; (001)2=1;

(110)2=6; (101)2=5;

(010)2=2; (10)2 =2;

entonces las cifras en base 8 son 6, 1, 6, 5, 2, 2 (tomadas de derecha a izquierda) es decir:

(10010101110001110)2=(225616)8.

Si quisiéramos pasar a base 16 escogeríamos grupos de a 4 y tendríamos:

(1110)2=E ; (1000)2=8; (1011)2=B; (0010)2=2; (1)2=1 y tenemos:

(10010101110001110)2= (12B8E)16.

Cuando la base es muy grande podríamos agotar las letras del alfabeto, entonces no se

acostumbra colocar nuevas letras, sino dejar los dígitos decimales. Por ejemplo para pasar

de base decimal a base 100, se necesitaría agregar 90 nuevos dígitos, mejor entender las

parejas de dígitos decimales como dígitos centesimales, así:

(19168639)10 = (19-16-86-39)100

Page 222: A Notas de Algebra Moderna

El aplicar el lema 2 con alguna de las proposiciones 1, 2 o 3, es un m‚todo para conseguir y

explicar otros criterios de divisibilidad, como se muestra en los siguientes ejemplos:

Ejemplo 7. (Base decimal) Para saber si un entero es múltiplo de 100, todos sabemos que

basta con saber si sus dos últimas cifras son exactamente 00. Claro! Cuando escribimos

los números en base 100 los múltiplos de 100 son los que terminan en cero, pero este dígito

se representa con dos ceros de la base decimal.

Por otra parte, aplicando la proposición 1 al lema 2 obtenemos que por ejemplo cualquier

entero es congruente modulo 25 con sus dos últimas cifras ya que 25 divide a 100.

Otra manera de saber si un número escrito en base 10 es divisible por 11 es hacer la suma

de sus cifras tomadas de dos en dos. Por ejemplo:

19168639≡19+16+86+39=160≡60+1≡6(mod 11).

Compare con el ejemplo 5!

Ejemplo 8. Busquemos un criterio para saber si un entero escrito en base 2 es múltiplo de

3. Como 4≡1(mod 3) entonces en base 4 se puede aplicar la proposición 2 y tenemos que

un número escrito en base 4 es congruente módulo 3 con la suma de sus cifras (ejemplo 4)

pero según el lema 2 las cifras de base 4 son las parejas de cifras en base 2, entonces: "Un

número escrito en base 2 es múltiplo de 3 si y solo si la suma de sus cifras tomadas de dos

es dos de derecha a izquierda es múltiplo de 3".

5. SEPARANDO LAS ÚLTIMAS CIFRAS

El operador "quitar" la última cifra, o las dos últimas, parece que no fuera un operador

aritmético. Según el Teorema fundamental de la numeración, y trabajando en base decimal,

si la última cifra de n es a, entonces n tendrá la forma

n=10n’+a

Page 223: A Notas de Algebra Moderna

con 0<a<10, en donde n' es precisamente “lo que queda”. Si lo que se quita a n son las dos

últimas cifras, entonces convendrá considerar la forma de n como

n=100n'+a

en donde a es el valor de las dos últimas cifras (0<a<100) y n' es “lo que queda”.

Utilizando este sencillo operador, podemos encontrar útiles criterios de divisibilidad, que se

justifican por alguna ecuación de congruencias.

Ejemplo 9. Un número escrito en base 10 es divisible por 17, si y solo si, al quitar sus dos

últimas cifras y restarlas del duplo de lo que queda el resultado es múltiplo de 17.

Por ejemplo, para saber si 4767 es múltiplo de 17: quito 67 y lo que queda es 47, su duplo

94, menos 67, obtengo 27, que no es múltiplo de 17 por tanto 4767 tampoco lo es, pero si

pruebo con 4267 tengo (42×2)-67 =84-67=17, y por tanto 4267 si es múltiplo de 17.

La demostración de que este procedimiento es válido, parte de considerar n con la forma

n=100n'+b , donde n' es lo “que queda” y b es el número representado por las dos últimas

cifras, debemos ver que n≡0(mod 17) si y solo si, 2n'-b≡0(mod 17) y esta es una

equivalencia de congruencias, que se demuestra fácilmente:

n=100n'+b≡0(mod 17) ⇔ 15n'+b ≡0(mod 17) ⇔ -2n'+b≡0(mod 17) ⇔ 2n'-b≡0(mod 17)

PREGUNTAS Y EJERCICIOS.

Demostrar los siguientes criterios de divisibilidad:

1. En base diez un número es divisible por 2, si termina en cifra par.

2. En base diez un número es divisible por 5, si termina en 0 o 5.

Page 224: A Notas de Algebra Moderna

3. En base diez un número es divisible por 20, si termina en 00, 20, 40, 60 ó 80.

4. En base 12 un número es divisible por 3, si su última cifra lo es.

5. En base 12 un número es divisible por 6, si termina en 0 ó 6.

6. En base 2 los múltiplos de 4 son aquellos que terminan en 00.

7. En base diez un número es divisible por 9, si la suma de sus cifras es divisible por 9.

8. En base 6 un número es divisible por 5, si la suma de sus cifras es divisible por 5.

9. En base diez un número es divisible por 37, si la suma de sus cifras tomadas de tres en

tres, es divisible por 37.

10. En base 4 un número es divisible por 15, si la suma de sus cifras, tomadas de os en dos,

es divisible por 15.

11. Si a un número que está escrito en base 10 y es múltiplo de 13, se le quita su última

cifra y se le suma multiplicada por 4 a lo que queda, el resultado es múltiplo de 13.

12. Si a un número que está escrito en base 10 y es múltiplo de 11, se le quita su última

cifra y se le suma a lo que queda, el resultado es múltiplo de 11.

13. Enuncie y demuestre un criterio para saber si un número escrito en base decimal es

divisible por 19.

Page 225: A Notas de Algebra Moderna

14. TODO DEPENDE DE SABER CONTAR

Nos proponemos analizar los algoritmos para efectuar operaciones aritméticas. Otra vez

nuestro interés más que práctico es teórico: El análisis de diferentes algoritmos nos

proporcionar para los números, sus relaciones y operaciones. Buscando lo fundamental

vemos que lo mas elemental que podemos hacer es contar. Sabiendo contar, es decir

sabiendo sumar 1, podemos realizar los algoritmos más comunes de la aritmética: Sumar

dos números, restarlos, multiplicar, hallar cociente y residuo, etc.

Para ver esto imaginemos que se deja programar, pero de aritmética no sabe sino sumar 1.

Por dejarse programar entendemos que guarda números en memoria, que se nota en letras

mayúsculas, asignar 6 a la memoria A, se notará

6 → A

asignar a la memoria B, lo que est en la memoria A así

Determinar si los contenidos de las memorias A y B coinciden y según la respuesta seguir

en procedimiento, lo que se notará así

Para presentar resultado se utilizar el símbolo

y para finalizar el signo

A → B

A = B Si

No

A ES EL RESULTADO

PARE

Page 226: A Notas de Algebra Moderna

Las asignaciones se enumeran en un rectángulo. Si suponemos que nuestra máquina sólo

suma 1, el siguiente algoritmo representa un algoritmo que muestra los números de dos en

dos, hasta llegar a m-2.

N es la variable donde se lleva la cuenta, se inicia en 0, y luego de presentarse se

incrementa en 1 dos veces, para ese N ¿M volverse a presentar y seguir sucesivamente

hasta que N sea igual a M, pero nótese que si en la variable M va un número impar el

algoritmo no se detiene. ¿Cómo hacer para que el proceso se detenga siempre que M sea

entero positivo?

El primer método que un niño conoce para sumar es contar con los dedos de las manos.

Para 3 y 6, se cuenta a partir de 6 y en una mano se lleva la cuenta de 1 incremento, cuando

esa mano se lleve 3 entonces el proceso se detiene. Este algoritmo es representado por el

siguiente diagrama en el cual se suma a y b.

Los sumandos a y b son guardados en las memorias s y t, las memorias s y MANO se van

incrementando de 1 en 1 hasta que MANO alcance a T, cuando esto suceda, la suma de los

dos números es lo que est en la memoria 5.

Para hacer el seguimiento de cualquier algoritmo se puede hacer lo que se llama "una

Prueba de Escritorio", que es construir una tabla que muestre el valor de las diferentes

PARE 0 → N N N+1 →N N+1 →N N = M Si

No

a → S b → T 0 → MANO

MANO=T Si

No

S+1 → S MANO+1 → MANO

a+b=5 PARE

Page 227: A Notas de Algebra Moderna

variables; una vez dados los datos iniciales. Para el ejemplo la tabla 1 muestra una prueba

de escritorio cuando? Sumar 3 y 6.

S T MANO MANO=T

6 3 0 NO 7 1 NO 8 2 NO 9 3 SI

Tabla 1.

Por otra parte debemos observar que esta acertada manera de sumar con los dedos de las

manos, está basada en la deficiencia recursiva de suma (a partir de sumar 1). Ese efecto

recursivamente se puede definir n+m así:

i) n+0=n

ii) n+(m+1)=(n+m)+1

Para sumar 6+3 según esta definición, por i) saben que 6+0=6 por ii) saben que

6+1=6+(0+1)=(6+0)+1=7, otra vez aplicando ii) obtenemos 6+2:

6+(1+1)=(6+1)+1=7+1=8

y así:

6+3=6+(2+1)=(6+2)+1=8+1=9

El algoritmo que acabamos de mostrar es una demostración de la siguiente proposición.

Proposición 1. Una máquina que sólo suma 1, puede sumar cualquier par de números

naturales.

La máquina que pensamos ni siquiera decide cúal es el mayor de un par de números. Ella

sólo sabe decir cuándo un par de números coinciden. Como ejercicio el lector debe

programarla para que decida cúal de los números naturales a y b es el mayor. Se trata de ir

calculando en memorias auxiliares incrementos de cada una de los números hasta que el

uno alcance el otro. Esto demuestra la siguiente proposición.

Page 228: A Notas de Algebra Moderna

Proposicion 2. Una máquina que sólo suma 1, puede programarse para que decida cúal

de los números es mayor. También se puede programar para hallar la diferencia.

Las operaciones que hace nuestra máquina las efectúa con cantidades positivas, pero se

pueden ampliar fácilmente para que trabaje con negativos. Sin embargo seguiremos

trabajando sólo en números naturales.

Las dos proposiciones anteriores no permiten que de ahora en adelante que nuestra máquina

ya puede sumar cualquier par de números naturales. Hallar la diferencia del mayor al

menor y decidir cúal es el más grande, es decir podemos hacer asignaciones del siguiente

tipo: A+B → C, A-B → C y tomar décimas de tipo

Con estos procedimientos describiremos un algoritmo para que dados a y b, enteros

positivos, encuentre el cociente al dividir a entre b.

La base de este algoritmo es supremamente sencilla; el método más fácil para repartir A

objetos entre B personas, es darle un objeto a cada persona (se resta B de A), luego vuelve a

repartir una a cada uno (resta B del RESTO) y así hasta lo que quede (EL RESTO) sea

A ≥ B Si

No

A > B Si

No

A : B A>B

A=B

a → A a → RESTO b → B 0 → C

RESTO<B Si

No

C+1 → C RESTO-B → RESTO

Cociente de dividir a entre b

PARE

Page 229: A Notas de Algebra Moderna

menor que el mínimo de personas (RESTO<B). El cociente es lo que a cada persona le

toco!

Ahora debemos enseñarle a multiplicar, expresar en base b un número, saber si tal número

es primo, etc. Pero esto lo dejamos como ejercicio para el lector.

EJERCICIOS

1. Suponga que la máquina C, que sólo suma uno, sin ninguna subrutina; hacer programas

para:

a. Muestre los primeros n múltiplos de 2.

b. Muestre los primeros n múltiplos de 3.

c. Determine si n es par o impar.

d. Halle el cociente y el residuo al dividir en 2.

e. Halle el cociente y el residuo al dividir en 3.

f. Muestre los números de la forma 4k+1.

g. Halle la suma de los n primeros números.

h. Dados a y b encuentre |a-b|.

i. Dados a y b encuentre a÷b que es a-b si a>b y cero en caso contrario.

2. Suponga una máquina C, que tiene subrutinas para sumar cualquier par de números y

decidir cúal de los dos es el mayor además de restar de un mínimo otro menor o igual.

Diseñar un algoritmo para que la máquina:

a. Dado a y b encuentre el residuo de dividir a entre b.

b. Multiplique a y b.

c. Halle el máximo común divisor de a y b ( se puede hacer sólo restando).

d. Dados n, k, r

( ) ( ) ( ) ( )∑=

+++++++=+n

inkrkrkrrrik

0...2

e. Dados m y n positivos decidir si n|m.

f. Decidir si n es primo.

Page 230: A Notas de Algebra Moderna

g. Encuentre n2.

3. Supóngase ahora que nuestra máquina tiene subrutinas para las operaciones siguientes:

Suma, resta, producto, cociente y residuo (se notan COC (a,b), RES (a,b)). Diseñar

algoritmos para que la máquina:

a. Calcule n!

b. Calcule a-6.

c. Exprese a en base b.

d. Exprese (a,b) como combinación lineal de a y b.

e. Exprese n como producto de factores primos.

4. ¿Qué hace el siguiente algoritmo?

0 → S a → A b → B

RES(4,2)=0 No

Si

COC(4.2) → A B × A → B

S=B+S

PARE

A=0

S

Page 231: A Notas de Algebra Moderna

15. ALGORITMOS BASADOS EN EL SISTEMA DE NUMERACION.

Los algoritmos expuestos en la sección anterior trabajan con números en "abstracto" por

cuanto en ellos nada tiene que ver la representación de los números. Esta es una de las

razones por la que esos algoritmos resultan ser tan poco prácticos, ya que una operación

con números moderadamente grandes supone mucho tiempo de computación para ser

llevada a cabo.

Cuando los números están escritos en determinado sistema de numeración se utilizan

algoritmos muchos más rápidos que los expuestos anteriormente y son estos algoritmos los

que se utilizan para hacer las operaciones básicas tanto manualmente como cuando se

trabaja con aparatos de computación.

El sistema de numeración en base diez, o sistema induarábico, se impuso definitivamente

en nuestra civilización precisamente por ser mas práctico especialmente cuando se trabaja

la multiplicación y la división, como ya se dijo en la sección 14. La palabra 'algoritmo'

apareció en Europa conjuntamente con la aceptación del sistema induarábigo, para referirse

a los métodos de cómputo de los calculistas de "pluma y papel'.

Hoy en día, con la aparición de los ordenadores la palabra algoritmo se refiere a cualquier

procedimiento ejecutable por la máquina, pero los cálculos sobre números que hace un

ordenador se siguen basando en algoritmos parecidos a los manuales pero diseñados para el

sistema de numeración posicional binario.

Estos ser n los algoritmos que trabajaremos en esta sección. Algoritmos rápidos, que se

utilizan para hacer las operaciones manualemente o por computador. Nuestro interés no es

que el lector llegue a manejar estos algoritmos con destreza, lo que perseguimos es que se

Page 232: A Notas de Algebra Moderna

logre descifrar el por qué de su efectividad, lo que a la larga puede dar la oportunidad de

diseñar algoritmos propios.

Empezamos con la descripción de un método muy antiguo empleado para multiplicar, que

identificaremos como método de multiplicación de los campesinos rusos, que además de

ser práctico, evita el uso de las tablas de multiplicar, pues solo exige saber sumar y saber

multiplicar y dividir por dos. Para multiplicar dos números se dice que los campesinos

rusos colocaban los dos números un al lado del otro y mientras uno se va dividiendo por

dos el otro se va multiplicando por dos. Las divisiones se hacen, naturalmente enteras y los

números no pares de esta columna se distinguen digamos con un * . El proceso termina

cuando en la columna de los números que se van dividiendo se encuentre 1. El resultado

del producto se encuentra sumando los números correspondientes a * en la columna de los

números que se van multiplicando por 2. En la tabla 1 se ilustra el proceso

cuando se trata de multiplicar 312 por 45.

El lector puede comprobar con otras multiplicaciones que realmente este método es

efectivo, pero el reto que se plantea es explicar el por qué. Es decir, se quiere una

demostración de que el método sí funciona (ejercicio 6). Por ahora como una pista digamos

312 45 156 90 78 180

* 39 360 * 19 720 * 9 1440 4 2880 2 5760

* 1 11520 312×45=360+720+1440+11520

=14040 Tabla 1. Método de multiplicación de los campesinos rusos aplicado para efectuar 312×45. En la columna de la izquierda se hacen sucesivas divisiones por 2 a partir de 312 (* denota que la división no es exacta). En la otra columna se hacen sucesivas multiplicaciones por 2 a partir del otro multiplicando que es 45. El producto se obtiene sumando los valores de la columna derecha correspondientes a *.

Page 233: A Notas de Algebra Moderna

que aunque este procedimiento es válido no importa en que sistema de numeración se

trabaje, su fundamentación radica en la expresión de uno de los multiplicandos en base 2.

Ahora queremos revisar los algoritmos usuales aprendidos en la escuela primaria para

efectuar las operaciones básicas. Empecemos por imaginarnos cómo se suma y se resta en

sistemas aditivos, por ejemplo en el sistema egipcio explicado en la sección 13. La suma y

la resta en estos sistemas es realmente sencilla y es de suponer que se actuaba tal como lo

hacían los abaquistas, o hoy en día, cualquier tendero que quiere contar su dinero. Se trata

de unir los signos similares y cuando se obtienen suficiente número de ellos se cambian por

otro que represente un número mayor. Veamos el mismo ejemplo para sumar 5 y 9 que se

mostró en la sección 10:

///)(////////////////

/////////∩∩ ==+=+

Cuando se completan 10 palotes se reemplazan por el numeral correspondiente ∩ . Como

cuando se cambian 10 monedas de $1 por una de $10! El lector debe practicar otras sumas

en éste y otros sistemas aditivos. Es al fin y al cabo un ejercicio de tenderos.

Ahora bien, cuando se suma en sistemas posicionales el principio es el mismo: Si

trabajamos en base 10, reunimos primero las unida des, si ellas sobrepasan a 10, guardamos

una decena, así reunimos luego las decenas, siempre que encontremos diez decenas

guardamos una centena y así sucesivamente. La razón por la cual cuando suma mos las

cifras de un columna si la suma pasa de diez "llevamos" a la siguiente columna, lo que

estamos haciendo es cambiar billetes de una denominación por otro de una denominación

mas alta.

El argumento también es válido para bases diferentes de diez. Si se trabaja en base b, para

sumar dos números se van sumando sus cifras correspondientes y cuando la suma

sobrepasa b se "lleva 1" a la columna siguiente. Así, si vamos a sumar (312)5 y (33)5

Page 234: A Notas de Algebra Moderna

podemos colocar los números como si fuéramos a sumarlos en sistema induarábigo y

sumamos columna por columna, de derecha a izquierda, teniendo en cuenta que cuando la

suma sobrepase 5 escribimos la suma menos 5 y "llevamos 1" a la siguiente columna.

40033

312

Al sumar los números de la primera columna de la derecha, las unidades, obtenemos

2+3=5, pero el dígito 5 no existe, en base 5 es 10 por lo tanto se escribe 0 y se lleva 1.

Entonces en la segunda columna sumamos 3 y 1 y el 1 que llevábamos, obtenemos de

nuevo 10 (o sea 5 ), escribimos 0 y llevamos 1 a la tercera columna.

Ahora describiremos este algoritmo de manera mas precisa pensando en una máquina que

recibe números hasta de n+1 dígitos y trabaja en base b. El fundamento del algoritmo lo

especificamos en la siguiente proposición:

Proposición 1. Si a=(an an-1 ... a0)b y entonces a+b=(cn+1 cn ... c0)b donde los ci se

calculan así: r0=0, si ai+bi+ri<b entonces ci=ai+bi+ri y ri+1=0

si ai+bi+ri>b entonces ci=ai+bi+ri-b y ri+1=1 cn+1=rn+1.

Demostración. (Ejercicio).

Los ri indican lo que se lleva de la anterior columna. Claro que si la máquina recibe sólo

números de n+1 cifras cuando rn+1=1 esto significa que la suma ha rebosado la capacidad

de la máquina y se debe emitir una señal de error.

Supongamos que la máquina trabaja con b=2 como en realidad sucede con los modernos

computadores. Un sumador lo podemos ver como una caja negra (en cuanto no nos

interesa cómo actúa internamente) que recibe tres entradas, dos por donde llegan los

números que se van a sumar dígito a dígito y otra por donde se retroalimenta con lo que

"lleva". El sumador tiene además dos salidas: una por donde indica los dígitos de la suma y

otra que indica cuanto lleva y retroalimenta al sumador pasando por un "delay" que retiene

Page 235: A Notas de Algebra Moderna

la señal por un "instante" (estos aparatos funcionan controlados por un reloj que emite

pulsos en intervalos de tiempo muy pequeños).

La figura 1 muestra un esquema del sumador en donde ai y bi son los dígitos de los

números a sumar, ri es lo que se lleva y si indica los dígitos de la suma.

Los ai y bi entran externamente, mientras los ri son producidos por el mismo sumador salvo

r0 que es 0. Insistimos en que el fundamento de este sumador es el mismo que el de la

suma usual y que se expone en la proposición 1. En cierto sentido el sumador sabe sumar

dos cifras en base 2. Esto es muy fácil y se lleva a cabo según se indica la tabla 2.

Recomendamos al lector hacer un análisis similar cambiando la operación suma por la

resta. Habrá que elaborar y probar una proposición análoga a la proposición 1 para en base

a ello describir el método general de restar (que es válido en cualquier base), y luego

elaborar una tabla análoga a la tabla 2 que nos indique cómo funcionaría un "restador

secuencial".

Sumador

Delay

ai bi

ri

si ri+1

Figura 1. Diagrama de un sumador secuencial

ai bi ri si ri+1 0 0 0 0 0 0 0 1 1 0 0 1 0 1 0 1 0 0 1 0 0 1 1 0 1 1 0 1 0 1 1 1 0 0 1 1 1 1 1 1

Tabla 2. Salidas de acuerdo a las entradas del sumador secuencial.

Page 236: A Notas de Algebra Moderna

La multiplicación usual. Sabemos ya que el algoritmo usual que se utiliza para sumar es

válido en cualquier base. La primera pregunta que nos proponemos responder es la

siguiente: Es válido el algoritmo usual de la multiplicación en cualquier base? Antes de

responder esta pregunta deberemos describir generosamente este algoritmo. Se distinguen

dos casos :

Multiplicación de cualquier número por otro de una sola cifra y multiplicación de dos

números cualesquiera.

En cualquier caso el primer paso consiste en memorizar unas odiosas tablas de multiplicar.

Consiste en aprender "de memoria" (o guardar en memoria ?) el producto de todos los

números de una sola cifra. En base 10 debemos aprender 81 resultados!

Para la multiplicación de cualquier número por otro de una sola cifra el procedimiento

secuencial es parecido al de la suma: se va multiplicando cifra por cifra y se "lleva" las

decenas para irlas acumulando con el siguiente producto. Recordemos: 234×6: 6 por 4 da

24 escribe 4 lleva 2; 6 por 3 da 18 y 2 que se llevaba son 20, escribe 0 y lleva 2; 6 por 2 da

2 y 2 que se llevaba son 14, como no hay más cifras escribe 14.

Sabiendo que para multiplicar por la 10 simplemente se agrega un cero la fundamentación

del algoritmo radica en la ley distributiva. Para el ejemplo, el procedimiento quedaría claro

si lo escribieramos así:

234×6 = (2×100 +3×10+4) ×6 = (2×100 +3×10) ×6+4×6 = (2×100 +3×10) ×6+2×10+4 = (2×100) ×6+(3×6+2) ×10+4 = (2×6×100)+(2×10+0) ×10+4 = (2×6+2) ×100)+0×10+4 = (1×10+4)×100+0×10+4 = 1×1000+4×100+0×10+4 = 1404

Page 237: A Notas de Algebra Moderna

Este procedimiento que indudablemente es mas largo pero en él todo queda justificado, lo

denominaremos multiplicación PASO A PASO.

Proposición 2. Si k=(anan1...a0)b y t es un número de una sola cifra entonces

kt=(cn+1cn...c0)b donde los ci se calculan así: r0=0, si ait+ri=(ef)b entonces ci=f y ri+1=e

Demostración. (ejercicio).

Mas importante que la demostración de la proposición 2, es comprender su significado:

describe y justifica el algoritmo usual para multiplicar un número cualquiera por otro de

una sola cifra. Para visualizar esto volvamos al ejemplo de la multiplicación de 234 por 6.

En este caso según la notación de la proposición 2, b=10, c0=4, c1=3, c2=2 y t=4. La

siguiente tabla muestra el procedimiento, que es el mismo utilizado anteriormente en donde

los ri indican lo que se "lleva":

i ai ri (6ai+ri) ri+1 ci 0 4 0 24 2 4 1 3 2 20 2 0 2 2 2 14 1 4 3 0 1 2 0 1

La proposición 2 además implica que el algoritmo para multiplicar en base 10 por un

número de una sola cifra es válido para cualquier base. Trabajemos por ejemplo en base 5.

La tabla de la suma y el producto se muestran en la tabla 3.

Hallemos el producto de (323)5 y 4 tendríamos: b=5, a0=3, a1=2, a2=3, t=4. Deseamos

hallar, según la proposición 2, los ci.

+ 1 2 3 4 1 2 3 4 102 3 4 10 113 4 10 11 124 10 11 12 13

× 2 3 4 2 4 11 13 3 11 14 22 4 13 22 31

Tabla 3. Suma y producto en base 5

Page 238: A Notas de Algebra Moderna

i ai ri (ait+ri)b ri+1 ci 0 3 0 22 2 2 1 2 2 20 2 0 2 3 2 24 2 4 3 0 2 2 0 2

Tenemos entonces que (323)5×4=(2402)5, como prueba podemos pasar a base 10:

(323)5=3×25+2×5 +3=(88)10 y (2402)5=2×125+4×25+2=(352)10, y en efecto sabemos que

88×4=352.

El lector deber ejercitarse en la multiplicación de un número cualquiera por otro de una sola

cifra para cualquier base. Esperamos que así quede explicado totalmente este caso.

Para la multiplicación de dos números cualesquiera el algoritmo usual es también válido

para cualquier base. Para multiplicar, por ejemplo, 342 por 135, primero multiplicamos 5

por 342, luego 3 por 342 y colocamos este producto un lugar corrido a la izquierda,

finalmente 342 por 1 y colocamos el resultado dos lugares hacia la izquierda para así hacer

la suma que nos proporciona el resultado.

4617034210261710

135342×

Lo aparentemente extraño en este procedimiento es por qué se corren los resultados a la

izquierda. Esto queda claro si lo escribimos así:

342×5 = 1710 342×30 = 10260

342×100 = 34200 342×135 = 46170

Page 239: A Notas de Algebra Moderna

Este procedimiento se deriva entonces de la aplicación de la ley distributiva del producto y

del hecho que al multiplicar por 10 (la base) se agrega un cero. Es fácil ver que entonces

nuestro procedimiento es válido en cualquier base. Ahora bien en base dos se tiene la gran

ventaja de que no toca aprender tablas de multiplicar, basta con saber sumar, pues

multiplicar por 0 y por 1 es trivial.

LA DIVISION USUAL: El algoritmo utilizado usualmente para dividir es de todos

indudablemente el más confuso. Se invita al lector a que haga una prueba no matemática:

Pregúntele a diez personas de nivel universitario el por qué hace las divisiones de la forma

corriente, y si alguno le explica satisfactoriamente el por qué, regálele mil pesos. Seguro

no gastar mucha plata!

Nos limitaremos a traducir una división corriente por la misma pero paso a paso, con la

esperanza de que el lector dilucide qué es lo que se hace para poder diseñar una forma de

escribir las divisiones sin que se pierda la esencia de lo que se hace. Pensemos en dividir

3472 entre 32:

3472 32 -32 108

272 -256

16

Ahora observemos la misma división PASO A PASO:

3472 = 3400+72 = 100×32+200+72 = 100×32+272 = 100×32+8×32+16 = (100+8)×32+16 = 1080×32+16

Recordemos primero que cuando se trata de dividir a entre n lo que se busca es un cociente

q y un residuo r tal que a=bn+r y que 0<r<n. Esto es fácil y rápido si los números son de

magnitudes parecidas es decir cuando el cociente no es muy grande, por ejemplo para

Page 240: A Notas de Algebra Moderna

dividir 272 entre 32 se puede ir multiplicando, 32 por 1 luego por 2 y así hasta que el

producto supere a 272.

Este método se sugirió en la sección anterior. Pero para dividir 3472 entre 32 habría que

hacer entonces 108 productos, cosa nada practica ! Según se ve en la división paso a paso

del ejemplo, lo que se hace es primero dividir 3400 entre 3200 (aparece 1 al cociente) luego

se divide 270 entre 320 (aparece el 0 del cociente) y finalmente 272 entre 32 (aparece el 8

del cociente y el residuo final).

Para la división el algoritmo usual también es válido cuando se trabaja con otras bases.

Dividamos por ejemplo (2324)5 entre (13)5.

Por comodidad omitamos la referencia a la base para este ejemplo. Los primeros múltiplos

de 13 en base 5 son: 13, 31, 44, 112; empezamos dividiendo 2300 entre 1300 nos da 1 y

sobra 1000; ahora debemos dividir 1020 entre 130, da 3 y nos sobra 1020-440=30.

Finalmente dividimos 34 entre 13 no da 2 y sobra 3. Esta división paso a paso la

escribimos así:

2324 = 2300+24 = (13×1×100+1000)+24 = 13×1×100+1020+4 = 13×1×100+(13×3×10+30)+4 = 13×1×100+13×3×10+34 = 13×1×100+13×3×10+13×2+3 = 13×(1×100+3×10+2)+3 = 13×132+3

Y resumidamente según el algoritmo usual, la operación se puede escribir así:

2324 13 102 132 34 3

Page 241: A Notas de Algebra Moderna

COMENTARIO FINAL.

El lector después de una lectura superficial a esta sección, puede pensar que lo que se ha

hecho es complicar cosas que para él estaban muy claras. El primer avance es descubrir

que los algoritmos usuales que se enseñan en la escuela primaria para las operaciones

básicas no son tan claros, en especial el caso del producto y la división. Esto se entiende

por cuanto estos algoritmos se impusieron por razones prácticas, como ya se dijo, y para los

calculistas antiguos de pluma y papel, omitir los ceros, por ejemplo en el caso de la

multiplicación, significaba ahorro de tiempo y tinta (que era muy costosa). Hoy en día,

estas razones no son v lidas y el maestro del siglo XXI deber explicar estos algoritmos

buscando primero su comprensión antes que la rapidez y efectividad en los cálculos.

Queda pues el reto pedagógico de diseñar métodos para efectuar los mismos algoritmos de

tal manera que el alumno no pierda de vista el sentido de lo que se est haciendo.

PREGUNTAS Y EJERCICIOS

1. Efectuar en la base indicada:

a. (231123)4+(322122)4

b. (A2B3)16-(BB1)16

c. (10110011000)2- (11101)2

2. Para multiplicar un número por 5 se le agrega un cero y se divide por 2. Demostrar que

esta regla es válida cuando se trabaja en base decimal. Es v lida esta regla en otras

bases?

3. a. Exprese un algoritmo para sumar d días, h horas, m minutos con d' días, h' horas, m'

minutos.

b. Exprese un algoritmo para restar d días, h horas, m minutos con d' días, h' horas, m'

minutos.

Page 242: A Notas de Algebra Moderna

4. Demostrar que si a=(anan-1...a0)b entonces ab=(anbnan-1...a0)b. Esto demuestra que para

multiplicar un número escrito en base b por b simplemente se agrega un 0.

5. Al multiplicar un múltiplo de 3 menor que 30 por 37 se obtiene siempre como producto

un número de 3 cifras iguales (base decimal). ¿Explique por qué?

6. Explicar el por qué del método de multiplicación de los campesinos rusos. (Ayuda:

Exprese en base 2 el multiplicando que se va dividiendo ).

7. Demostrar la proposición 1 por inducción sobre n el número de cifras de los sumandos

(ayuda: Si a=(anan-1...a0)b entonces a=anbn+(an-1...a0)b ) .

8. Una docena son 12 unidades y una grueza son 12 docenas. Usando aritmética en base

12 resolver las siguientes preguntas:

a. Si 3 gruezas,5 docenas y 8 huevos se sustraen te un total de 9 gruesas y 2 docenas,

cuántos huevos quedan.

b. Si un supermercado recibe 3 cajas de remesas de huevos cada una de 3 gruesas 5

docenas y 8 huevos, cuantos huevos recibió en total.

c. Si 11 gruesas, 10 docenas y 6 huevos se dividen en tres grupos de igual tamaño, con

cuantos huevos queda cada grupo?

9. Efectuar las siguientes operaciones en la base indicada:

a. (231123)4×(3)4 b. (A2B3)16×(3)16

c. (10110011000)2×(11101)2 d. (2313)4×(21)4

e. (10110011000)2÷(11101)2 f. (1123)4÷(12)4

10. Efectuar las operaciones de los ejercicios 2 y 9 ,paso a paso.

Page 243: A Notas de Algebra Moderna

11. (Base decimal).

a. Demostrar que si se toma un número de 3 cifras no repetidas y se suma con otro con

las mismas 3 cifras pero en sentido contrario, y se resta del mayor el menor, se

obtiene siempre un número de tres cifras cuyas cifras de los extremos suman 9 y la

intermedia es 9.

b. Si a,b,c son tres dígitos tales que a+c=9 y b=9 entonces demuestre que

(abc)10+(cba)10=1089

c. Lo anterior justifica el siguiente acertijo:

Piense un número de tres cifras cualesquiera (ejemplo 347) sin divulgarlo tome el

número con las cifras contrarias (743) y reste del mayor el menor (743-347=396), el

resultado súmele el mismo número pero de cifras contrarias (396+693=1089) siempre

se obtiene 1089 .

12. Para elevar al cuadrado un número que escrito en base decimal termina en 5 (por

ejemplo 145) se toma el numero que queda al omitir el 5 (es decir 14) y se multiplica

por su siguiente (14×15=210) al resultado se le adjunta 25 y ese es el cuadrado del

número ( 1452=21025 ).

a. Demostrar que esta regla es válida.

b. Enuncie y demuestre una regla similar para elevar al cuadrado números que, escritos

en base 2b, terminen en b.

13. Análogo al sumador secuencial que se presentó en esta sección:

a. Construir un "sustractor secuencial" para números escritos en base 2.

b. Construir un "duplicador secuencial" para números escritos en base decimal.

14. Para multiplicar en base decimal se puede utilizar la siguiente regla:

"Sumar a cada dígito, si es par la mitad de su vecino a la derecha, si es impar sumarle 5

además ( las mitades se consideran en su parte entera y el vecino del primer dígito, de

derecha a izquierda, es el 0)"

Page 244: A Notas de Algebra Moderna

Por ejemplo para multiplicar 152 por 6

2 no tiene vecino a la derecha y es par, pasa 2

5 su vecino es 2, mitad 1, 5+1=6 como 5 es impar se le agrega

además 5, obteniéndose 11, llevamos 1 y pasa 1

1 su vecino es 5, mitad 2, suma 7 mas lo que llevábamos nos da

8 y como 1 es impar se suman 5 obteniendo 13

el producto es…………………………………............................................1312

a. Describir formalmente el algoritmo.

b. Explicar formalmente por qué funciona (Ayuda: Nótese que multiplicar por 6 es

multiplicar por 5+1. Aproveche el ejercicio 2 de esta sección).

15. Diseñe un método para dividir dos números que sea el inverso de la multiplicación de

los campesinos rusos.

16. Demostrar la proposición 2 por inducción sobre n el número de cifras del número de

varias cifras que se va a multiplicar (ayuda: Si a=(anan-1...a0)b entonces

a=anbn+(an-1...a0)b

Page 245: A Notas de Algebra Moderna

" A parte de las imágenes que

es todo lo que podemos ver realmente, imaginemos un mundo de cosas sólidas;

y ..., este mundo está constituido de tal manera que cumple un cierto

código de reglas, algunas llamadas axiomas, otras definiciones, otras postulados y

algunas admitidas en el curso de la demostración ..."

WILLIAM K. CLIFFORD

(1845-1879)

A1. EL MÉTODO AXIOMÁTICO.

Hemos visto anteriormente algunas demostraciones. Se hace una demostración para

despejar dudas sobre una proposición viendo que ella se deriva de otras de las cuales no se

tienen dudas. Pero “tener dudas” es algo relativo. Podría suceder lo que pasa ante un niño

que pregunta y al obtener la respuesta hace otra pregunta y así indefinidamente. Lo mismo

sucede con las definiciones. Se define un término conocido en base a otros ya conocidos,

pero estos a su vez, para no dejar dudas, necesitan ser definidos y así sucesivamente.

¿Hasta cuándo?

Parece que las explicaciones de la vida terminan en círculos viciosos. (Terminan?)

Históricamente, hasta donde se conoce, los griegos fueron los que hicieron demostraciones

matemáticas. Pitágoras o alguno de sus alumnos, demostró el teorema que lleva su nombre,

ya conocido por civilizaciones anteriores como la de babilonia; los pitagóricos y otras

escuelas lograron otras demostraciones tanto de proposiciones de la geometría como de la

aritmética. Fue Euclides quien elaboró un cuerpo admirablemente armónico de todos sus

conocimientos en sus famosos “ELEMENTOS DE GEOMETRÍA” en donde todas las

proposiciones eran demostradas a partir de otras que ya lo habían sido (TEOREMAS) o que

eran aceptadas desde un principio como verdades (AXIOMAS)

Page 246: A Notas de Algebra Moderna

Los “Elementos de Geometría ” de Euclides forman una de las obras más importantes de

toda la historia y por más de veinte siglos son la base de la enseñanza de la geometría y la

matemática en el mundo occidental.

El método axiomático desarrollado por los griegos, esto es, a partir de verdades aceptadas e

indiscutibles para que por medio únicamente de raciocinios lógicos se lleguen a otra s no

triviales, se convierte en la máxima aspiración de filósofos y científicos de casi todas las

épocas, algunas veces con gran éxito como en el caso de la mecánica de Newton.

Pero la geometría de Euclides tuvo su gran crisis en el siglo XIX cuando la discusión sobre

el quinto postulado(*) desemboca en la construcción que hacen Lobachewsky y Bolyai y

más tarde Riemman, de geometrías que contradicen dicho postulado y conservan todo su

rigor lógico. Surgen pues, muchas inquietudes que obligan a replantear ¿QUÉ EL

MÉTODO AXIOMÁTICO? ¿Cuál es la “verdadera geometría”? La que adopta como

axiomas las “verdades evidentes?” Pero, las “verdades evidentes” son claramente relativas

y no siempre son verdades universales pues en principio, por ejemplo, es evidente que la

tierra es plana y que el sol gira alrededor de ella, pero sabemos que esto no es así.

Hoy día el método axiomático sigue siendo tan importante como antes aunque en las épocas

modernas la concepción general sobre dicho método ha cambiado.

Hay dos diferentes que resaltamos: Primero, la teoría que resulta susceptible de ser

aplicada no sólo a una situación particular sino a todas aquellas donde los axiomas,

dándoles cierta interpretación, se cumplen; en estos casos se dice que se tiene un MODELO

para la teoría. Segundo, los axiomas ya no se entienden con verdades autoevidentes por sí

mismas, sino que se consideran simplemente como proposiciones de las que se parte para

de mostrar todo lo demás. Casi siempre se trabaja con referencia a un modelo, por lo que

hay que tener sumo cuidado pues, puede haber verdades “evidentes” según el modelo, pero

que necesitan ser demostradas por todos los modelos posibles.

Page 247: A Notas de Algebra Moderna

Ahora bien, cuando se trabaja con pocos axiomas loa posible modelos son generalmente

muchos. A medida que se agregan axiomas el número de modelos va disminuyendo hasta

que se reduce básicamente a uno. En este caso se han logrado definir los elementos de

dicho modelo, de manera implícita, pues no se ha dicho que son, cual es su naturaleza, sino

que se han dado ciertas propiedades entre ellos que en fin de cuentas los caracterizan. Esto

es lo que haremos a continuación para captar el concepto de número entero

axiomáticamente. Empezaremos mencionando los axiomas para dominios enteros que se

refieren a las operaciones de suma y producto (+,·), pero, para este sistema hay muchos

modelos: los enteros, los polinomios, las clases residuales, los números racionales, los

reales, etc.

Entonces introduciremos unos axiomas que se refieren al orden hasta que básicamente no

encontramos sino un modelo: Los números enteros.

Esperamos que esta introducción sirva al lector para que no se extrañe al demostrar

cuestiones que para él siempre han sido evidentes como que el cero multiplicado por

cualquier número siempre es cero, o que el uno (1) es el primer entero positivo.

Demostraremos precisamente todas estas propiedades básicas de los enteros.

Page 248: A Notas de Algebra Moderna

A2. AXIOMAS DE LOS ENTEROS

No definiremos que es un número entero, ni la suma ni la multiplicación entre ellos.

simplemente diremos que los enteros están dotados de estas operaciones (+,·) las cuales

caracterizamos por cumplir los axiomas. Además aceptamos la existencia de dos enteros 0

y 1 (diferentes) de manera que se cumple:

AXIOMAS ALGEBRAICOS

Siendo a, b, c, d enteros cualesquiera se cumple:

1. CLAUSURA: a=c y b=d ⇒ (a+b)=(c+d)

2. ASOCIATIVIDAD: (a+b)+c=a+(b+c)

(a·b)·c=a·(b·c)

3. MODULATIVA: a+o=a y a·1=a

4. INVERSOS ADITIVOS: Existe (-a) tal que a+(-a)=0

5. CONMUTATIVA: a+b=b+a)

a·b=b·a

6. DISTRIBUTIVA: a·(b+c)=a·b+a·c

7. CANCELATIVA PARA EL PRODUCTO: Si a≠0 a·b=a.c⇒b=c

Estos axiomas los cumplen los enteros, como es fácil ver; pero hay otros objetos, sistemas,

que también los cumplen. Cuando un conjunto (no importa la naturaleza de sus elementos)

dotado de dos operaciones (no importa lo que ellos signifiquen) cumplen los axiomas

algebraicos anteriores se llama DOMINIO ENTERO o un dominio de integridad (Integral

Domain). A continuación mostraremos algunos ejemplos:

Page 249: A Notas de Algebra Moderna

Ejemplo 1. El dominio entero más pequeño en el que se puede pensar debe tener por lo

menos dos elementos: el uno y el cero. Sea A={0,1} y definamos las operaciones + y ·

como se indican en los cuadros:

Entonces se verifican todos los axiomas. La clausura se garantiza por la manera como se

han definido las operaciones. La conmutativa para + se ve haciendo tadas las

combinaciones posibles, se ve que

0+1=1=1+0

1+1=0=1+1

0+0=0=0+0

Así se prueba todos los axiomas

Ejemplo 2. Sea A={a,b,c,d} y se definen las operaciones así

En este caso como en el anterior, hay que demostrar cada propiedad observando la

posibilidad. Vemos que el papel del 0 es jugado por a mientras que b hace el papel del 1,

pues son módulos para el producto y la suma, respectivamente. Se ve, pues que se cumplen

todas las propiedades a excepción de la cancelativa para el producto. En efecto la

cancelativa exige que si x≠0 para todo y, z se tiene que xy=yz debe obligar y=z. En nuestro

caso c≠a (hace el papel de 0) y se tiene cb=cd sin embargo b≠d.

Ejemplo 3. Los números reales R y los racionale Q son como los enteros, dominios

enteros. Es más, R y Q tienen propiedades que no tiene Z (¿cuáles?).

+ 0 1 0 0 1 1 1 0

· 0 1 0 0 0 1 0 1

+ a b c d a a b c d b b c d a c c d a b d d a b c

· a b c d a a a a a b a b c d c a c a c d a d c b

Page 250: A Notas de Algebra Moderna

Ejemplo 4. Sea A el conjunto de polinomios con coeficientes reales, con la suma y

multiplicación corrientes. A forma un dominio entero. El papel de 0 y 1 lo hacen los

polinomios constantes de valores 0 y 1 respectivamente.

PROPIEDADES

Proposición 1. El "0" es único como módulo de la suma.

Demostración: Supongamos que existe otro elemento, llamémoslo 0' que se comporta

como módulo, o sea para todo x∈Z se cumple que:

x+0'=x.

En especial para 0 y aplicando conmutativa se tendrá

0'+0=0. (1)

y como 0 es módulo se tendrá

0'+0=0'.

que en combinación con (1) por clausura, obliga

0=0'. g

Proposición 2. El "1" es único como módulo del producto.

Demostración: (Ejercicio)

Proposición 3. El universo aditivo es único.

Demostración: Supongamos que (-a) y a' son inversos aditivos de a por lo tanto se deben

cumplir:

(-a)+a=0 (1)

a'+a=0 (2)

De (2) por clausura:

(a'+a)+(-a)=0+(-a)

Page 251: A Notas de Algebra Moderna

Como 0 es módulo y asociando a la izquierda tenemos:

a'+(a+(-a))=(-a)

Pero: (a+(-a))=0

Entonces: a'=-a.

Proposición 4. a·0=0

Demostración: Por modulativa 0+0=0, y multiplicando por a La clausura asegura que

a(0+0)=a·0

Vemos que a·0 está actuando como módulo aditivo por la Proposición 1 se tiene que a·0=0

Proposición 5. (-1)a=-a

Demostración: La idea es ver que (-1)a es el inverso aditivo de a. Calculemos pues

(-1)a+a por clausura como 1a=a tenemos:

(-1)a+a=((-1)+1)a.

Pero

(-1)+1=1+(-1)=0

Por lo tanto

(-1)a+a=0· a

Y la proposición 4 nos asegura que

(-1)a+a=0 (1)

lo cual por proposición 3 nos indica que

-a=(-1)a

Pues (-1)a según (1) actúa como módulo.

Proposición 6. -(-a)=a.

Page 252: A Notas de Algebra Moderna

Demostración: Se trata de ver, apoyándonos de nuevo en la proposición 3, que a actúa

como inverso aditivo para (-a) y esto es claro pues

a+(-a)=(-a)+a=0

Proposición 7. (-1)·(-1)=1

Demostración: (Ejercicio)

Proposición 8. (-a)(-a)=a·a

Demostración: (Ejercicio)

Proposición 9. La ecuación x+b=c, cualesquiera que sean b y c, tiene una única solución:

Demostración : Si hacemos

x=c+(-b)

y reemplazando en la ecuación vemos que este valor la satisface. Esto asegura que existe al

menos una solución.

Veamos que ésta es la única. Supongamos otra, o sea x' se cumple también:

x'+ b=c

Entonces por clausura:

(x'+b)+(-b)=c+(-b)

asociando en el miembro de la izquierda:

x'+(b+(-b)=c+(-b)

Pero como b+(-b)=0 aplicando modulativa se obtiene

x'=c+(-b)

Page 253: A Notas de Algebra Moderna

o sea x'=x lo que indica que la solución es única.

NOTACION: Se notará a-b al elemento a+(-b).

Proposición 10. En un dominio entero no hay divisores de cero, es decir, si ab=0 entonces

se debe tener que, o bien a=0, o bien b=0.

Demostración: Esto es una consecuencia del axioma 7, o sea la ley cancelativa del

producto. En efecto, supongamos que a·b=0 y a≠0. Por lo tanto se tiene

a·b=a·0

Aplicando el axioma debemos tener que

b=0

De la misma forma vemos que si a·b=0 y b=0 entonces a≠0.

Se ve entonces que siempre que un producto sea nulo, uno de sus factores debe ser nulo.

EJERCICIOS

1. Sea A={0,1,2} indicar si los axiomas de dominio entero se cumplen para las

operaciones definidas como indican las tablas

a)

b)

¿Quién hace el papel de 1?

+ 0 1 2 0 0 1 2 1 1 2 0 2 2 0 1

· 0 1 2 0 0 0 0 1 0 1 2 2 0 2 2

+ 0 1 2 0 0 1 2 1 1 2 0 2 2 2 0

· 0 1 2 0 0 0 0 1 0 1 2 2 0 2 2

Page 254: A Notas de Algebra Moderna

c)

2. Sea x un conjunto y A el conjunto de todos los subconjuntos de x. Entonces la ∪ e ∩ ;

son operaciones definidas en A. Entendiendo la ∪ como la suma y la ∩ como

producto, ¿Cuáles axiomas de dominio entero se cumplen?

3. Complete la tabla de multiplicación para que el conjunto A={0,1,2,3,4} forme un

dominio entero

NOTA: 4 juega el papel de módulo para el producto. Para averiguar, por ejemplo,

cuanto es 2·3 se sabe que 4+4 = 3 entonces

2·3=2(4+4)=2·4+2·4=2+2+4

4. Demuestre la Proposición 2.

5. Demuestre la Proposición 7.

6. Basándose en la Proposición 7 demostrar 8.

7. Demuestre que en un dominio entero si a, b, c, d son elementos cualesquiera, se

cumplen las siguientes igualdades (justifique cada paso):

a) a·(b-c)=a·b-c·d

b) (a-c)+(c-d)=(a-d)

+ 0 1 2 0 0 1 2 1 1 2 0 2 2 0 1

· 0 1 2 0 0 1 2 1 1 1 2 2 2 2 2

+ 0 1 2 3 4 0 0 1 2 3 4 1 1 2 3 4 0 2 2 3 4 0 1 3 3 4 0 1 2 4 4 0 1 2 3

+ 0 1 2 3 4 0 0 0 0 0 0 1 0 2 0 3 0 4 0 1 2 3 4

Page 255: A Notas de Algebra Moderna

c) (a+b)·(a+c)=a·a+(b+c)·a+b·c

d) (a-b)

e) (a-b)·(a+b)=(a·a)-(b·b)

f) (a-b)·(c-d)=(a.c+b.d)-(b.c+a.d)

8. Demuestre que si en un dominio entero x+a = x+b entonces a = b (cancelativa suma).

9. Se define a2=a·a demuestre que (a+b)2=a2+2ab+b2 y que (a-b)2.

10. Diga dónde está el error de la siguiente demostración:

"Como 6=6 y 2=2 puedo decir que 6+2 = 6+2 lo cual ímplica que 6-6 = 2-2 y se tiene

3(2-2)=1(2-2) de donde 3=1"

11. En Z se define:

a⊕b=a+b-1 a⊗b= a.b - (a + b) + 2

Demostrar que (Z, ⊕, ⊗) también forma un dominio entero en donde el módulo para +

es 1. ¿Cuál es el módulo para el producto?

12. Demostrar que los números reales de la forma a+b 2 de donde a, b son enteros

forman un dominio entero.

13. Entre las matrices 2X2 con coeficientes reales está definida la suma y la multiplicación,

sin embargo, no se forma un dominio entero. (¿Porqué?)

14. Demuestre que las matrices de la forma ⎟⎟⎠

⎞⎜⎜⎝

⎛ −abba

reales sí forman un dominio entero.

Page 256: A Notas de Algebra Moderna
Page 257: A Notas de Algebra Moderna

A3. EL PRINCIPIO DEL BUEN ORDEN

Como vimos en la sección anterior Z es un dominio entero ordenado, pero no es único, pues

existen otros radicalmente diferentes. En esta sección introducimos un nuevo axioma

llamado "Principio de Buen Orden" y desarrollamos sus consecuencias.

Este axioma caracteriza a los enteros en cuanto el único dominio entero ordenado que lo

cumple es Z (salvo notación). No podemos mostrar ejemplos radicalmente distintos, pues

como se puede demostrar (1) básicamente no hay sino un ejemplo. Otro problema es saber

si con estos axiomas podemos demostrar cualquier proposición que sea cierta en Z. La

sorprendente respuesta a este problema fué dada por G del alrededor de 1930 y se la

dejamos como inquietud al lector. Antes de enunciar el citado axioma, damos otra

definición que corresponde a la intuición.

Definición 1. Sea A un subconjunto de un dominio entero ordenado. Se dice que a es el

primer elemento de A. sí y sólo sí, a∈A y a< x para todo x∈A.

Es claro que si un conjunto tiene primer elemento, es único. Por otra parte hay

subconjuntos no vacíos de un dominio entero que no poseen primer elemento, considérese

por ejemplo el conjunto de números pares como subconjunto de Z.

Por las razones explicadas anteriormente, habiendo aceptado este axioma sólo nos

referíamos a Z; tenemos pues que todo subconjunto no vacío de enteros positivos tiene un

primer elemento, en especial los enteros positivos deben tener un primer elemento.

Proposición 1. El primer elemento de los enteros positivos es 1.

AXIOMA DEL BUEN ORDEN:

Todo subconjunto no vacío de positivos tiene un

primer elemento.

Page 258: A Notas de Algebra Moderna

Demostración. Supongamos que existe un elemento c positivo que está antes que 1, a sea

que:

0<c<1

Consideremos el conjunto cuyos elementos estén entre 0 y 1. Llamémoslo A. Entonces:

A es un subconjunto de enteros positivos.

A es no vacío, pues hemos supuesto que c∈A.

Por el principio de buen orden A debe poseer un primer elemento, llamémoslo a. Como

a∈A entonces:

0<a<1

Aplicando propiedades de orden (ejercicio 3f. de la sección anterior) tenemos:

0<a2<a

Luego aparece otro elemento, a2, que pertenece a A y es menor que a, lo cual es una

contradicción pues a es el primer elemento de A.

Concluimos entonces que A debe ser vacío, o lo que es equivalente, que no hay enteros

menores que 1.

Corolario. Si a es entero, entonces entre a y a+1 no hay ningún entero.

Demostración. Supongamos que hay un entero c que está entre a y a+1. Tenemos:

a<c<a+1

Sumando (-a) a estas dos desigualdades nos queda

0<c-a<1

Y como c-a es un entero, se está contradiciendo la proposición 1 nos indica que tal c no

existe.

Proposición 2. (Principio de inducción) Sea A un subconjunto de enteros positivos tal

que:

i) 1∈A

Page 259: A Notas de Algebra Moderna

ii) Si x∈A⇒(x+1)∈A

En estas condiciones A coincide con los enteros positivos.

Demostración. Sea B el conjunto de enteros positivos que no están en A.

Si B es no vacío, por el principio de buen orden debe tener un primer elemento llamémoslo

b. ¿Qué ocurre con b-1? Es entero y positivo pues b=1 ya que 1∈A; entonces b-1 puede

pertenecer o no a A. Analicemos ambos casos:

a. Si b-1 es elemento de A por la hipótesis ii) se tiene que (b-1)+1∈A o sea que b∈A lo

cual es imposible pues b∈B.

b. Si b-1 no es elemento de A pertenece a B, pero b-1< b entonces b no sería el primer

elemento de B, contrario a lo supuesto.

Ambas proposiciones nos lleva a negar la existencia de b y la única posibilidad es que B sea

vacío o sea que A comprende todos los enteros positivos.

Corolario. Sea A un conjunto y a un entero tal que:

i) a∈A

ii) Si k∈A se puede asegurar que a+1∈A entonces A comprende todos los enteros mayores

que a.

Demostración. Sea B={x-a+1| x∈A} demuéstrese por inducción que N⊆B. Sea y entero

tal que y>a entonces y-a+1∈N, lo que implica que (y-a+1)∈B o sea y∈A.

Otra forma del principio de inducción se da en el ejercicio 4. El principio de inducción

también sirve para definir funciones evaluadas en números enteros a partir de k:

Proposición 3. (Definiciones Recursivas) Si se conoce el valor de f(k) y si conociendo el

valor de f(n) se puede conocer el valor de f(n+1), entonces se conocen los valores de f para

todo entero mayor o igual a k.

Page 260: A Notas de Algebra Moderna

Demostración. (Ejercicio)

El lector debe estar familiarizado tanto en las demostraciones por inducción como con las

definiciones recursivas; aquí ilustraremos estas aplicaciones con la demostración de un

hecho muy sutil con el cual estamos tan familiarizados que no se ve generalmente la

necesidad de su demostración. El axioma de asociatividad que se dió para dominios enteros

se enuncia para tres ejemplos e intuitivamente se dá uno cuanto que es válida para n

elementos o sea que en una suma de n elementos se pueden eliminar los paréntesis (por esta

razón, se puede hablar de "una suma de n elementos" no así de "una resta de n elementos").

Demostraremos formalmente este hecho. Primero definíamos lo que sería

(...((a1+a2)+a3)+...)+an-1+an

para a1, a2,..., an-1, an elementos de un dominio entero.

Definición. Sean a1, a2,..., an,... elementos de un dominio entero. Se define:

i) S1=a1

ii) Si+1=Si+ai+1

Así mismo definimos S -r- para k > r -r+1-

i) 1+rrS =ar+1

ii) Sr= 1+krS +ak+1

Nota. Obsérvese que intuitivamente

Si = (...(a1+a2)+a3)+...+ai-1)+ai

y krS = (...((ar+1+ar+2)+...+ak-1)+ak con (k>r)

Si se define por inducción sobre i y Sk por inducción sobre k.

Page 261: A Notas de Algebra Moderna

Proposición 4. (Ley Asociativa Generalizada) En las condiciones de la definición anterior

se tiene que para todo n≥3 y para todo k<n, k y n enteros positivos se cumple: -n-

Sk+ nrS =Sn

Demostración. Haremos inducción sobre n:

i) Para n=3 tenemos:

S1=a1; S2=a1+a2; S3=(a1+a2)+a3

Si k=1: 21S =a2 y 3

1S =a2+a3

Sk+ nkS =S1+S1=a1+(a2+a3)

Y aplicando asociativa al último término de ésta ecuación vemos que

Sk+ nkS =(a1+a2)+a3

ó sea

Sk+ nkS =Sn

Si k=2 compruébelo usted mismo. Es mas fácil!

ii) Supongamos ahora que el teorema es válido para n es decir que para 0<k<n se cumple

Sk+ nkS =Sn

Pero 1+npS = n

pS +an+1 por definición y así mismo

Sn+1=Sn+an+1

entonces

Sp+ 1+npS = n

pS +( npS +an+1)=(Sp+ n

pS )+an+1

Aplicando la hipótesis de inducción al último miembro de la igualdad vemos que:

Sp+ 1+npS =Sn+an+1

Lo que implica que

Sp+ 1+npS =Sn+1

Completando la demostración.

Page 262: A Notas de Algebra Moderna

EJERCICIOS

1. Definir Ultimo Elemento de A para A un subconjunto de un dominio entero ordenado.

2. Mostrar un subconjunto de los reales positivos, no vacío, que no tenga primer elemento.

3. Usando el principio del buen orden probar que si k>0 y a es cualquier entero, entonces

existe un n∈Z tal que nk<a<(n+1)k.

4. Use el hecho de que Z está bien ordenado para probar que la siguiente forma del

principio de inducción es correcta: "Supongamos que a cada entero positivo n está

asociado una proposición Pn Entonces Pn es cierta para cada entero positivo n si

cumple:

i) P1 es cierta

ii) Si k es un entero positivo tal que Pi es cierta para todo i<k, entonces Pk también es

cierta". Esta forma del principio de inducción se llama "Principio de Inducción

completa"

5. Completar la demostración del corolario de la proposición 2.

6. Demuestre la proposición 3.

7. Enuncie y demuestre la ley distributiva generalizada en un dominio entero.